Ứng dụng định lý Viète trong các bài toán số học

Tài liệu gồm 38 trang được sưu tầm và tổng hợp bởi các tác giả Doãn Quang Tiến và Nguyễn Minh Tuấn, giới thiệu cho bạn đọc một số các bài toán số học có sử dụng định lý Viète (Vi-ét) và nâng cao hơn nữa là phương pháp bước nhảy Viète (Vieta Jumping) để giải quyết các bài toán số học hay và khó.

ỨNG DỤNG ĐỊNH LÝ VIÈTE
TRONG C BÀI TOÁN SỐ HỌC
Tạp c và liệu toán học sưu tầm và tổng hợp
Hướng tới VMO 2020
Ngày 14 tháng 12 năm 2019
Tóm tắt nội dung
Trong chương trình toán lớp 9, chúng ta đã được tìm hiểu đến một định cực nối tiếng đó
định Viète, tuy nhiên ứng dụng của không chỉ biểu diễn mối quan hệ của các nghiệm
trong phương trình đa thức, còn ứng dụng trong nhiều mảng khác như số học, đa thức,... bài
viết này, chúng tôi sẽ giới thiệu cho bạn đọc một số các bài toán số học sử dụng định Viète và
nâng cao hơn nữa phương pháp bước nhảy Viète - Vieta Jumping - để giải quyết các bài toán số
học hay và khó. Trong bài viết tham khảo các liệu trong và ngoài nước, các bạn xem mục
tài liệu tham khảo cuối bài viết. Mọi ý kiến thắc mắc, đóng góp vui lòng gửi về địa chỉ.
1. Doãn Quang Tiến. fb.com/profile.php?id=100016406718327
2. Nguyễn Minh Tuấn. fb.com/tuankhmt.fpt
1 Nhà toán học Francois Viète.
Francois Viète (1540-1603) nhà toán học Pháp đại. Ông người đầu tiên đưa ra các hiệu bằng
chữ, do thế, người ta gọi ông người cha của môn Đại số. Tên tuổi của ông gắn liền với một định v
nghiệm số của phương trình học sinh lớp 9 đều biết đó định Viète, nhưng công lao của ông to
lớn hơn nhiều.
Ông vốn một trạng sư, từng làm “cố vấn mật” cho các triều vua Henry III và Henri IV. Giữa
những bộn rộn của công việc cung đình, hễ ít phút rảnh rỗi ông lại giải trí bằng cách... nghiên
cứu Toán học! Trong cuộc chiến tranh Pháp... Tây Ban Nha thời y, quân Tây Ban Nha thường liên
lạc với những k nội phản trong nước Pháp bằng các mật thư. được viết bằng các mật gồm
toàn các chữ số, nên các mật thư ấy hầu như không thể khám phá được.
Biết vị “cố vấn” Viète thích toán, vua Henry III đã nhờ ông thử tìm “chìa khóa” các mật thư
y. Nhận lời, suốt hai tuần lễ, ông làm việc quên ăn quên ngủ. Cuối cùng, chính Viète đã tung
tấm màn mật: ông đã tìm ra quy luật thay thế các chữ và số trong cách viết mật thư. Đọc được
các mật thư, quân Pháp đã làm thất bại hoàn toàn những mưu đồ của Tây Ban Nha. V phía địch,
chúng gắng tìm nguyên nhân: cuối cùng chúng biết được những hiệu đã bị phơi trần, nhiều
lần thay đổi mật mã, và kẻ tìm ra mật Francois Viète! Quân Tây Ban Nha tuyên bố Viète
k tử thù và đã xử án hỏa thiêu vắng mặt ông, nhưng bản án man đó không bao giờ thực hiện
được. Không chỉ quan tâm sâu sắc đến Đại số; nghiên cứu các phương trình, Viète còn nghiên cứu
cả Hình học và Lượng giác. Ông cũng đã khảo cứu lưỡng nhiều công trình của các nhà toán học thời cổ.
Phần lớn cuộc đời của Viète bị các công việc pháp của nghề trạng chiếm mất nên khó thể
tưởng tượng ông đã lấy đâu ra thời gian để làm nên những công trình toán học của mình. quyết
của ông chính khả năng tập trung cao độ khi làm việc. Người ta còn kể lại, lúc gặp đươc một vấn đề
thú vị, ông thể ngồi bàn làm việc suốt ba ngày đêm liền.
1
Hướng tới VMO 2020 Tạp chí và liệu toán học
2 Định Viète.
Định Viète được trình y trong sách giáo khoa toán 9 - tập 2, cho ta mối quan hệ giữa các nghiệm
của phương trình bậc hai và các hệ số của nó. Sau đây ta sẽ nhắc lại nó.
Định Viète.
Nếu phương trình bậc hai
ax
2
+
bx
+
c
= 0
(a 6= 0)
hai nghiệm
x
1
và
x
2
thì
tổng và tích của chúng
S = x
1
+ x
2
=
b
a
P = x
1
.x
2
=
c
a
Ngược lại nếu hai số
x
1
và
x
2
thỏa mãn
S
=
x
1
+
x
2
và
P
=
x
1
.x
2
thì
x
1
và
x
2
hai nghiệm
của phương trình t
2
St + P = 0.
Chú ý rằng trong khi giải toán, đôi khi ta không quan tâm tới giá trị của
x
1
và
x
2
chỉ cần quan
tâm đến 2 giá trị tổng và tích của chúng, từ đó ta những đánh giá cần thiết. Ngoài ra cũng từ định
Viète ta nhận thấy nếu một phương trình bậc hai
ax
2
+
bx
+
c
= 0 một nghiệm
x
1
thì sẽ
thêm một nghiệm
x
2
nữa. Ngoài ra ta thể mở rộng định cho phương trình đa thức bậc
n
bất kì.
Cho phương trình
a
0
+ a
1
x + a
2
x
2
+ ... + a
n
x
n
= 0, a
n
6= 0
. Gọi
x
1
, x
2
, . . . , x
n
n
nghiệm của phương
trình trên, khi đó thì
a
0
+ a
1
x + a
2
x
2
+ ... + a
n
x
n
= a(x x
1
)(x x
2
)...(x x
n
)
Nhân toàn bộ vế phải ra, chúng ta sẽ công thức Viète, được phát biểu như sau
a = a
n
a(x
1
+ x
2
+ ... + x
n
) = a
n1
. . .
. . .
(1)
n1
a(x
1
x
2
...x
n1
+ x
1
x
2
...x
n2
x
n
+ ... + x
2
x
3
...x
n
) = a
1
(1)
n
a(x
1
x
2
...x
n
) = a
0
và trong hàng
k
bất kỳ, vế phải của đẳng thức
a
nk
a
nk
còn vế trái được tính theo công thức
(1)
k
a nhân với tổng của các tích từng cụm (n k) các nghiệm của phương trình trên.
3 Các bài toán bản.
Sau đây ta sẽ đi tìm hiểu một vài dụ trước khi đi tìm hiểu v phương pháp bước nhảy Viète.
Bài toán 1.
Tìm tất cả các giá trị của
m
để phương trình
x
2
mx
+
m
+ 2 = 0 các nghiệm
đều nguyên.
Lời giải
Lời giải 1.
Điều kiện để phương trình nghiệm =
m
2
4
(m + 2) >
0. Phương trình đầu
nghiệm nguyên thì phải số chính phương, tức tồn tại số nguyên
k
sao cho
m
2
4
(m + 2)
=
k
2
.
Ta
m
2
4 (m + 2) = k
2
(m 2 + k) (m 2 k) = 12
Đến đây ta một c ý rằng, theo định Viète, ta
x
1
+
x
2
=
m
, do vậy
m
sẽ số nguyên. Từ
đây do m và k các số nguyên nên ta tìm được m = 2 hoặc m = 6. Ta xét các trường hợp.
1.
Với
m
=
2 thì ta được phương trình
x
2
+ 2
x
= 0, khi đó tìm được hai nghiệm nguyên
x
1
= 0
và x
2
= 2.
2
Chinh phục olympic toán Ứng dụng định Viète trong các bài toán số học
2.
Với
m
= 6 thì ta được phương trình
x
2
6
x
+ 8 = 0, khi đó ta được hai nghiệm nguyên
x
1
= 2
và x
2
= 8.
Như vậy đến đây bài toán đã được giải quyết.
Lời giải 2. Theo định Viète với hai nghiệm x
1
; x
2
thì ta
x
1
+ x
2
= m
x
1
.x
2
= m + 2
(1)
Do đó nếu ta tìm được các nghiệm nguyên của phương trình thì ta sẽ tìm được giá trị của
m
. Điều này
làm ta ý tưởng giải phương trình nghiệm nguyên 2 ẩn x
1
và x
2
. Từ (1) ta được
x
1
.x
2
(x
1
+ x
2
) = 2 (x
1
1) (x
2
1) = 3
Khi đó
x
1
1 và
x
2
1 các ước của 3, lại 3 = 1
.
3 =
1
. (3)
. Đến đây việc tìm hai nghiệm
x
1
;
x
2
hoàn toàn đơn giản và qua đó ta tìm được các giá trị m m = 2 và m = 6.
!
Nhận xét.
lời giải 1 sẽ một số học sinh mắc sai lầm chưa chỉ ra được
m
số nguyên
đề bài không đề cập. Do đó ta phải sử dụng định Viète để chỉ ra điều y.
Bài toán 2. Tìm nghiệm nguyên của hệ phương trình
xy + yz + zx = 8
x + y + z = 5
Lời giải
Trước tiên ta sẽ đưa bài toán về việc giải phương trình nghiệm nguyên. Ta biến đổi
xy + (y + x) z = 8
x + y = 5 z
xy + (5 z) z = 8
x + y = 5 z
xy = 8 (5 z) z
x + y = 5 z
Theo định Viète thì
x
và
y
hai nghiệm của phương trình bậc hai
t
2
(5 z) t
+ 8
(5 z) z
= 0,
trong đó z tham số. Phương trình nghiệm thì > 0 hay ta được
3z
2
10z + 7 6 0 1 6 z 6
7
3
Do z nguyên nên z = 1 hoặc z = 2. Ta xét các trường hợp sau.
Với
z
= 1, khi đó phương trình trở thành
t
2
4
t
+ 4 = 0, đến đây ta tìm được
x
=
y
= 2 thỏa
mãn.
Với
z
= 2 khi đó phương trình trở thành
t
2
3
t
+ 2 = 0, đến đây ta tìm được
x
= 2;
y
= 1 hoặc
x = 1; y = 2 thỏa mãn.
Vậy hệ phương trình các nghiệm nguyên (x; y; z) = (2; 2; 1) , (1; 2; 2) , (2; 1; 2).
Bài toán 3.
Giải phương trình
x
2
mx
+
n
= 0, biết phương trình hai nghiệm nguyên dương
phân biệt và m, n hai số nguyên tố.
Lời giải
Với bài toán y nếu dùng công thức nghiệm để xác định nghiệm của phương trình thì sẽ y cho ta
nhiều khó khăn do phương trình đến hai tham số. Do vậy ta sẽ sử dụng định Viète. Gọi
x
1
;
x
2
các nghiệm nguyên dương của phương trình đã cho
(x
1
< x
2
)
. Ta biết rằng một số nguyên tố khi viết
thành tích hai số thì một thừa số 1 và một thừa số chính nó. Để ý ta lại thấy theo định Viète
thì
x
1
x
2
=
n
. Do đó rất tự nhiên ta nghĩ đến sử dụng định Viète giả quyết bài toán. Thật vy theo
định Viète ta được
x
1
+ x
2
= m
x
1
x
2
= n
3
Hướng tới VMO 2020 Tạp chí và liệu toán học
Do
n
một số nguyên tố nên ta được
x
1
= 1;
x
2
=
n
, suy ra
m
=
n
+ 1, do vậy
m
và
n
hai số tự
nhiên liên tiếp nên ta được n = 2; m = 3, thử lại ta thấy tỏa mãn yêu cầu bài toán.
Bài toán 4.
Cho phương trình 2
x
2
+
mx
+ 2
n
+ 8 = 0, trong đó
m
và
n
các tham số nguyên.
Giả sử phương trình các nghiệm đều số nguyên. Chứng minh rằng m
2
+ n
2
hợp số.
Lời giải
Để chứng minh được
m
2
+
n
2
hợp số thì một suy nghĩ hết sức tự nhiên đó y dựng biểu thức
m
2
+
n
2
theo các nghiệm của phương trình đề rồi từ đó phân tích biểu thức nghiệm thành nhân tử.
hai ý tưởng để xây dựng biểu thức
m
2
+
n
2
đó áp dụng công thức nghiệm để tìm các nghiệm của
phương trình rồi từ đó tính
m
2
+
n
2
hoặc áp dụng định Viète. ràng trong hai ý tưởng đó việc áp
dụng định Viète giúp ta xây dựng biểu thức nghiệm không chứa các căn bậc hai.
Gọi x
1
, x
2
hai nghiệm của phương trình trên, theo định Viète ta được
(
x
1
+ x
2
=
m
2
x
1
.x
2
= n + 4
Khi đó ta
m
2
+ n
2
= (2x
1
+ 2x
2
)
2
+ (x
1
x
2
4)
2
= 4x
2
1
+ 4x
2
2
x
2
1
+ x
2
2
x
2
1
+ 16 =
x
2
1
+ 4
x
2
2
+ 4
Do x
1
; x
2
các số nguyên nên x
2
1
+ 4; x
2
2
+ 4 các số nguyên dương lớn hơn 1.
Từ đó ta được m
2
+ n
2
hợp số.
!
Nhận xét. Xét 2m = a + b; 2n = a b, khi đó từ m
2
+ n
2
=
a
2
+ b
2
2
, ta bài toán sau.
Bài toán.
Cho phương trình 4
x
2
+
(a + b) x
+ 2
(a b)
+ 16 = 0, trong đó
a
và
b
các tham số
nguyên. Giả sử phương trình các nghiệm đều số nguyên, chứng minh rằng
a
2
+ b
2
2
hợp số.
Bài toán tương tự.
Giả sử phương trình bậc hai
x
2
+
ax
+
b
+ 1 = 0, trong đó
a, b
các tham
số nguyên, đồng thời
b 6
=
1 hai nghiệm đều số nguyên khác 0, chứng minh rằng
a
2
+
b
2
hợp số.
Bài toán 5.
Tìm các số nguyên dương
a
và
b
, trong đó
(a > b)
sao cho phương trình bậc hai
x
2
abx + a + b = 0 các nghiệm đều số nguyên.
Lời giải
Điều kiện để phương trình nghiệm =
(ab)
2
4
(a + b) >
0, giả sử phương trình hai nghiệm
nguyên x
1
; x
2
(x
1
6 x
2
), khi đó theo định Viète ta được
x
1
+ x
2
= ab
x
1
.x
2
= a + b
Do
a
và
b
các số nguyên dương nên suy ra các nghiệm
x
1
;
x
2
cũng các số nguyên dương. Ta c ý
rằng với hai số lớn hơn 2 thì tích của chúng bao giờ cũng lớn hơn tổng của chúng. Do đó ta nghĩ đến
chứng minh một trong bốn số dương trên không vượt quá 2.
Thật vậy, nếu a > 2; b > 2 thì ta ab > 2a; ab > 2b nên 2ab > 2 (a + b) hay ab > a + b.
Nếu cả bốn số dương
x
1
;
x
2
;
a
;
b
đều lớn hơn 2 thì
x
1
.x
2
> x
1
+
x
2
và
ab > a
+
b
, khi đó định Viète
trên không thể xảy ra. Như vậy trong bốn số dương
x
1
;
x
2
;
a
;
b
tồn tại ít nhất một số không vượt quá
2. Theo giả thiết và theo cách chọn hai nghiệm
x
1
;
x
2
thì trong hai số
x
1
và
b
ít nhất một số không
lớn hơn 2. Do vai trò của hai số
x
1
và
b
như nhau nên không mất tính tổng quát ta giả sử rằng
0 < x
1
6 2, đến đây ta xét từng trường hợp của x
1
.
4
Chinh phục olympic toán Ứng dụng định Viète trong các bài toán số học
1.
Nếu
x
1
= 1, khi đó từ
x
1
+ x
2
= ab
x
1
.x
2
= a + b
ta suy ra
1 + x
2
= ab
x
2
= a + b
, từ đó ta được
ab a b
= 1,
suy ra
(a 1) (b 1)
= 2. Chú ý
a > b
nên từ phương trình trên ta được
a
= 3;
b
= 2, khi đó ta
tìm được x
2
= 3.
2.
Nếu
x
1
= 2, tương tự như trường hợp trên ta tìm được các cặp số nguyên dương
(a; b)
thỏa mãn
(a; b) = (5; 1) , (2; 2).
Bài toán được giải quyết hoàn toàn.
Bài toán 6. Tìm tất cả các số nguyên tố p, q sao cho tồn tại số tự nhiên m thỏa mãn
pq
p + q
=
m
2
+ 1
m + 1
Lời giải
Quan sát hệ thức
pq
p + q
=
m
2
+ 1
m + 1
thì hai đại lượng
pq
và
p
+
q
làm ta liên tưởng đến hệ thức Viète.
Nếu
m
2
+ 1
m + 1
phân số tối giản thì từ hệ thức của bài toán ta được
p + q = m + 1
pq = m
2
+ 1
Còn nếu
m
2
+ 1
m + 1
chưa tối giản thì chỉ cần rút gọn ta cũng được một hệ điều kiện tương tự. Vấn đề
ta cần kiểm tra xem phân số
m
2
+ 1
m + 1
rút gọn được hay không.
Nếu p = q thì từ
pq
p + q
=
m
2
+ 1
m + 1
ta được
p =
2
m
2
+ 1
m + 1
= 2m 2 +
4
m + 1
Do
m N
và
p
số nguyên tố nên
(m + 1) |
4
m
= 0;
m
= 1;
m
= 3. Từ đó ta tìm được
p = 2; p = 5 thỏa mãn yêu cầu bài toán.
Nếu
p 6
=
q
thì
pq
và
p
+
q
nguyên tố cùng nhau
pq
chỉ chia hết cho các ước nguyên tố
p
và
q
còn
p
+
q
thì không chia hết cho
p
và không chia hết cho
q.
Gọi
r
một ước chung của
m
2
+ 1 và m + 1. Khi đó ta
r | [(m + 1) (m 1)] r |
m
2
1
Do đó r |

m
2
+ 1
m
2
1

r | 2 suy ra r = 1 hoặc r = 2.
1. Với r = 1 suy ra
p + q = m + 1
pq = m
2
+ 1
, khi đó p và q hai nghiệm của phương trình
x
2
(m + 1) x + m
2
+ 1 = 0
Ta =
3
m
2
+ 2
m
3 =
(m 1)
2
2m
2
+ 2
<
0 nên phương trình trên nghiệm.
2. Với r = 2 suy ra
2pq = m
2
+ 1
2 (p + q) = m + 1
, khi đó p và q hai nghiệm của phương trình
2x
2
(m + 1) x + m
2
+ 1 = 0
Ta = 7m
2
+ 2m 7 = (m 1)
2
6m
2
+ 6
< 0 nên phương trình vô nghiệm.
Vậy bộ các số nguyên tố cần tìm (p; q) = (2; 2) , (5; 5).
5
Hướng tới VMO 2020 Tạp chí và liệu toán học
Bài toán 7. Tìm số nguyên tố p để p
2
p + 1 lập phương của một số nguyên tố khác.
Lời giải
Do
p
số nguyên tố nên ta được
p | q
1 hoặc
p | q
2
+
q
+ 1. Đến đây ta sẽ chứng minh
p | q
1
không xẩy ra. Thật vậy. nếu
p | q
1 thì ta được
q
1 =
kp, k N
, do đó
q
=
kp
+ 1. Khi đó từ
p
2
p + 1 = q
3
ta được
p
2
p + 1 = (kp + 1)
3
p
2
p + 1 = k
3
p
3
+ 3k
2
p
2
+ 3kp + 1
Nhận thấy với k > 2 thì hiển nhiên
p
2
p + 1 < k
3
p
3
+ 3k
2
p
2
+ 3kp + 1
Từ đó suy ra được k 6 1.
1. Với k = 0, khi đó ta được p
2
p + 1 = 1 p (p 1) = 0, điều y do p số nguyên tố.
2. Với k = 2, khi đó ta được
p
3
2p
2
+ 4p = 0 p
2
2p + 4 = 0
không tồn tại p thỏa mãn.
Vậy với p | q 1 thì không tồn tai số nguyên tố p thỏa mãn yêu cầu bài toán.
Như vậy ta phải
p | q
2
+
q
+ 1, khi đó
(p, q 1)
= 1 và
(q 1) | p (p 1)
nên
q
1
| p
1. Đặt
p
=
(q 1) k
+ 1 với
k N
, khi đó từ
p | q
2
+
q
+ 1 ta suy ra được
q
2
+ q + 1
(q 1) k + 1
số nguyên dương
hay ta được
q
2
+ qk + k
qk k + 1
= q + 2 +
3k q 2
qk k + 1
số nguyên dương. Từ đó ta phải |3k q 2| > qk k + 1. Ta xét các trường hợp sau
Nếu 3
k q
2
> qk k
+ 1, khi đó ta được
k (4 q) > q
+ 3. Từ đó nếu
q >
4 thì ta được
k (4 q) < q
+ 3, điều y mâu thuẫn. Do đó ta suy ra được
q <
4, ta lại
q >
1 nên
q
= 2
hoặc q = 3.
1. Khi q = 2 thì từ p
2
p + 1 = q
3
ta được p (p 1) = 7, phương trình vô nghiệm.
2. Khi q = 3 thì từ p
2
p + 1 = q
3
ta được p (p 1) = 26, phương trình vô nghiệm.
Nếu 3k k 2 6 (qk k + 1) thì ta suy ra được
2 + q 3k > qk k + 1 k (q + 2) 6 q + 1
Điều y lí.
Nếu 3
kq
2 = 0
q
= 3
k
2, khi đó từ
p
=
(q 1) k
+1 ta được
p
= 3
k (k 1)
+1 = 3
k
2
3
k
+1
và đồng thời
q
2
+ q + 1 = (3k 2)
2
+ (3k 2) + 1 = 9k
2
9k + 3
Từ đó suy ra
q
2
+ q + 1
p
=
9k
2
9k + 3
3k
2
3k + 1
= 3
Do đó từ
p (p 1)
=
(q 1)
q
2
+ q + 1
ta được
p
1 = 3
(q 1)
nên suy ra ra được
(k 1) (k 3) = 0. Từ đó ta được k = 1 hoặc k = 3.
1. Với k = 1 thì p = q, khi đó ta được p
2
p + 1 = p
3
p
p
2
p + 1
= 1, điều y vô lí.
2.
Với
k
= 3 thì
p
= 3
q
2, khi đó ta được 9
q
2
15
q
+ 7 =
q
3
(q 1) (q 7)
= 0 nên
q
= 1
hoặc
q
= 7. Thử trực tiếp ta được
q
= 7 thỏa mãn yêu cầu bài toán. Khi đó ta được
p
= 19.
Vậy p = 19 số nguyên tố duy nhất thỏa mãn yêu cầu bài toán.
6
Chinh phục olympic toán Ứng dụng định Viète trong các bài toán số học
Bài toán 8.
Cho
x
=
a
+
b c
;
y
=
c
+
a b
;
z
=
b
+
c a
với
a, b, c
các số nguyên tố. Giả sử
rằng x
2
= y và
z
y bình phương của một số nguyên tố. Tìm giá trị của biểu thức
T = (a + 2) (b 10) (c + 2)
Lời giải
Biến đổi giả thiết tương đương
2a = x + y
2b = x + z
2c = y + z
2a = x + x
2
2b = x + z
2c = x
2
+ z
Xét phương trình bậc 2
x
2
+
x
= 2
a x
2
+
x
2
a
= 0. Dễ thấy = 8
a
+ 1
>
0 nên nên phương
trình hai nghiệm phân biệt. Gọi hai nghiệm của phương trình
x
1
và
x
2
. Như vậy nếu một trong
hai nghiệm số nguyên thì nghiệm còn lại cũng nguyên. Chú ý rằng
a
số nguyên tố nên ta nghĩ
đến sử dụng định Viète để xác định các nghiệm. Theo định Viète ta
x
1
+ x
2
= 1
x
1
.x
2
= 2a
Do 2 và
a
số nguyên tố nên từ
x
1
.x
2
=
2
a
ta được
x
1
{−2; a; 2; a}
. Ta xét các trường hợp sau.
Nếu x
1
= 2, khi đó ta tìm được a = 1 không phải số nguyên tố.
Nếu
x
1
=
a
, khi đó ta được
a
2
3
a
= 0, do
a
số nguyên tố nên
a
= 3. Từ đó ta tìm được hai
nghiệm của phương trình x
1
= 3 và x
2
= 2.
Nếu x
1
= 2, khi đó ta tìm được a = 3, từ đó ta tìm được hai nghiệm x
1
= 2 và x
2
= 3.
Nếu x = a, khi đó ta được a
2
a = 0 nên a = 0 và a = 1, loại không phải số nguyên tố.
Vậy phương trình trên hai nghiệm nguyên
x
= 2 và
x
=
3, đồng thời ta
a
= 3. y giờ ta sẽ
xác định các số nguyên tố b, c ứng với mỗi trường hợp.
1.
Với
x
= 2 khi đó ta được
y
= 4. Do đó
z
2 =
p
2
với
p
số nguyên tố. Do
x
số chẵn và
2
b
=
x
+
z
nên
z
số chẵn. Khi đó
p
số chẵn, dẫn đến
p
= 2. Khi đó ta được
z
= 36, suy ra
c = 20, loại do c không phải số nguyên tố.
2.
Với
x
=
3, khi đó ta được
y
= 9. Do đó
z
3 =
p
2
với
p
số nguyên tố. Do
x
số lẻ và
2
b
=
x
+
z
nên
z
số chẵn. Khi đó
p
số chẵn, dẫn đến
p
= 2. Khi đó ta được
z
= 49, suy ra
c = 29 và b = 23 các số nguyên tố.
Như vậy ta tính được T = (a + 2) (b 10) (c + 2) = (3 + 2) (23 10) (29 + 2) = 2015.
Bài toán 9.
Tìm các cặp số nguyên
(a; b)
sao cho hai số
a
2
+ 4
b
và
b
2
+ 4
a
đều số chính
phương.
Lời giải
Ta sẽ chứng minh các cặp số sau thỏa mãn yêu cầu bài toán
(a; b) =
0; k
2
,
k
2
; 0
, (4; 4) , (5; 6) , (6 5) , (k; 1 k) , (1 k, k) , k Z
Thật vậy, do vai trò của a và b như nhau nên không mất tính tổng quát ta thể giả sử |a| > |b|.
Nếu b = 0, khi đó để a
2
+ 4b và b
2
+ 4a đều số chính phương thì a = k
2
với k số nguyên.
Nếu
b 6
= 0, khi đó biểu thức
a
2
+ 4
b
ta liên tưởng đến biệt thức của phương trình
x
2
+ ax b = 0.
7
Hướng tới VMO 2020 Tạp chí và liệu toán học
Do =
a
2
+ 4
b
số chính phương nên phương trình trên sẽ hai nghiệm nguyên
x
1
và
x
2
. Theo
định Viète ta được
1
|x
1
|
+
1
|x
2
|
>
1
x
1
+
1
x
2
=
x
1
+ x
2
x
1
x
2
=
a
b
> 1
Từ đó suy ra một trong hai nghiệm nguyên của phương trình trên, chẳng hạn
x
1
thỏa mãn
|x
1
| 6
2.
Từ đó ta được x
1
{−2; 1; 1; 2}. Đến đây ta xét các trường hợp sau.
1. Nếu x
1
= 2, khi đó từ phương trình x
2
+ ax b = 0 ta được b = 2a + 4. Suy ra
b
2
+ 4a = (2a + 4)
2
+ 4a = 4a
2
+ 20a + 16 = (2a + 5)
2
9
số chính phương. Đặt
(2a + 5)
2
9 =
y
2
với
y N
, từ đó ta được
(2a + 5 y) (2a + 5 + y)
= 9.
Lúc y tìm được a = 4 và a = 1.
Với a = 4, khi đó a = 4. Từ đó ta được (a; b) = (4; 4) thỏa mãn yêu cầu bài toán.
Với a = 1, khi đó b = 2. Trường hợp y loại do không thỏa mãn |a| > |b|.
2. Nếu x
1
= 2, khi đó từ phương trình x
2
+ ax b = 0 ta được b = 4 2a. Suy ra
b
2
+ 4a = (4 2a)
2
+ 4a = 4a
2
12a + 16 = (2a 3)
2
+ 7
số chính phương. Đặt
(2a 3)
2
+ 7 =
y
2
với
y N
, từ đó ta được
(y 2a 3) (y 2a + 3)
= 7.
Giải phương trình trên ta được
(a; b)
=
(3; 2) , (0; 4)
, trong đó nghiệm
(0; 4)
bị loại do không
thỏa mãn |a| > |b|. Chú ý (3; 2) dạng (k; 1 k).
3. Nếu x
1
= 1, khi đó từ phương trình x
2
+ ax b = 0 ta được b = a + 1. Suy ra
b
2
+ 4a = (a + 1)
2
+ 4a = a
2
+ 6a + 1 = (a + 3)
2
8
số chính phương. Đặt
(a + 3)
2
8 =
y
2
với
y N
, từ đó ta được
(a + 3 y) (a + 3 + y)
= 8.
Giải phương trình trên ta được
(a; b)
=
(6; 5) , (0; 1)
, trong đó nghiệm
(0; 1)
bị loại do không
thỏa mãn |a| > |b|.
4. Nếu x
1
= 1, khi đó từ phương trình x
2
+ ax b = 0 ta được b = 1 a. Suy ra
b
2
+ 4a = (1 a)
2
+ 4a = a
2
+ 2a + 1 = (a + 1)
2
số chính phương và
a
2
+ 4b = a
2
+ 4 (1 a) = a
2
4a + 4 = (a 2)
2
cũng số chính phương. Do đó
(a; b)
=
(k; 1 k)
với
k
số nguyên thỏa mãn yêu cầu bài toán.
Chú ý với
(a; b)
thỏa mãn yêu cầu bài toán thì
(b; a)
cũng thỏa mãn yêu cầu bài toán. Do vy kết hợp
các trường hợp lại ta được các cặp số nguyên (a; b) như trên thỏa mãn yêu cầu bài toán.
Bài toán 10. Cho a, b, c, d các số thực thỏa mãn
a
2
1
5a
=
b
2
1
5b
=
c
2
1
4c
=
d
2
1
4d
= p
trong đó
p
số nguyên dương. Chứng minh rằng
(a c) (b c) (a + d) (b + d)
một số chính
phương.
Lời giải
8
Chinh phục olympic toán Ứng dụng định Viète trong các bài toán số học
Biến đổi giả thiết tương đương
a
2
+ 5pa 1 = b
2
+ 5pb 1 = 0; c
2
+ 4pc 1 = d
2
+ 4pd 1 = 0
Xét hai phương trình bậc hai ẩn
x
x
2
+ 5
px
1 = 0 và
x
2
+ 4
px
1 = 0. Khi đó ta thấy
a, b
hai
nghiệm của phương trình
x
2
+ 5
px
1 = 0 và
c, d
hai nghiệm của phương trình
x
2
+ 4
px
1 = 0.
Theo định Viète ta được
a + b = 5p
ab = 1
và
c + d = 4p
cd = 1
Ta
(a c) (b c) (a + d) (b + d)
=
ab (a + b) c + c
2
ab + (a + b) d + d
2
. Áp dụng các hệ thức
Viète trên ta được
ab (a + b) c + c
2
ab + (a + b) d + d
2
=
c
2
+ 5pc 1
d
2
9pd 1
Chú ý rằng
c
2
+ 5pc 1
d
2
9pd 1
=
c
2
+ 4pc 1 + pc
d
2
+ 4pd 1 9pd
, và đồng thời kết
với c
2
+ 4pc 1 = d
2
+ 4pd 1 = 0 ta được
(a c) (b c) (a + d) (b + d) = 9p
2
cd = 9p
2
= (3p)
2
Vậy (a c) (b c) (a + d) (b + d) một số chính phương.
4 Phương pháp bước nhảy Viète - Vieta Jumping.
Đây một phương pháp mạnh để xử lớp phương trình Diophantine bậc hai trở lên. Sau đây ta sẽ
tìm hiểu v phương pháp giải của nó.
Phương pháp. Ta tiến hành qua 2 bước sau.
1. Bước 1.
Cố định một giá trị nguyên đề bài cho, rồi giả sử tồn tại một cặp nghiệm
thỏa mãn một vài điều kiện không làm mất tính tổng quát của bài toán.
2. Bước 2.
Dựa vào định Viète để tìm các mối quan hệ và sự mâu thuẫn, từ đó tìm được
kết luận của bài toán.
Một trong các bài toán nổi tiếng nhất để minh họa cho phương pháp y và luôn xuất hiện trong bất
các tài liệu nói v vấn đề y, mỗi khi nhắc tới học sinh chuyên toán không thể không biết đó
chính bài toán trong thi IMO 1988.
Bài toán 1 [IMO 1988].
Cho
a, b
các số nguyên dương thỏa mãn
ab
+ 1
| a
2
+
b
2
. Chứng
minh rằng
a
2
+ b
2
ab + 1
số chính phương.
Lời giải
Bàn luận.
Đây chính bài toán khó nhất thi năm đó, và chỉ mười một học sinh cho lời giải
hoàn chỉnh của bài toán. Trong số 11 học sinh giải được bài toán đó, Việt Nam chúng ta một đại
diện chính Giáo Ngô Bảo Châu. Sau đây lời giải cho bài toán y.
Lời giải.
Đặt
k
=
a
2
+ b
2
ab + 1
, khi đó theo phương pháp đã đề cập tới trên, ta cố định
k
, sau đó xét tất
cả các cặp (a, b) nguyên dương thỏa mãn phương trình
k =
a
2
+ b
2
ab + 1
Hay nghĩa ta xét tập
S =
(a, b) N
× N
| k =
a
2
+ b
2
ab + 1
.
S
tập các cặp số nguyên dương
nên luôn tồn tại một cặp (a
0
, b
0
) trong S a
0
+ b
0
thỏa mãn a
0
> b
0
đạt giá trị nhỏ nhất.
9
Hướng tới VMO 2020 Tạp chí và liệu toán học
Xét phương trình
x
2
+ b
2
0
xb
0
+ 1
= k x
2
kx.b
0
+ b
2
0
k = 0
một phương trình bậc hai ẩn
x
. Ta đã biết rằng phương trình trên một nghiệm
a
0
. Như vy
theo định Viète thì tồn tại nghiệm a
1
thỏa mãn phương trình bậc hai với ẩn x trên và
a
1
= kb
0
a
0
=
b
2
0
k
a
0
Từ đây ta a
1
cũng số nguyên. Ta chứng minh a
1
không âm. Thật vậy, nếu a
1
< 0 thì
a
2
1
kb
0
a
1
+ b
2
0
k > a
2
1
+ k + b
2
0
k > 0
điều y mâu thuẫn. Do đó ta a
1
> 0. Đến đây ta xét a
1
> 0 thì (a
1
, b
0
) một cặp thuộc S.
Theo định nghĩa của (a
0
, b
0
) ta
a
0
+ b
0
6 a
1
+ b
0
a
0
6 a
1
Mặt khác cũng theo định Viète thì
a
2
0
6 a
0
a
1
= b
2
0
k < b
2
0
a
0
< b
0
điều y trái với giả thiết ban đầu. Do đó
a
1
= 0, vậy suy ra
k
=
b
2
0
một số chính phương, ta
điều cần chứng minh.
!
Nhận xét.
Trong bài toán y, ta đã sử dụng tới nguyên cực hạn: Trong tập hợp các số nguyên
dương thì luôn tồn tại số nguyên dương nhỏ nhất. Mệnh đề trên không những hữu dụng trong
các lớp bài toán y còn trong nhiều bài toán tổ hợp, tổ hợp số học và số học. Bài toán các
bạn sẽ tìm hiểu sau đây cũng một kết quả rất nối tiếng.
Bài toán 2 [Phương trình Markov]. Giải phương trình nghiệm nguyên
x
2
+ y
2
+ z
2
= 3xyz.
Lời giải
Bàn luận.
Đây một phương trình cực nổi tiếng, xuất hiện trong luận án tiến tại trường Đại học
Saint Petersburg với ch đề “Dạng toàn phương xác định dương” của nhà toán học Andrei Andreevich
Markov (1856 - 1922) - nhà toán học nổi tiếng người Nga. Luận án tiến của Markov đã giải quyết
được một số vấn đề khó trong “Lý thuyết số” và mở ra một hướng nghiên cứu trong toán học, đó “Lý
thuyết xấp xỉ Diophant”. Phương trình Markov - một phương trình Diophant bậc hai đặc biệt đóng vai
trò chủ đạo trong các nghiên cứu của Markov về các dạng toàn phương.
Lời giải. Ta thấy rằng phương trình Markov một nghiệm (1, 1, 1). Đặt
S = {(x, y, z); x, y, z Z
+
| x
2
+ y
2
+ z
2
= 3xyz}
tập hợp tất cả các nghiệm nguyên dương của phương trình Markov thì
S 6
=
.
Do vai trò của
x, y, z
trong phương trình như nhau, không mất tính tổng quát ta thể giả sử rằng
x 6 y 6 z.
Với mỗi
cặp (
x, y, z
)
S
; (
x
0
, y
0
, z
0
)
S
ta định nghĩa (
x, y, z
)
>
(
x
0
, y
0
, z
0
) nếu
x
+
y
+
z > x
0
+
y
0
+
z
0
.
Markov
đã dùng ý tưởng “thông minh”sau để chứng minh vô hạn b ba số nguyên dương (
x, y, z
) thỏa mãn
phương trình trên. Với mỗi nghiệm (
x
n
, y
n
, z
n
)
S
ta y dựng b nghiệm mới như sau: Ta coi
x
n
ẩn và các biến còn lại các tham số thì ràng phương trình bậc hai
x
2
3y
n
z
n
x + y
2
n
+ z
2
n
= 0
một nghiệm x
n
, nên nghiệm thứ hai x
0
. Theo định Viète, ta
x
n
+ x
0
= 3y
n
z
n
và x
n
x
0
= y
2
n
+ z
2
n
(1)
10
Chinh phục olympic toán Ứng dụng định Viète trong các bài toán số học
Từ đây ta được x
0
một số nguyên dương, kết hợp với giả thiết x
n
6 y
n
6 z
n
và (1) ta được
x
0
=
y
2
n
+ z
2
n
x
n
>
2x
2
n
x
n
= 2x
n
> x
n
.
Đặt (
x
n+1
, y
n+1
, z
n+1
) = (
x
0
, y
n
, z
n
) thì (
x
n+1
, y
n+1
, z
n+1
) một nghiệm của phương trình Markov.
Cách xây dựng này cho ta một y hạn các nghiệm của phương trình Markov các nghiệm tiếp
theo lớn hơn các nghiệm trước theo định nghĩa thứ tự trên. Do đó phương trình Markov vô số
nghiệm. Ta thấy ý tưởng của Markov trong chứng minh trên coi một biến nghiệm của tam thức
bậc hai khi cố định các nghiệm còn lại để từ đó y dựng nghiệm mới từ một nghiệm đã biết bằng các
định Viète. Cụ thể ta xét phương trình Diophant phương trình bậc hai đối với một biến nào đó,
chẳng hạn
x
2
1
+
G
(
x
2
, x
2
, . . . , x
n
)
6
= 0 phương trình bậc 2 ẩn
x
1
.
Nếu phương trình này nghiệm
(
x
1
, x
2
, . . . , x
n
) = (
a
1
, a
2
, . . . , a
n
) thì ràng
a
1
nghiệm phương trình
X
2
+
G
(
a
2
, a
3
, . . . , a
n
) = 0.
Phương trình trên phải còn một nghiệm nữa
a
0
1
.
Kết hợp với định Viète và dữ kiện của đầu bài ta
sẽ y dựng b (a
0
1
, a
2
, . . . , a
n
) nghiệm của phương trình trên.
!
Nhận xét.
Thông qua 2 bài toán đầu tiên, ta đã phần nào hiểu được ý tưởng của phương pháp
y, bài toán thứ 3 sau đây bài toán tổng quát của bài toán y, sẽ trả lời cho ta câu
hỏi “Nếu tổng các bình phương
S
ba số nguyên dương chia hết cho tích
P
của chúng thì khi đó
thương số
S
P
bằng bao nhiêu?”.
Bài toán 3.
Tìm tất cả các số nguyên dương
k
để phương trình
x
2
+
y
2
+
z
2
=
kxyz
nghiệm
nguyên dương.
Lời giải
Lời giải 1.
Trước tiên ta thấy rằng với
k
= 1 phương trình đã cho nghiệm
(3; 3; 3)
và với
k
= 3
thì phương trình nghiệm
(1; 1; 1)
. Như vậy với
k
= 1 hoặc
k
= 3 thì phương trình luôn nghiệm
nguyên dương. y giờ ta cần kiểm tra xem với
k 6
= 1 và
k 6
= 3 thì phương trình nghiệm nguyên
dương không. Giả sử với
k 6
= 1 và
k 6
= 3 phương trình đã cho nghiệm nguyên dương
(x
0
; y
0
; z
0
)
.
Không mất tính tổng quát ta giả sử
x
0
6 y
0
6 z
0
và
x
0
+
y
0
+
z
0
giá trị nhất. Ta xét các trường
hợp.
1.
Nếu
y
0
< z
0
, ta xét phương trình bậc hai
z
2
kx
0
y
0
z
+
x
2
0
+
y
2
0
= 0. Khi đó phương trình một
nghiệm z
0
. Theo định Viète thì phương trình một nghiệm nữa z
1
. Như vậy thì
z
0
+ z
1
= kx
0
y
0
z
0
z
1
= x
2
0
+ y
2
0
,
từ đó suy ra
z
1
= kx
0
y
0
z
0
=
x
2
0
+ y
2
0
z
0
Ta thấy
z
1
nhận giá trị nguyên dương nên
(x
0
; y
0
; z
1
)
một nghiệm nguyên dương của phương
trình ban đầu. Từ điều giả sử ta
x
0
+ y
0
+ z
0
6 x
0
+ y
0
+ z
1
nên z
0
6 z
1
. Do đó ta được
x
2
0
+ y
2
0
kx
0
y
0
= z
0
z
1
z
1
z
0
= (z
1
1) (z
0
1) 1 > y
2
0
1
Suy ra 1
> x
0
(ky
0
x
0
) > x
0
(kx
0
x
0
) > x
0
. Do
x
0
số nguyên dương nên ta được
x
0
= 1.
Từ đây ta đưa phương trình ban đầu về thành
y
2
+
z
2
+ 1 =
kyz
. Đến đây ta cần chỉ ra rằng
phương trình
y
2
+
z
2
+ 1 =
kyz
nghiệm nguyên dương khi và chỉ khi
k
= 3, tuy nhiên điều y
đơn giản nếu ta sử dụng phương pháp bước nhảy Viète. Do đó điều y mâu thuẫn với k 6= 3.
11
Hướng tới VMO 2020 Tạp chí và liệu toán học
2. Nếu y
0
= z
0
thì ta
2y
2
0
kx
2
0
y
2
0
+ x
2
0
= 0 x
2
0
= y
2
0
(kx
0
2) > x
2
0
(kx
0
2)
Từ đó dẫn đến 3
> kx
0
, ta lai
kx
0
>
2 nên
kx
0
= 3, suy ra
k
= 1 hoặc
k
= 3, điều này
trái với k 6= 1 và k 6= 3.
Vậy với
k 6
= 1 và
k 6
= 3 thì phương trình đã cho không nghiệm nguyên dương. Như vy với
k
= 1
hoặc k = 3 thì phương trình đã cho nghiệm nguyên dương.
Lời giải 2. Với x, y, z Z
+
, ta viết phương trình đã cho dưới dạng
x
2
kxyz + y
2
+ z
2
= 0. (1)
Giả sử
k
số nguyên dương sao cho phương trình (1) nghiệm nguyên dương. Cố định
k
và xét tập
hợp
S = {(x, y, z); x, y, z Z
+
| x
2
kxyz + y
2
+ z
2
= 0}.
Theo điều giả sử trên thì
S 6
=
, khi đó theo nguyên sắp thự tự tốt tồn tại (
x
0
, y
0
, z
0
)
S
sao
cho
x
0
+
y
0
+
z
0
nhỏ nhất. Ta thấy rằng, nếu (
x
0
, y
0
, z
0
)
S
thì các hoán vị của cũng thuộc
S
,
không mất tính tổng quát ta thể giả sử x
0
> y
0
> z
0
. Phương trình
f(x) = x
2
xky
0
z
0
+ y
2
0
+ z
2
0
= 0
hiển nhiên một nghiệm x
0
. Gọi nghiệm còn lại x
1
, theo định Viète, ta
x
0
+ x
1
= ky
0
z
0
; x
0
x
1
= y
2
0
+ z
2
0
.
Từ đây, ta được
x
1
không âm, do đó (
x
1
, y
0
, z
0
)
S,
theo cách xác định của bộ (
x
0
, y
0
, z
0
) thì ta thu
được x
1
+ y
0
+ z
0
> x
0
+ y
0
+ z
0
hay x
1
> x
0
. Do đó ta
x
1
> x
0
> y
0
> z
0
. (2)
Theo định v dấu của tam thức bậc hai và từ (2) ta được
0 6 f(y
0
) 6 y
2
0
ky
2
0
z
0
+ 2y
2
0
= y
2
0
(3 kz
0
).
Suy ra kz
0
6 3 k 6 kz
0
6 3 k Z
+
nên k {1, 2, 3}.
1. Nếu k = 1, phương trình (1) nghiệm nguyên dương x = y = z = 3.
2. Nếu k = 2, thì từ kz
0
6 3 ta được z
0
= 1 khi đó ta (x
0
y
0
)
2
+ 1 = 0, mâu thuẫn.
3. Nếu k = 3, phương trình (1) nghiệm nguyên dương x = y = z = 1.
Vậy với k {1, 3} thì phương trình nghiệm nguyên dương.
!
Nhận xét.
Trong lời giải 1 ta đề cập tới một kết quả đó Phương trình
y
2
+
z
2
+ 1 =
kyz
có
nghiệm nguyên dương khi chỉ khi
k
= 3, đây bài toán trong đề thi HSG toán 9 Tỉnh Thanh
Hóa 2015 2016. Sau đây ta sẽ cùng xét tới bài toán y.
Bài toán 4.
Tìm các số nguyên dương
m
để phương trình
x
2
mxy
+
y
2
+ 1 = 0 nghiệm
nguyên dương.
Lời giải
Với các nghiệm nguyên dương
(x; y)
thỏa mãn phương trình, giả sử
(x
0
; y
0
)
một nghiệm thỏa mãn
x
0
+
y
0
nhỏ nhất. Do vai trò của
x
và
y
trong phương trình như nhau nên không mất tính tổng quát
12
Chinh phục olympic toán Ứng dụng định Viète trong các bài toán số học
ta thể giả sử x
0
6 y
0
.
Xét phương trình bậc hai ẩn y
y
2
mx
0
y + x
2
0
+ 1 = 0
La
y
0
một nghiệm của phương trình trên. Ta gọi nghiệm còn lại
y
1
. Khi đó theo định Viète
ta
y
0
+ y
1
= mx
0
y
0
.y
1
= x
2
0
+ 1
Dễ dàng nhận thấy
y
1
giá trị nguyên và từ cách chọn
(x
0
; y
0
)
ta suy ra được
y
0
6 y
1
. Đến đây ta
xét các trường hợp sau.
1.
Nếu
x
0
=
y
0
thì từ phương trình ban đầu ta được
m
= 2 +
1
x
2
0
. Nên để
m
và
x
0
giá trị nguyên
thì x
0
= 1 và m = 3.
Với m = 3 ta thấy (x; y) = (1; 1) một nghiệm nguyên dương của phương trình đã cho.
2. Nếu y
0
= y
1
thì từ y
0
.y
1
= x
2
0
+ 1 hay
(y
0
x
0
) (y
0
+ x
0
) = 1
Từ đó ta suy ra được
y
0
x
0
= 1
y
0
+ x
0
= 1
y
0
= 1
x
0
= 0
Trường hợp này loại (x
0
; y
0
) nguyên dương.
3. Nếu x
0
< y
0
< y
1
khi đó ta được
y
0
> x
0
+ 1; y
1
> x
0
+ 2
Kết hợp với
y
0
.y
1
=
x
2
0
+ 1 ta được
x
2
0
+ 1
> x
2
0
+ 3
x
0
+ 2
3
x
0
+ 1
6
0, điều y vô
x
0
>
0.
Như vy để phương trình đã cho nghiệm nguyên dương thì
m
= 3 và khi đó phương trình nghiệm
nguyên dương (x; y) = (1; 1).
Bài toán 5.
Tìm tất cả các số nguyên dương
k
sao cho phương trình
x
2
+
y
2
+
x
+
y
=
kxy
nghiệm nguyên dương.
Lời giải
Gọi
(x
0
, y
0
)
b nghiệm nguyên dương của phương trình thỏa mãn
x
0
+
y
0
nhỏ nhất. Không mất
tính tổng quát, ta giả sử x
0
> y
0
> 1. Xét phương trình bậc hai ẩn x
x
2
+ y
2
0
+ x + y
0
= kxy
0
x
2
+ x(1 ky
0
) + y
2
0
+ y
0
= 0
Phương trình bậc hai y hiển nhiên một nghiệm
x
0
, gọi nghiệm còn lại
x
1
. Theo định Viète ta
x
0
+ x
1
= ky
0
1 (1)
x
0
x
1
= y
2
0
+ y
0
(2)
Từ (1) ta
x
1
nguyên, từ (2) ta
x
1
dương. Như vậy (
x
1
, y
0
) cũng một nghiệm thỏa mãn phương
trình, mặt khác, do tính nhỏ nhất của tổng x
0
+ y
0
ta x
1
> x
0
. Do đó từ (1), ta
ky
0
1 > 2x
0
2x
0
y
0
+
1
y
0
6 k
Ta
k =
x
0
y
0
+
y
0
x
0
+
1
x
0
+
1
y
0
=
x
0
y
0
+
1
2y
0
+
y
0
x
0
+
1
x
0
+
1
2y
0
6
k
2
+ 1 + 1 +
1
2
k 6 5 k {1; 2; 3; 4; 5}
13
Hướng tới VMO 2020 Tạp chí và liệu toán học
1. Với k = 1, ta x
2
+ y
2
+ x + y = xy, phương trình này nghiệm nguyên dương
x
2
+ y
2
+ x + y > 2xy + x + y > xy
2.
Với
k
= 2, tương tự như trên, ta cũng lập luận được phương trình này vô nghiệm nguyên dương.
3. Với k = 3, phương trình nghiệm nguyên dương (2; 2).
4. Với k = 4 thì phương trình nghiệm (1; 1).
5. Với k = 5, dấu bằng phải đồng thời xảy ra các điểm
x
y
+
1
2y
=
k
2
=
5
2
,
1
x
= 1,
1
2y
= 1,
y
x
= 1
Dễ thấy không tồn tại các số nguyên dương
x, y
thỏa mãn tất cả các điều trên. Trường hợp y
bị loại.
Vậy các giá trị của k thỏa mãn k {3; 4}.
Bài toán 6 [IMO 2007].
Cho trước
a, b
hai số nguyên dương. Chứng minh rằng nếu 4
ab
1
ước số của (4a
2
1)
2
thì a = b.
Lời giải
Theo giả thiết thì 4ab 1 | (4a
2
1)
2
nên ta 4ab 1 ước của
b
2
(4a
2
1)
2
(4ab 1)(4a
3
b 2ab + a
2
) = (a b)
2
.
Đến đây ta đặt k =
(a b)
2
4ab 1
thì k Z
+
. Cố định k và xét tập hợp
S =
(a, b); a, b Z
+
; a 6= b | a
2
2ab(2k + 1) + b
2
+ k = 0
Giả sử
S 6
=
, khi đó theo nguyên sắp thứ tự tốt tồn tại cặp số (
a
0
, b
0
)
S
sao cho
a
0
6
=
b
0
và
a
0
+
b
0
nhỏ nhất. Chú ý rằng, nếu (
a
0
, b
0
)
S
thì (
b
0
, a
0
)
S
, do vậy không mất tính tổng quát ta
thể giả sử
a
0
> b
0
. Phương trình
T
2
2
T b
0
(2
k
+ 1) +
b
2
0
+
k
= 0 một nghiệm hiển nhiên
a
0
. Gọi
nghiệm còn lại a
1
, theo định Viète ta
a
0
+ a
1
= 2b
0
(2k + 1)
a
0
a
1
= b
2
0
+ k.
(1)
Suy ra a
1
số nguyên không âm, do đó (a
1
, b
0
) S, theo cách xác định (a
0
, b
0
) thì
a
1
+ b
0
> a
0
+ b
0
a
1
> a
0
.
Kết hợp với (1) ta được
a
0
=
b
2
0
+ k
a
1
6
b
2
0
+ k
a
0
k > a
2
0
b
2
0
.
Như vậy ta suy ra
(a
0
b
0
)
2
4a
0
b
0
1
= k > a
2
0
b
2
0
= (a
0
b
0
)(a
0
+ b
0
).
Mặt khác lại do a
0
> b
0
nên a
0
b
0
> 1, vy
a
0
b
0
> (a
0
+ b
0
)(4a
0
b
0
1) > a
0
+ b
0
,
điều y mâu thuẫn, do đó điều giả sử sai hay S 6= .
14
Chinh phục olympic toán Ứng dụng định Viète trong các bài toán số học
Bài toán 7.
Chứng minh rằng nếu
a, b
các số nguyên dương sao cho
k
=
a
2
+ b
2
ab 1
số nguyên
thì k = 5.
Lời giải
Lời giải 1.
Đẳng thức đề bài tương đương với
a
2
kab
+
b
2
+
k
= 0. Không mất tính tổng quát giả sử
a > b. Do a, b các số nguyên dương và ab 6= 1 nên ta xét a = 2; b = 1 thì được k = 5.
Như vậy ta cần chứng minh với
a
+
b >
3 thì
k
= 5, giả sử cặp số dương
(a
0
; b
0
)
tổng nhỏ nhất
thỏa mãn bài toán. Khi đó ta được
a
2
0
ka
0
b
0
+ b
2
0
+ k = 0
Xét phương trình bậc 2 ẩn x
a
2
kab + b
2
+ k = 0
Ta thấy rằng
a
0
một nghiệm của phương trình. Như vậy theo định Viète thì phương trình trên còn
nghiệm
a
1
, khi đó ta
a
=
kb
0
a
0
hay ta cặp số
(kb
0
a
0
; b
0
)
thỏa mãn yêu cầu bài toán.
Theo điều giả sử ta a
0
6 kb
0
a
0
hay
a
0
b
0
6
k
2
. Từ k =
a
2
+ b
2
ab 1
ta suy ra
a
0
b
0
+
b
0
a
0
+
k
a
0
b
0
= k.
Do
a
0
b
0
6
k
2
và a
0
b
0
> 3 nên k 6
k
2
+ 1 +
k
3
hay k 6 6. Mặt khác ta lại
a
0
b
0
+
b
0
a
0
> 2
nên k > 3. Như vậy ta được 3 6 k 6 6.
1. Với a = 3; b = 1 ta tìm được k = 5.
2. Với a = b = 2 hay a = 4; b = 1 thì không tìm được giá trị của k.
3.
Với
ab >
5, lại dùng đánh giá tương tự như trên ta
k 6
3. Xét
k
= 3 thì
a
2
+ 3
ab
+
b
2
= 3, ta
thấy không cặp số dương (a; b) thỏa mãn.
Do đó suy ra
ab >
6. Thử với
a
= 6;
b
= 1 hoặc
a
= 3;
b
= 2 đều không thỏa nên ta lại được
ab >
7.
Lại dùng đánh giá như trên ta được suy ra
k 6
14
5
, điều y mâu thuẫn với
k
nguyên và lớn hơn 2.
Vậy chỉ k = 5 thỏa mãn bài toán.
Lời giải 2. Đặt k =
a
2
+ b
2
ab 1
thì k số nguyên dương. Theo bất đẳng thức AM GM thì
k =
a
2
+ b
2
ab 1
>
2ab
ab 1
= 2 +
2
ab 1
> 2
hay k > 3. Nếu a = b thì ta được k = 2 +
2
a
2
1
< 3, mâu thuẫn.
Ta sẽ chứng minh k = 5. Thật vậy, cố định k và xét tập hợp
S =
(a, b); a, b Z
+
| k =
a
2
+ b
2
ab 1
Theo giả thiết
S 6
=
, khi đó theo nguyên sắp thứ tự tốt tồn tại cặp số (
a
0
, b
0
)
S
sao cho
a
0
6
=
b
0
và
a
0
+
b
0
nhỏ nhất. Ta thấy rằng nếu (
a
0
, b
0
)
S
thì (
b
0
, a
0
)
S
, không mất tính tổng quát ta thể
giả sử a
0
> b
0
. Nhận thấy rằng phương trình
T
2
+ b
2
0
T b
0
1
= k T
2
kT b
0
+ b
2
0
+ k = 0
15
Hướng tới VMO 2020 Tạp chí và liệu toán học
một nghiệm hiển nhiên a
0
. Gọi nghiệm còn lại a
1
, theo định Viète ta
a
0
+ a
1
= kb
0
; a
0
a
1
= b
2
0
+ k.
Từ đây, ta được
a
1
Z
+
, do đó (
a
1
, b
0
)
S
, theo cách xác định (
a
0
, b
0
) thì
a
1
+
b
0
> a
0
+
b
0
hay
a
1
> a
0
. a
0
> b
0
nên a
0
> b
0
+ 1, từ đó ta thu được:
b
2
0
+ k kb
0
= a
0
a
1
a
0
a
1
= (a
0
1)(a
1
1) 1 > b
2
0
1.
Do đó
k
(
b
0
1)
6
1. Nếu
b
0
6
= 1 theo chứng minh trên thì
k
(
b
0
1)
>
3
>
1, vậy ta phải
b
0
= 1.
Khi đó a
0
+ a
1
= k và a
0
a
1
= k + 1, suy ra
a
0
a
1
a
0
a
1
1 = 0 (a
0
1)(a
1
1) = 2
a
1
> a
0
nên a
0
= 2, a
1
= 3, từ đây ta được
k = a
0
+ a
1
= 5.
Như vậy, ta điều phải chứng minh.
Bài toán 8.
Tìm tất cả các giá trị k sao cho phương trình
(x + y + z)
2
=
kxyz
nghiệm
nguyên dương.
Lời giải
Ta gọi
k
giá trị cần tìm và
(x
0
; y
0
; z
0
)
nghiện nguyên dương của phương trình
(x + y + z)
2
=
kxyz
x
0
+
y
0
+
z
0
nhỏ nhất. Khi đó không mất tính tổng quát, ta thể giả sử rằng
x
0
> y
0
> z
0
. Phương
trình đã cho được viết dưới dạng
x
2
(kyz 2y 2z) x + (y + z)
2
= 0
Theo định Viète ta
x
1
= ky
0
z
0
2y
0
2z
0
x
0
=
(y
0
+ z
0
)
2
x
0
cũng nghiệm của phương trình trên, suy ra
(x
1
; y
0
; z
0
)
nghiệm của phương trình đầu. Ngoài ra ta
cũng suy ra được
x
1
nguyên dương, hay nói cách khác
(x
1
; y
0
; z
0
)
nghiệm nguyên dương của phương
trình đầu. Từ tính nhỏ nhất của x
0
+ y
0
+ z
0
ta được x
1
> x
0
, suy ra
ky
0
z
0
2y
0
2z
0
x
0
> x
0
và
(y
0
+ z
0
)
2
x
0
> x
0
Từ bất đẳng thức thứ 2 ta
y
0
+
z
0
> x
0
, áp dụng bất đẳng thức thứ nhất ta được
ky
0
z
0
>
4
x
0
. Chia
2 vế của
x
2
0
+ y
2
0
+ z
2
0
+ 2x
0
y
0
+ 2y
0
z
0
+ 2z
0
x
0
= kx
0
y
0
z
0
cho x
0
y
0
z
0
ta thu được
x
0
y
0
z
0
+
y
0
x
0
z
0
+
z
0
x
0
y
0
+
2
z
0
+
2
x
0
+
2
y
0
= k
Như vy ta được
k
4
+ 1 + 1 + 2 + 2 + 2
> k
hay
k 6
32
3
, suy ra
k 6
10. Chú ý rằng nếu
x
0
= 1 thì
y
0
=
z
0
= 1 suy ra
k
= 9. Nếu
k 6
= 9 thì
x
0
>
2 và đánh giá trên trở thành
k
4
+ 1 +
1
2
+ 2 + 1 + 2
> k
,
như thế thì ta suy ra được k 6
26
3
nên k 6 8. Giá trị k = 10 bị loại. Ta xét các trường hợp sau.
1. Với k = 1 phương trình nghiệm, chẳng hạn (9; 9; 9).
2. Với k = 2 phương trình nghiệm, chẳng hạn (4; 4; 8).
16
Chinh phục olympic toán Ứng dụng định Viète trong các bài toán số học
3. Với k = 3 phương trình nghiệm, chẳng hạn (3; 3; 3).
4. Với k = 4 phương trình nghiệm, chẳng hạn (2; 2; 4).
5. Với k = 5 phương trình nghiệm, chẳng hạn (1; 4; 5).
6. Với k = 6 phương trình nghiệm, chẳng hạn (1; 2; 3).
7. Với k = 8 phương trình nghiệm, chẳng hạn (1; 1; 2).
8. Với k = 9 phương trình nghiệm, chẳng hạn (1; 1; 1).
y giờ ta cần chứng chứng minh được rằng trường hợp
k
= 7 phương trình không nghiệm nguyên
dương. Thật vậy, giả sử với
k
= 7 thì phương trình đã cho nghiệm nguyên dương
(x
0
; y
0
; z
0
)
các
tính chất như trên. Khi đó ta
7 =
(x
0
+ y
0
+ z
0
)
2
x
0
y
0
z
0
=
x
0
y
0
z
0
+
y
0
z
0
x
0
+
z
0
x
0
y
0
+
2
x
0
+
2
y
0
+
2
z
0
<
1
y
0
+
1
z
0
+
y
0
+ z
0
y
0
z
0
+
2
x
0
+
2
y
0
+
2
z
0
6
10
z
0
Do đó ta z
0
<
10
7
nên z
0
= 1, khi đó ta
7 <
3
x
0
+
3
y
0
+
4
1
1 <
1
x
0
+
1
y
0
6
2
y
0
y
0
< 2
Từ đó tại được y
0
= 1. Như vậy ta
7 =
(x
0
+ y
0
+ z
0
)
2
x
0
y
0
z
0
=
(x
0
+ 2)
x
0
=
4
x
0
+ x
0
+ 4 > 2.
r
4
x
0
.x
0
+ 4 = 8
Như vậy khi k = 7 thì phương trình không nghiệm nguyên dương.
Vậy các giá trị k cần tìm k {1; 2; 3; 4; 5; 6; 8; 9}.
!
Bài toán tương tự.
1.
Cho các số nguyên dương
x, y, z
thỏa mãn điều kiện
(x + y + z)
2
chia hết cho
xyz
. Tính
các giá trị của A =
(x + y + z)
2
xyz
.
2. Chứng minh rằng phương trình (x + y + z)
2
= 7xyz không nghiệm nguyên dương.
Bài toán 9.
Chứng minh rằng phương trình
(x + y + z)
2
= 7
xyz
không nghiệm nguyên
dương.
Lời giải
Gọi
(x
0
; y
0
; z
0
)
một nghiệm thỏa mãn phương trình với
z
0
số nhỏ nhất. Không mất tính tổng quát,
ta giả sử x
0
6 y
0
6 z
0
, khi đó ta
z
0
| (x
0
+ y
0
+ z
0
)
2
z | (x
0
+ y
0
)
2
+ 2z
0
(x
0
+ y
0
) + z
02
z
0
| (x
0
+ y
0
)
2
Ta xét phương trình bậc hai ẩn
z
z
2
(7
x
0
y
0
2
x
0
2
y
0
)
z
+ (
x
0
+
y
0
)
2
= 0, hiển nhiên phương trình
y một nghiệm z
0
, nên theo định Viète thì nghiệm còn lại của
(x
0
+ y
0
)
2
z
0
Z.
Như vậy
x
0
; y
0
;
(x
0
+ y
0
)
2
z
0
cũng một bộ số thỏa mãn phương trình. Nếu giả sử
x
0
+ y
0
< z
0
(x
0
+ y
0
)
2
z
0
< z
0
17
Hướng tới VMO 2020 Tạp chí và liệu toán học
thì vô (
x
0
;
y
0
;
z
0
) cũng một bộ số thỏa mãn phương trình và tính nhỏ nhất của
z
0
. Do đó phải
z
0
6 x
0
+ y
0
. Khai triển phương trình ban đầu và chia hai vế của cho x
0
y
0
z
0
ta được
7 6
x
0
y
0
z
0
+
y
0
z
0
x
0
+
z
0
x
0
y
0
+
2
x
0
+
2
y
0
+
2
z
0
6
1
z
0
+
1
x
0
+
x
0
+ y
0
x
0
y
0
+
2
x
0
+
2
y
0
+
2
z
0
=
4
x
0
+
3
y
0
+
3
z
0
6
10
x
0
x
0
6
10
7
x
0
= 1
Khi đó ta được y
0
6 z
0
6 y
0
+ 1 z
0
= y z
0
= y
0
+ 1. Xét các trường hợp
1. Nếu z
0
= y
0
thì ta phương trình
(1 + 2z
0
)
2
= 7z
02
3z
02
4z
0
1 = 0 z
0
=
2 ±
7
3
trường hợp y loại.
2. Nếu z
0
= y
0
+ 1 thì ta phương trình
(2 + 2z
0
)
2
= 7z
0
(z
0
+ 1) 3z
02
z
0
4 = 0 z
1;
4
3
Như vậy không tồn tại nghiệm nguyên dương của phương trình đã cho.
Bài toán 10 [VMO 2012].
Xét các số tự nhiên lẻ
a, b
thỏa mãn
a | b
2
+ 2 và
b | a
2
+ 2. Chứng
minh rằng a, b các số hạng của y số tự nhiên (v
n
) được xác định bởi công thức
v
1
= v
2
= 1
v
n
= 4v
n1
v
n2
, n > 2
Lời giải
Đầu tiên ta sẽ đi chứng minh rằng b | a
2
+ 2 a | b
2
+ 2 ab | a
2
+ b
2
+ 2. Thật vậy, ta
b | a
2
+ 2 a | b
2
+ 2 ab |
a
2
+ 2
b
2
+ 2
ab | 2
a
2
+ b
2
+ 2
Do
a, b
lẻ nên
ab | a
2
+
b
2
+ 2. Ngược lại nếu
ab | a
2
+
b
2
+ 2 thì dễ dàng suy ra ngay được
b | a
2
+ 2
và a | b
2
+ 2. Từ đó giả thiết đề bài tương đương với việc tồn tại số nguyên dương k sao cho
a
2
+ b
2
+ 2 = kab.
Sử dụng phương pháp bước nhảy Viète ta sẽ đi chứng minh k = 4. Cố định k và xét tập
S =
a, b
Z
+
2
| k =
a
2
+ b
2
+ 2
ab
N
Trong
S
ta chọn ra cặp (
A, B
) sao cho tổng
A
+
B
nhỏ nhất. Không mất tính tổng quát, ta giả
sử
A > B
. Xét phương trình bậc hai ẩn
t
,
t
2
ktB
+
B
2
+ 2 = 0. Dễ thấy phương trình này một
nghiệm A, gọi nghiệm còn lại t
0
. Theo định Viète ta
t
0
+ A = kB
t
0
A = B
2
+ 2
Từ đây suy ra được t
0
nguyên dương. Chú ý A + B nhỏ nhất nên ta được t
0
> A.
Suy ra t
0
+ A = kB > 2A hay
A
B
6
k
2
.
Nếu một trong hai số a, b bằng 1, giả sử b = 1 thì ka = a
2
+ 3, dễ suy ra k = 4.
Nếu cả hai số a, b > 2. Ta A > B > 2. Thì
k =
A
B
+
B
A
+
2
AB
6
k
2
+ 1 +
2
2.2
k 6 3
18
Chinh phục olympic toán Ứng dụng định Viète trong các bài toán số học
Theo bất đẳng thức AM GM ta
kab = a
2
+ b
2
+ 2 > 2(ab + 1) k > 3
lúc này ta được
k
= 3. Khi đó
A
2
+
B
2
+ 2 = 3
AB
. Từ đẳng thức này dễ dàng suy ra phải một
trong hai số chia hết cho 3, giả sử 3
| A
thì
A >
3. Nếu
B
= 1 ta gặp mâu thuẫn, do đó
B >
2. Tức
AB > 6. Tuy nhiên
3 =
A
B
+
B
A
+
2
AB
6
3
2
+ 1 +
2
6
Điều y vô lí. Vậy
k
= 4 giá trị duy nhất cần tìm. Ta chứng minh xong việc các số
a, b
thỏa giả
thiết thì cũng phải thỏa mãn phương trình
a
2
+ b
2
+ 2 = 4ab (1)
Bài toán sẽ hoàn tất nếu ta chỉ rằng nếu cặp (
a, b
) bất kỳ thỏa mãn (1) thì sẽ luôn tồn tại số tự nhiên
n
sao cho
a
=
x
n
, b
=
x
n+1
. Giả sử (
u
0
, u
1
) một cặp số nguyên dương bất kỳ thỏa (1). Ta hoàn
toàn quyền giả sử
u
0
> u
1
. Nếu
u
0
= 1 thì
u
1
= 1, tức tồn tại
n
= 1 để
u
0
=
v
1
, u
1
=
v
2
. Tương tự
khi xét
u
1
= 1. Do đó ta chỉ cần xét
u
0
, u
1
>
1. Khi đó ta chọn cặp (
u
1
, u
2
) = (
u
1
,
4
u
1
u
0
), dễ thấy
u
1
, u
2
nguyên dương và (u
1
, u
2
) cũng thỏa mãn (1). Lúc này ta chú ý 4u
1
u
0
< u
0
4u
1
u
0
=
u
2
1
+ 2
u
0
u
0
=
2 (u
0
u
1
)(u
0
+ u
1
)
u
0
6 0.
Suy ra
u
1
+
u
2
=
u
1
+ (4
u
1
u
0
)
< u
1
+
u
0
. Tương tự ta cũng chọn được cặp (
u
2
, u
3
) = (
u
2
,
4
u
2
u
1
)
cũng thỏa
u
2
, u
3
nguyên dương, cũng thỏa (1) và
u
2
+
u
3
< u
1
+
u
2
< u
1
+
u
0
. Cứ tiếp tục quá trình
y, ta được
... < u
i
+ u
i+1
< ... < u
1
+ u
2
< u
1
+ u
0
Thế nhưng
u
1
+
u
0
>
2 nên phải tồn tại
k
sao cho
u
k
+
u
k+1
= 2, suy ra
u
k
=
u
k+1
= 1. Tức ta
u
k
= v
2
, u
k+1
= v
1
. Ta thể thấy được cách xác định u
n
như sau
u
n+2
= 4u
n+1
u
n
hay u
n
= 4u
n+1
u
n+2
.
Từ đó ta
u
k1
= 4u
k
u
k+1
= 4v
2
v
1
= v
3
u
k2
= 4u
k1
u
k
= 4v
3
v
2
= v
4
...
u
1
= 4u
2
u
3
= 4v
k
v
k1
= v
k+1
Như vậy tồn tại n = k + 1 để với cặp (u
0
, u
1
) bất kỳ thỏa (1) thì ta (u
1
, u
0
) = (v
k+1
, v
k+2
).
!
Bài toán tương tự [Canada MO 1998].
Cho
m
một số nguyên dương. Dãy số (
u
n
) với
n > 0 được xác định như sau
u
0
= 0, u
1
= m
u
n+1
= m
2
u
n
u
n1
với mọi
n >
1. Chứng minh rằng, các cặp số (
a, b
) với
a, b Z
+
, a > b
, nghiệm của phương
trình
a
2
+ b
2
ab + 1
= m
2
khi và chỉ khi (a, b) = (u
n
, u
n+1
) với mọi số tự nhiên n.
Bài toán 11 [Vietnam TST 1992].
Tìm tất cả các nghiệm nguyên dương (
x, y
) của phương
trình
x
2
5xy + y
2
+ 5 = 0. (1)
Lời giải
19
Hướng tới VMO 2020 Tạp chí và liệu toán học
Lời giải và bình luận bài toán y chúng tôi xin được trích từ chuyên đề bước nhảy Viète của thầy
Tuấn Dũng - Khoa Toán - ĐH Phạm Nội 2.
Đầu tiên, chúng ta chứng minh b đề.
Bổ đề. Xét hai y số (u
n
) và (v
n
) được xác định như sau
u
0
= 1, u
1
= 2, u
n+2
= 5u
n+1
u
n
, n = 0, 1, . . . ;
v
0
= 1, v
1
= 3, v
n+2
= 5v
n+1
v
n
, n = 0, 1, . . .
Khi đó, với mọi n N các cặp số (u
n
, u
n+1
) và (v
n
, v
n+1
) nghiệm nguyên dương của (1).
Chứng minh. Ta sẽ chứng minh mệnh đề sau bằng phương pháp quy nạp toán học.
Với
n
= 0, ta
u
2
1
+
u
2
0
5
u
0
u
1
=
5. Do đó (
u
0
, u
1
) nghiệm của phương trình (1). Như vy, mệnh
đề đúng với n = 0.
Giả sử mệnh đề đúng với n = k > 0, tức
u
2
k
+ u
2
k+1
5u
k
u
k+1
+ 5 = 0.
Khi đó
u
2
k+1
+ u
2
k+2
5u
k+1
u
k+2
= u
k+2
(u
k+2
5u
k+1
) + u
2
k+1
= u
2
k+1
+ u
2
k
5u
k
u
k+1
.
Từ giả thiết quy nạp, ta được
u
2
k+1
+ u
2
k+2
5u
k+1
u
k+2
+ 5 = 0.
Do đó (
u
k+1
, u
k+2
) cũng nghiệm của phương trình (1). Theo nguyên quy nạp toán học thì mệnh
đề đúng với mọi
n N
. Chứng minh tương tự, ta cũng thu được với mọi
n N
thì (
v
n
, v
n+1
) nghiệm
của phương trình (1). Từ công thức xác định số hạng tổng quát của hai y số (
u
n
) và (
v
n
) ta được
các số hạng của hai dãy đều các số nguyên dương. Do đó, các cặp số (
u
n
, u
n+1
) và (
v
n
, v
n+1
)
nghiệm nguyên dương của phương trình (1). Như vậy b đề được chứng minh.
Quay lại bài toán. Xét tập hợp
S = {(a, b); a, b Z
+
| a
2
5ab + b
2
+ 5 = 0}.
Với (a, b) S nếu a = b thì ta
3a
2
5 = 0 a
2
=
5
3
,
điều y mâu thuẫn. Do đó
a 6
=
b
. Ta thấy rằng (
a, b
)
S
thì (
b, a
)
S
, không giảm tính tổng quát ta
thể giả sử rằng với mọi (
a, b
)
S
thì
a < b
. Với (
a, b
) một phần tử bất thuộc
S
. Xét y số
(a
n
) được xác định như sau
a
0
= b, a
1
= a, a
n+2
= 5a
n+1
a
n
, n N.
Ta
b
(5
a b
) =
a
2
+ 5
>
0
5
a > b.
Từ công thức xác định số hạng tổng quát của y số (
a
n
) ta
được a
n
Z
+
với mọi n N. Ta (a
0
, a
1
) = (a, b) S, giả sử (a
k
, a
k+1
) S với mọi k > 1, khi đó
a
2
k+1
+ a
2
k+2
5a
k+1
a
k+2
= a
k+2
(a
k+2
5a
k+1
) + a
2
k+1
= a
2
k+1
+ a
2
k
5a
k
a
k+1
.
Từ đây ta được (
a
k+1
, a
k+2
)
S
, theo nguyên quy nạp toán học thì (
a
n
, a
n+1
)
S, n N.
Nếu
a = 1 thì từ (1) ta được b
2
5b + 6 = 0 b {2, 3}..
1. Nếu b = 2 ta (a, b) = (u
0
, u
1
).
2. Nếu b = 3 thì (a, b) = (v
0
, v
1
).
Ta xét trường hợp
a >
1, khi đó (4
a b
)(
a b
) = 3
a
2
5
>
0,
a < b
nên 4
a < b
,
a
0
> a
1
nên từ
đây ta được
a
n
> a
n+1
với mọi
n N
. Như vậy với
a >
1 thì y (
a
n
) một y giảm ngặt, nên phải
tồn tại một chỉ số
k
sao cho
a
0
> a
1
> ··· > a
k+1
= 1. Do (
a
k
, a
k+1
) một nghiệm của phương trình
(1) nên ta
a
k
{
2
,
3
}.
Với
a
k
= 2 thì ta
a
k+1
=
u
0
,
a
k
=
u
1
, khi đó
a
k1
= 5
a
k
a
k+1
=
u
2
, từ
đó ta được
a
i
=
u
k+1i
. Tương tự với
a
k
= 3 thì (
a, b
) các số hạng liên tiếp trên y (
v
n
). Như vậy,
20
Chinh phục olympic toán Ứng dụng định Viète trong các bài toán số học
các b (
u
n
, u
n+1
) và (
v
n
, v
n+1
) (với mọi
n N
) tập tất cả các nghiệm nguyên dương của phương
trình (1).
!
Nhận xét. Ta thiết lập quan hệ thứ tự trên S như sau nếu (x, y) S, (x
0
, y
0
) S thì
(x, y) > (x
0
, y
0
) x > x
0
và y > y
0
Từ một nghiệm bất của phương trình (1) bằng phương pháp bước nhảy Viète ta thiết lập được
mới nhỏ hơn nghiệm (
a, b
) theo quan hệ thứ tự nói trên. Từ nghiệm mới vừa thu được này ta lại
y dựng nghiệm mới nhỏ hơn, cứ tiếp tục quá trình như vậy đến khi không thể y dựng được
nữa. Khi đó, ta thu được nghiệm nhỏ nhất. Dãy (
a
n
) đã tả các nghiệm của phương trình (1)
được y dựng từ quá trình trên và được xây dựng dựa vào các tính chất:
a, b
hai số hạng đầu
tiên của dãy: (
a
i
, a
i+1
) một nghiệm của phương trình (1). Để xác định được công thức truy
hồi của y (a
n
) ta đã sử dụng phương pháp bước nhảy Viète. Xét phương trình
T
2
5T a
n+1
+ a
2
n+1
+ 5 = 0
một nghiệm a
n
, gọi nghiệm còn lại a
n+2
thì theo hệ thức Viète ta
a
n
+ a
n+2
= 5a
n+1
a
n
a
n+2
= a
2
n+1
+ 5
(2)
Từ đây, ta a
n+2
số nguyên dương, do đó (a
n
, a
n+2
) cũng một nghiệm của phương trình,
và từ (2) ta được
a
n+2
= 5
a
n+1
a
n
. Sau khi thu được nghiệm nhỏ nhất, ta xây dựng các nghiệm
của phương trình từ nghiệm nhỏ nhất đó thông qua hai dãy (u
n
) và (v
n
).
Bài toán 12 [VMO 2002]. Tìm tất cả các giá trị nguyên dương k sao cho phương trình
(x + y + z + t)
2
= k
2
xyzt
nghiệm nguyên dương.
Lời giải
Biến đổi phương trình ban đầu về dạng
x
2
+ (2y + 2z + 2t k
2
yzt)x + (y + z + t)
2
= 0
Trong các nghiệm nguyên dương của phương trình, ta chọn ra b nghiệm (
x
0
, y
0
, z
0
, t
0
) tổng
x
0
+ y
0
+ z
0
+ t
0
nhỏ nhất. Khi đó dễ thấy x
0
một nghiệm của phương trình bậc hai
x
2
+ (2y
0
+ 2z
0
+ 2t
0
k
2
y
0
z
0
t
0
)x + (y
0
+ z
0
+ t
0
)
2
= 0 (*)
Gọi nghiệm còn lại của () x
1
, theo định Viète ta
x
0
+ x
1
= k
2
y
0
z
0
t
0
2y
0
2z
0
2t
0
(1)
x
0
.x
1
= (y
0
+ z
0
+ t
0
)
2
(2)
Từ (1) ta
x
1
nguyên và từ (2) ta
x
1
dương. Như vậy (
x
1
, y
0
, z
0
, t
0
) cũng một b số thỏa (
),
nhưng tính nhỏ nhất của tổng x
0
+ y
0
+ z
0
+ t
0
ta x
1
> x
0
. Do đó từ (2) ta suy ra
x
1
=
(y
0
+ z
0
+ t
0
)
2
x
0
> x
0
y
0
+ z
0
+ t
0
> x
0
Kết hợp với (1) ta được
k
2
y
0
z
0
t
0
2y
0
2z
0
2t
0
x
0
> x
0
k
2
y
0
z
0
t
0
> 2x
0
+ 2(y
0
+ z
0
+ t
0
) > 4x
0
.
21
Hướng tới VMO 2020 Tạp chí và liệu toán học
Chia hai vế của đẳng thức
x
2
0
+
y
2
0
+
z
2
0
+
t
2
0
+ 2
x
0
y
0
+ 2
x
0
z
0
+ 2
x
0
t
0
+ 2
y
0
z
0
+ 2
y
0
t
0
+ 2
z
0
t
0
=
k
2
x
0
y
0
z
0
t
0
cho x
0
y
0
z
0
t
0
, ta được
x
0
y
0
z
0
t
0
+
y
0
x
0
z
0
t
0
+
z
0
x
0
y
0
t
0
+
t
0
x
0
y
0
z
0
+
2
z
0
t
0
+
2
y
0
t
0
+
2
y
0
z
0
+
2
x
0
t
0
+
2
x
0
z
0
+
2
x
0
y
0
= k
2
Không mất tính tổng quát ta giả sử rằng x
0
> y
0
> z
0
> t
0
> 1. Khi đó suy ra
x
0
y
0
z
0
t
0
6
k
2
4
,
y
0
z
0
t
0
x
0
6
1
z
0
t
0
6 1 ,
z
0
x
0
y
0
t
0
6
1
x
0
t
0
6 1,
t
0
x
0
y
0
z
0
6
1
x
0
y
0
6 1
Như vậy ta được
k
2
6
k
2
4
+ 1 + 1 + 1 + 2 + 2 + 2 + 2 + 2 + 2 =
k
2
4
+ 15 k
2
6 20 k {1, 2, 3, 4}
1. Nếu k = 1, phương trình nghiệm (4, 4, 4, 4).
2. Nếu k = 2, phương trình nghiệm (2, 2, 2, 2).
3. Nếu k = 3, phương trình nghiệm (1, 1, 2, 2).
4. Nếu k = 4, phương trình nghiệm (1, 1, 1, 1).
Như vậy để phương trình nghiệm nguyên dương thì tập hợp tất cả các giá trị nguyên dương của
k
k {1, 2, 3, 4}.
Bài toán 13 [IMO 2003].
y tìm tất cả các cặp số nguyên dương (
a
;
b
) sao cho
a
2
2ab
2
b
3
+ 1
một số nguyên dương.
Lời giải
Giả sử tồn tại cặp số nguyên dương (
a, b
) thỏa mãn điều kiện bài toán. Đặt
k
=
a
2
2ab
2
b
3
+ 1
thì
k
một số nguyên dương. Cố định k và xét tập hợp
S = {(a, b); a, b Z
+
| a
2
2akb
2
+ k(b
3
1) = 0}.
Như vậy ta ta S 6= . Do k Z
+
nên với (a, b) S ta 2ab
2
b
3
+ 1 > 0 suy ra
b
2
(2a b) > 1 b
2
(2a b) > 0.
Do đó 2a = b hoặc 2a > b. Nếu 2a > b thì do k > 1 nên ta được
a
2
> 2ab
2
b
3
+ 1 > b
2
(2a b) > b
2
.
Từ đó suy ra nếu (
a, b
)
S
thì 2
a
=
b
hoặc
a > b
. Gọi (
a
0
, b
0
) một phần tử bất thuộc
S
. Xét
phương trình
T
2
2
T kb
2
0
+
k
(
b
3
0
1) = 0 phương trình bậc hai ẩn
T
một nghiệm
a
0
. Gọi
nghiệm còn lại a
1
, theo định Viète ta
a
0
+ a
1
= 2kb
2
0
a
0
a
1
= k
b
3
0
1
(1)
Như vậy ta được
a
1
Z
và
a
1
>
0. Nếu
a
1
= 0, thì từ (1) ta
b
0
= 1 và
a
0
= 2
k
, như thế thì (2
k,
1)
một cặp số thỏa mãn điều kiện bài toán. Nếu
a
1
Z
+
thì (
a
1
, b
0
)
S
. Không giảm tính tổng quát
ta thể giả sử
a
1
> a
0
. Chú ý rằng, theo nhận xét trên thì 2
a
0
=
b
0
hoặc
a
0
> b
0
. Nếu
a
0
> b
0
thì
ta ngay a
1
> a
0
> b
0
, kết hợp với (1) ta thu được
kb
2
0
6 a
1
=
k(b
3
0
1)
a
0
6
k(b
3
0
1)
b
0
< kb
2
0
.
Điều y mâu thuẫn. Với 2
a
0
=
b
0
thì ta được (
k,
2
k
) một cặp số thỏa mãn điều kiện bài toán. Từ
hệ thức
a
0
a
1
=
k
(
b
3
0
1) ta thu được (8
k
3
1
,
2
k
) một cặp số cần tìm. Vậy các cặp số (
a, b
) thỏa
mãn điều kiện bài toán (2k, 1), (k, 2k) và (8k
3
1, 2k) với k số nguyên không âm.
22
Chinh phục olympic toán Ứng dụng định Viète trong các bài toán số học
Bài toán 14.
Cho phương trình
x
2
+
y
2
+
z
2
+
t
2
N xyzt N
= 0 trong đó
N
một số nguyên
dương cho trước.
a)
Chứng tỏ rằng, số giá trị nguyên dương
N
để phương trình trên nghiệm nguyên
dương (nghĩa mỗi nghiệm gồm 4 số nguyên dương x, y, z, t).
b)
Cho
N
= 4
k
(8
m
+ 7) với
k, m
các số nguyên không âm. Chứng minh rằng, khi đó phương
trình trên không nghiệm nguyên dương.
Lời giải
a) Biến đổi phương trình tương đương
x
2
+ y
2
+ z
2
+ t
2
Nxyzt N = 0 (1)
t(t Nxyzt) = N
x
2
+ y
2
+ z
2
(2)
Với ba số nguyên dương bất kỳ a, b, c và N = a
2
+ b
2
+ c
2
thì dễ thấy phương trình (2) nghiệm
x
0
= a, y
0
= b, z
0
= c, t
0
= Nabc =
a
2
+ b
2
+ c
2
abc (*)
Chú ý rằng khi hoán vị bốn số a, b, c, Nabc ta lại được nghiệm (x
1
, y
1
, z
1
, t
1
) của phương trình (1).
b) Giả sử phương trình (1) nghiệm nguyên dương, chọn
(x
0
, y
0
, z
0
, t
0
)
nghiệm nguyên dương của
(1) sao cho tổng x
0
+ y
0
+ z
0
+ t
0
số nguyên dương nhỏ nhất. Không làm mất tính chất tổng quát,
giả sử rằng
x
0
6 y
0
6 z
0
6 t
0
. Ta sẽ chứng minh rằng với
N >
7 thì nghiệm nguyên dương của phương
trình (1) với x
0
6 y
0
6 z
0
6 t
0
nếu phải dạng () như trên.
Theo giả thiết t
0
nghiệm của phương trình bậc hai
t
2
Nx
0
y
0
z
0
t + x
2
0
+ y
2
0
+ z
2
0
N = 0 (3)
Phương trình (3) nghiệm thứ hai t
1
thoả mãn
t
1
+ t
0
= N (x
0
y
0
z
0
) (4)
t
1
· t
0
= x
2
0
+ y
2
0
+ z
2
0
N (5)
Từ (4) suy ra t
1
Z. Lại theo giả thiết ta
N (1 + x
0
y
0
z
0
t
1
) = t
2
1
+ x
2
0
+ y
2
0
+ z
2
0
> 0
nên ta được
t
1
>
1
x
0
y
0
z
0
,
t
1
Z
nên
t
1
>
0. Giả sử
t
1
>
0 khi đó
(x
0
, y
0
, z
0
, t
1
)
nghiệm nguyên
dương của (1). Do cách chọn (x
0
, y
0
, z
0
, t
0
) thì
x
0
+ y
0
+ z
0
+ t
1
> x
0
+ y
0
+ z
0
+ t
0
t
1
> t
0
Từ đó theo (5) ta
t
2
0
6 t
1
t
0
= x
2
0
+ y
2
0
+ z
2
0
N < x
2
0
+ y
2
0
+ z
2
0
6 3z
2
0
Ta
N
1 + x
0
y
0
z
2
0
6 N (1 + x
0
y
0
z
0
t
0
) = x
2
0
+ y
2
0
+ z
2
0
+ t
2
0
6 z
2
0
+ z
2
0
+ z
2
0
+ 3z
2
0
= 6z
2
0
Từ đó, do N > 7, nên ta suy ra được
N
1 + x
0
y
0
z
2
0
6 6z
2
0
< Nz
2
0
1 + x
0
y
0
z
2
0
< z
2
0
Điều vô y chứng tỏ t
1
> 0 sai, suy ra t
1
= 0. Từ (4), (5) suy ra
N = x
2
0
+ y
2
0
+ z
2
0
t
0
= Nx
0
y
0
z
0
nghiệm (
) của phương trình (1). Với
N
= 4
k
(8
m
+ 7)
>
7, áp dụng kết quả trên thì
N
=
x
2
+
y
2
+
z
2
.
Do đó nếu chứng minh được phương trình
x
2
+
y
2
+
z
2
= 4
k
(8
m
+ 7) không nghiệm nguyên dương
thì phương trình (1) cũng không nghiệm nguyên dương. Ta xét các trường hợp sau.
23
Hướng tới VMO 2020 Tạp chí và liệu toán học
1.
Khi
k
= 0 ta
x
2
+
y
2
+
z
2
= 8
m
+ 7 hay
x
2
+
y
2
+
z
2
7(
mod
8). Trong ba số
x, y, z
phải
một số lẻ hoặc cả ba số lẻ. Nếu số a lẻ thì a
2
1 (mod 8), do đó x
2
+ y
2
+ z
2
6= 7(mod8).
2. Khi k > 0 ta
x
2
+ y
2
+ z
2
= 4
k
(8m + 7) (**)
hay
x
2
+
y
2
+
z
2
0(
mod
4). Trong ba số
x, y, z
phải một số chẵn hoặc ba số chẵn. Nếu
một số chẵn, còn hai số
a, b
lẻ thì
a
2
+
b
2
2(
mod
4), suy ra
x
2
+
y
2
+
z
2
6
= 7(
mod
8). Nếu
x, y, z
đều chẵn, đặt x = 2x
1
, y = 2y
1
, z = 2z
1
thì (∗∗) tương đương với
x
2
+ y
2
+ z
2
= 4
k1
(8m + 7)
Sau
k
lần biến đổi như thế ta
x
2
+
y
2
+
z
2
= 8
m
+ 7, nhưng phương trình y nghiệm
nguyên dương như khi xét k = 0.
Bài toán được giải quyết.
5 Các bài toán tổng hợp.
5.1 Đề bài
Câu 1.
Cho
a, b, k
các số nguyên dương thỏa mãn
k
=
a
2
+ ab + b
2
ab + 1
. Chứng minh rằng
k
một số
chính phương.
Câu 2. Cho các số nguyên dương a, b thỏa mãn
ab(5a
2
+ 5b
2
2)
5ab 1
Z. Chứng minh rằng a = b.
Câu 3.
Cho các số nguyên dương
x, y, A
thỏa mãn hệ thức
A
=
x
2
+ y
2
+ 30
xy
. Chứng minh rằng
A
lũy thừa bậc năm của một số nguyên.
Câu 4.
Cho các số nguyên dương
x, y, z
thỏa mãn
x
2
+ y
2
+ z
2
xyz + 1
nhận giá trị nguyên dương. Chứng
minh rằng
x
2
+ y
2
+ z
2
xyz + 1
thể biểu diễn được thành tổng của hai số chính phương.
Câu 5.
Cho
a, b, c
các số nguyên dương thỏa mãn 0
< a
2
+
b
2
abc 6 c
. Chứng minh rằng
a
2
+
b
2
abc
số chính phương.
Câu 6.
Cho
m > n
các số nguyên dương lẻ và
n
2
1 chia hết cho
m
2
n
2
+ 1. Chứng minh rằng
m
2
n
2
+ 1 một số chính phương.
Câu 7.
Cho
x
và
y
các số nguyên dương thỏa mãn điều kiện
x
2
+
y
2
+ 1 chia hết cho 2
xy
+ 1. Chứng
minh rằng x = y.
Câu 8.
Cho
a, b
các số nguyên dương lẻ thỏa mãn
a
2
+ 2 chia hết cho
b
và
b
2
+ 2 chia hết cho
a
.
Chứng minh rằng
a
2
+ b
2
+ 2
ab
số chính phương.
Câu 9.
Cho các số nguyên dương
a, b, c, d
thỏa mãn
b
2
+ 1 =
ac
và
c
2
+ 1 =
bd
. Chứng minh rằng
a + c = 3b và b + d = 3c.
Câu 10.
Giả sử phương trình
x
2
+
y
2
+
x
+
y
+ 1 =
xyz
nghiệm nguyên dương. Tìm tất cả các giá
trị của z.
Câu 11. Tìm các số nguyên dương x và y sao cho x + 1 chia hết cho y và y + 1 chia hết cho x.
Câu 12. Tìm các số nguyên dương x, y để x
2
+ 2 chia hết cho xy + 1.
Câu 13.
Tìm tất cả các số ba chữ số chia hết cho 11 sao cho thương số của phép chia số đó cho 11
bằng tổng bình phương của các chữ số của số đó.
Câu 14 [Kiran Kedlaya].
Cho các số nguyên dương
a, b, c
thỏa mãn
(ab + 1) (bc + 1) (ca + 1)
số chính phương. Chứng minh rằng ba số ab + 1; bc + 1; ca + 1 đều số chính phương.
Câu 15. Tồn tại hay không năm số nguyên dương a
1
; a
2
; a
3
; a
4
; a
5
thỏa mãn hệ điều kiện
a
2
2
+ 1 = (a
1
+ 1) (a
3
+ 1)
a
2
3
+ 1 = (a
2
+ 1) (a
4
+ 1)
a
2
4
+ 1 = (a
3
+ 1) (a
5
+ 1)
24
Chinh phục olympic toán Ứng dụng định Viète trong các bài toán số học
5.2 Hướng dẫn giải - Lời giải
Câu 1.
Cho
a, b, k
các số nguyên dương thỏa mãn
k
=
a
2
+ ab + b
2
ab + 1
. Chứng minh rằng
k
một số chính phương.
Lời giải. Trước tiên ta sẽ cố định k và xét tập
S =
(a, b) N × N | k =
a
2
+ ab + b
2
ab + 1
Giả sử phản chứng
k
không số chính phương, khi đó trong các phần tử của
S
ta chọn ra cặp (
A, B
)
thỏa mãn điều kiện
A
+
B
nhỏ nhất. Không mất tính tổng quát, ta giả sử rằng
A > B >
0. Xét
phương trình bậc hai ẩn x
k =
x
2
+ xB + B
2
xB + 1
x
2
+ (B kB)x + B
2
k = 0
Phương trình y hiển nhiên hai nghiệm A và x
0
, khi đó theo định Viète ta
x
0
+ A = kB B (1)
x
0
.A = B
2
k (2)
Từ (1) ta suy ra được x
0
số nguyên.
1.
Nếu
x
0
<
0 thì
x
0
6
1
x
2
(
Bk B
)
x
+
B
2
k > x
2
+ (
Bk B
) +
B
2
k >
0, điều này
mâu thuẫn.
2. Nếu x
0
= 0 thì k = B
2
một số chính phương, trường hợp y loại.
3. Nếu x
0
> 0 thì (x
0
, B) S.
Như vậy ta được
x
0
+ B =
B
2
k
A
+ B <
B
2
A
+ B <
A
2
A
+ B = A + B
điều y mâu thuẫn với tính nhỏ nhất của tổng
A
+
B
. Do đó giả thiết phản chứng sai, từ đó ta
k phải một số chính phương.
Câu 2.
Cho các số nguyên dương
a, b
thỏa mãn
ab(5a
2
+ 5b
2
2)
5ab 1
Z
. Chứng minh rằng
a
=
b
.
Lời giải.
gcd
(
ab,
5
ab
1) = 1 nên ta
5a
2
+ 5b
2
2
5ab 1
Z
. Đặt
5a
2
+ 5b
2
2
5ab 1
=
k Z
, ta dễ dàng
được
5(a
2
+ b
2
) 2 > 10ab 2 > 5ab 1 k =
5a
2
+ 5b
2
2
5ab 1
> 1 k > 2.
Xét tập
S
=
(a, b) Z
+
× Z
+
| k =
5a
2
+ 5b
2
2
5ab 1
Z
. Cố định
k
và trong các phần tử của
S
, ta
chọn ra cặp số (
A, B
) nguyên dương thỏa mãn tổng
A
+
B
nhỏ nhất. Gỉa sử
A 6
=
B
, không mất tính
tổng quát, xét A > B. Xét phương trình bậc hai ẩn x
5x
2
+ 5B
2
2
5xB 1
= k 5x
2
5xBk + 5B
2
+ k 2 = 0
Dễ thấy phương trình y một nghiệm A, gọi nghiệm còn lại x
0
. Theo định Viète, ta
A + x
0
= Bk (1)
Ax
0
=
5B
2
+ k 2
5
(2)
Từ (1) ta x
0
nguyên.
25
Hướng tới VMO 2020 Tạp chí và liệu toán học
1.
Nếu
x
0
<
0 thì
x
0
6
1
5
x
2
0
5
x
0
Bk
+ 5
B
2
+
k
2
>
5 + 5
Bk
+ 5
B
2
+
k
2
>
0, điều này
mâu thuẫn.
2. Nếu x
0
> 0 thì (x
0
, B) S. Khi đó do tính nhỏ nhất của tổng A + B ta
x
0
> A
5B
2
+ k 2
5A
> A
5A
2
+ 5B
2
2
5AB 1
2 > 5(A B)(A + B)
5(A B)
2
5AB 1
> 5(A B)(A + B)
A B > (A + B)(5AB 1).
ràng điều y vô lí.
Như vậy phải x
0
= 0, suy ra 5B
2
= 2 k > 0, lại k > 2, do đó k = 2. Suy ra
5a
2
+ 5b
2
2
5ab 1
= 2 (a b)
2
= 0 a = b
Bài toán được giải quyết.
Câu 3.
Cho các số nguyên dương
x, y, A
thỏa mãn hệ thức
A
=
x
2
+ y
2
+ 30
xy
. Chứng minh rằng
A lũy thừa bậc năm của một số nguyên.
Lời giải.
Gọi
(x
0
; y
0
)
cặp số thỏa mãn đề bài và tổng
x
0
+
y
0
nhỏ nhất. Ta giả sử
x
0
6 y
0
. Xét
phương trình bậc hai ẩn y
y
2
A.x
0
.y + x
2
0
+ 30 = 0 (*)
(x
0
; y
0
)
thỏa mãn đề bài nên
y
0
một nghiệm của phương trình (
). Gọi nghiệm còn lại
y
1
. Theo
định Viète ta
y
0
+ y
1
= Ax
0
(1)
y
0
y
1
= x
2
0
+ 30 (2)
Ta
x
0
, y
0
, A Z
nên từ (1) suy ra
y
1
Z
. Các cặp
(x
0
; y
0
)
;
(x
0
; y
1
)
đều thỏa mãn (
)
x
0
+
y
0
nhỏ nhất nên ta được
x
0
+ y
0
6 x
0
+ y
1
y
0
6 y
1
.
Như vậy x
0
6 y
0
6 y
1
. Ta xét các trường hợp sau.
Trường hợp 1. Nếu x
0
= y
0
khi đó ta thay vào A thì ta được
A = 2 +
30
x
2
0
Z x
0
= 1 A = 32
Trường hợp 2. Nếu y
0
= y
1
thì từ (2) ta được
x
2
0
+ 30 = y
2
0
(y
0
+ x
0
) (y
0
x
0
) = 30
Dễ thấy
y
0
+
x
0
;
y
0
x
0
cùng tính chẵn lẻ 30 = 1
.
30 = 2
.
15 = 5
.
6 = 3
.
10. Trường hợp y
không xảy ra.
Trường hợp 3. Nếu x
0
< y
0
< y
1
, thì ta suy ra
y
0
> x
0
+ 1
y
1
> x
0
+ 2
Do đó từ (2) suy ra
x
2
0
+ 30 > (x
0
+ 1) (x
0
+ 2) x
0
6 9
Khi
x
0
= 9 thì từ (2) suy ra
y
0
y
1
= 9
2
+ 30 = 111,
y
0
< y
1
(y
0
; y
1
)
= (1; 111); (3; 37). Điều
y phải
x
0
< y
0
. Tương tự khi xét
x
= 1; 2; 3; 4; 5; 6; 7; 8. Tất cả đều dẫn đến vô lí.
Trường hợp này loại.
Do đó ta luôn A = 32 = 2
5
lũy thừa bậc năm của một số nguyên.
26
Chinh phục olympic toán Ứng dụng định Viète trong các bài toán số học
Câu 4.
Cho các số nguyên dương
x, y, z
thỏa mãn
x
2
+ y
2
+ z
2
xyz + 1
nhận giá trị nguyên dương.
Chứng minh rằng
x
2
+ y
2
+ z
2
xyz + 1
thể biểu diễn được thành tổng của hai số chính phương.
Lời giải.
Ta đặt
n
=
x
2
+ y
2
+ z
2
xyz + 1
, khi đó ta sẽ chứng minh
n
tổng của hai số chính phương. Viết
lại đẳng thức trên thành
x
2
+
y
2
+
z
2
=
n (xyz + 1)
. Giả sử
(x
0
; y
0
; z
0
)
một b số nguyên dương
thỏa mãn yêu cầu bài toán, điều đó nghĩa
x
2
0
+ y
2
0
+ z
2
0
= n (x
0
y
0
z
0
+ 1)
Hay ta viết lại được
x
2
0
nx
0
y
0
z
0
+ y
2
0
+ z
2
0
n = 0
Xét phương trình bậc hai
x
2
nxy
0
z
0
+
y
2
0
+
z
2
0
n
= 0, khi đó ta thấy
x
0
một nghiệm của phương
trình. Theo định Viète thì ngoài nghiệm
x
0
phương trình còn một nghiệm nữa, ta gọi nghiệm đó
x
1
. Như vậy theo định Viète ta
x
1
+ x
0
= ny
0
z
0
x
1
x
0
= y
2
0
+ z
2
0
n
Từ hệ thức trên ta suy ra được
x
1
nhận giá trị nguyên. Không mất tính tổng quát ta chọn
x
0
+
y
0
+
z
0
bé nhất và x
0
> y
0
> z
0
. Ta xét các trường hợp sau
Trường hợp 1. Nếu y
2
0
+ z
2
0
< n, khi đó x
1
số nguyên âm. Từ đó suy ra
0 = x
2
1
ny
0
z
0
x
1
+ y
2
0
+ z
2
0
n > x
2
1
+ n + y
2
0
+ z
2
0
n = x
2
1
+ y
2
0
+ z
2
0
> 0
điều y lí.
Trường hợp 2.
Nếu
y
2
0
+
z
2
0
> n
, khi đó
x
1
số nguyên dương. Khi đó
(x
1
; y
0
; z
0
)
một b số
nguyên dương thỏa mãn yêu cầu bài toán. Theo cách chọn
(x
0
; y
0
; z
0
)
ta suy ra được
x
0
6 x
1
.
Khi đó từ định Viète ta
y
2
0
+ z
2
0
n ny
0
z
0
= (x
0
1) (x
1
1) 1 > (x
0
1)
2
1
Ta lại xét 2 khả năng sau.
1. Nếu x
0
> y
0
thì ta được (x
0
1)
2
1 > y
2
0
1. Do đó
y
2
0
+ z
2
0
n ny
0
z
0
> y
2
0
1
Từ đó suy ra z
2
0
+ 1 > ny
0
z
0
+ n > n
z
2
0
+ 1
, như vậy ta n = 1.
2. Nếu x
0
= y
0
, khi đó ta được 2y
2
0
+ z
2
0
= n
y
2
0
z
0
+ 1
. Do đó
z
2
0
= y
2
0
(nz
0
2) + n > z
2
0
(nz
0
2) + n > z
2
0
(nz
0
2)
Từ đây ta suy ra được
nz
0
<
3 nên
n
= 1 hoặc
n
= 2, chú ý rằng
n
= 1 =
0
2
+
1
2
và
n = 2 = 1
2
+ 1
2
.
Trường hợp 3.
Nếu
y
2
0
+
z
2
0
=
n
thì nghĩa
n
viết được thành tổng của hai số chính phương.
Vậy bài toán được chứng minh xong.
Câu 5.
Cho
a, b, c
các số nguyên dương thỏa mãn 0
< a
2
+
b
2
abc 6 c
. Chứng minh rằng
a
2
+ b
2
abc số chính phương.
27
Hướng tới VMO 2020 Tạp chí và liệu toán học
Lời giải.
Giả sử tồn tại các số nguyên dương
a, b, c
thỏa mãn 0
< a
2
+
b
2
abc 6 c
k
=
a
2
+
b
2
abc
không phải số chính phương. Khi đó ta 0
< k 6 c
. Không mất tính tổng quát ta giả sử
a > b
. Xét
phương trình bậc hai ẩn x
x
2
bcx + b
2
k = 0
Khi đó
a
một nghiệm của phương, khi đó theo định Viète thì phương trình còn một nghiệm
nữa x = a
1
. Từ đó ta được
a + a
1
= bc
a.a
1
= b
2
k
Như vậy ta suy ra a
1
số nguyên.
1.
Nếu
a
1
= 0, khi đó từ hệ thức
a.a
1
=
b
2
k
ta được
k
=
b
2
số chính phương, điều này mâu
thuẫn với giả sử trên.
2. Nếu a
1
< 0, khi đó k = a
2
1
+ b
2
a
1
bc > a
2
1
+ b
2
+ bc > c, mâu thuẫn do 0 < k 6 c.
Như vậy ta được a
1
số nguyên dương. Cũng theo định Viète ta
a
1
=
b
2
k
a
b
2
k
a
< a
a
1
< a
Ta thấy cặp số
(a
1
; b)
cũng một nghiệm. Khi đó ta
a
1
+
b < a
+
b
, điều y sẽ vô khi ta chọn
cặp số
(a; b)
với
a
+
b
bé nhất. Như vy không thể tồn tại
k
để
k
=
a
2
+
b
2
abc
không phải số
chính phương. Như vy a
2
+ b
2
abc phải số chính phương.
Câu 6 [Taiwan MO 1998].
Cho
m > n
các số nguyên dương lẻ và
n
2
1 chia hết cho
m
2
n
2
+ 1. Chứng minh rằng m
2
n
2
+ 1 một số chính phương.
Lời giải.
Theo giả thiết ta
n
2
1 chia hết cho
m
2
n
2
+ 1 nên ta được
m
2
m
2
n
2
+ 1
chia
hết cho m
2
n
2
+ 1. Từ đó suy ra
m
2
n
2
+ 1
| m
2
Từ điều trên ta suy ra tồn tại k để
m
2
= k
m
2
n
2
+ 1
đây ta chú ý rằng
m
2
=
m + n
2
+
m n
2
2
và
m
2
n
2
+1 = 4
.
m + n
2
.
m n
2
1. Do
m
và
n
các số
nguyên dương lẻ, lại
m > n
nên
m + n
2
;
m n
2
các số nguyên dương, đặt
x
=
m + n
2
;
y
=
m n
2
,
khi đó ta được
(x + y)
2
= k (4xy + 1)
y giờ ta sẽ đi chứng minh rằng 4
xy
+ 1 số chính phương, tuy nhiên trước tiên ta cần phải chứng
minh
k
số chính phương. Thật vậy, giả sử cặp số nguyên dương
(x
0
; y
0
)
với
x
0
+
y
0
nhỏ nhất thỏa
mãn thỏa đẳng thức trên, khi đó ta
(x
0
+ y
0
)
2
= k (4x
0
y
0
+ 1)
Xét phương trình bậc hai ẩn
x
x
2
(4k 2) y
0
x
+
y
2
0
k
= 0. Khi đó
x
0
một nghiệm của phương
trình trên. Như vậy theo định Viète thì phương trình còn một nghiệm nữa x
1
, lúc y ta
x
0
+ x
1
= (4k 2) y
0
x
0
.x
1
= y
2
0
k
Từ hệ thức thứ nhất
x
0
+
x
1
=
(4k 2) y
0
ta suy ra được
x
1
số nguyên. Ta xét các trường hợp sau
28
Chinh phục olympic toán Ứng dụng định Viète trong các bài toán số học
Nếu x
1
< 0 thì từ hệ thức thứ hai x
0
.x
1
= y
2
0
k ta được y
2
0
k < 0 y
2
0
< k, ta suy ra
x
2
1
(4k 2) y
0
x
1
+ y
2
0
k = (x
1
+ y
0
)
2
+ k (4x
1
1) > 0
điều y mâu thuẫn x
1
nghiệm của phương trình.
Nếu x
1
= 0, khi đó từ x
0
.x
1
= y
2
0
k ta được y
2
0
k = 0 k = y
2
0
số chính phương.
Nếu
x
1
>
0 thì ta được
y
2
0
k >
0
k > y
2
0
. Khi đó
(x
1
; y
0
)
một nghiệm của phương trình
(x + y)
2
= k (4xy + 1). Theo cách chọn cặp số (x
0
; y
0
) ta
x
0
+ y
0
6 x
1
+ y
0
y
0
6 x
0
6 x
1
Kéo theo
y
2
0
(4k 2) y
2
0
+
y
2
0
k
=
(4 4k) y
2
0
k >
0, điều y
k
số nguyên dương.
Vậy ta được k số chính phương nên dẫn đến m
2
n
2
+ 1 số chính phương.
Câu 7.
Cho
x
và
y
các số nguyên dương thỏa mãn điều kiện
x
2
+
y
2
+ 1 chia hết cho 2
xy
+ 1.
Chứng minh rằng x = y.
Lời giải.
Do 2
xy
+ 1
| x
2
+
y
2
+ 1 nên ta đặt
k
=
x
2
+ y
2
+ 1
2xy + 1
với
k
số nguyên dương. Nhận thấy
khi
x
=
y
thì
k
= 1 và ngược lại vẫn đúng. Do đó ta đi chứng minh
k
= 1. Giả sử cặp số nguyên dương
(a
0
; b
0
)
với
a
0
+
b
0
bé nhất thỏa mãn yêu cầu bài toán. Không mất tính tổng quát ta giả sử
a
0
> b
0
.
Xét phương trình
k =
x
2
+ b
2
0
+ 1
2xb
0
+ 1
x
2
2kb
0
x + b
2
0
+ 1 k = 0
Khi đó ta
0
= b
2
0
k
2
b
2
0
1 + k = (k 1)
b
2
0
(k + 1) + 1
> 0
Do đó phương trình luôn nghiệm. Dễ thấy
a
0
một nghiệm của phương trình nên theo định
Viète thì phương trình còn thêm một nghiệm nữa a
1
. Khi đó ta
a
0
+ a
1
= 2kb
0
a
0
.a
1
= b
2
0
k + 1
Từ hệ thức thứ nhất ta được a
1
số nguyên. Giả sử a
1
< 0, khi đó
a
2
1
2kb
0
a
1
+ b
2
0
+ 1 k > a
2
1
+ 2kb
0
+ b
2
0
+ 1 k > 0
điều y vô
a
1
một nghiệm của phương trình
x
2
2
kb
0
x
+
b
2
0
+ 1
k
= 0, suy ra
a
1
>
0. Ta xét
các trường hợp sau.
Trường hợp 1. Nếu a
1
= 0, khi đó ta được a
1
b
0
= 0, điều y dẫn đến xy = 0.
Trường hợp 2. Nếu a
1
> 0, khi đó nếu a
0
> b
0
thì ta được
a
1
+ b
0
=
b
2
0
k + 1
a
0
+ b
0
<
a
2
0
1 + 1
a
0
+ b
0
= a
0
+ b
0
Đều y mâu thuẫn với các chọn (a
0
; b
0
) với a
0
+ b
0
bé nhất.
Như vậy a
0
= b
0
, suy ra k =
2a
2
0
+ 1
2a
2
0
+ 1
= 1. Do đó ta được x = y.
Câu 8.
Cho
a, b
các số nguyên dương lẻ thỏa mãn
a
2
+ 2 chia hết cho
b
và
b
2
+ 2 chia hết cho
a. Chứng minh rằng
a
2
+ b
2
+ 2
ab
số chính phương.
29
Hướng tới VMO 2020 Tạp chí và liệu toán học
Lời giải.
Ta thấy rằng
a, b
2 số nguyên tố cùng nhau. Thật vy nếu
d
=
(a, b)
thì do
b | a
2
+ 2 nên
d | a
2
+ 2, lại
d | a
nên
d |
2. Mặt khác
a, b
số lẻ nên suy ra
d
= 1. Do
b | a
2
+ 2 và
a | b
2
+ 2 nên
ab |
a
2
+ 2
b
2
+ 2
ab | 2a
2
+ 2b
2
+ 4
Do
a
và
b
số lẻ nên suy ra
ab | a
2
+
b
2
+ 2. Khi đó tồn tại số nguyên dương
k
sao cho
a
2
+
b
2
+ 2 =
kab
.
y giờ ta sẽ đi chứng minh
k
số chính phương. Trước tiên thử một vài giá trị đặc biệt ta nhận thấy
được
k
= 4. Đến đây thì tưởng giống như các dụ trên đó sử dụng định Viète đển chứng
minh
k
= 4 thỏa mãn bài toán. Giả sử cặp số dương
(a
0
; b
0
)
với
a
0
+
b
0
nhỏ nhất thỏa mãn yêu cầu
bài toán, tức ta sẽ
a
2
0
+ b
2
0
+ 2 ka
0
b
0
= 0
Không mất tính tổng quát ta giả sử a
0
> b
0
. Xét phương trình bậc hai ẩn a
a
2
kb
0
a + b
2
0
+ 2 = 0
Khi đó ta thấy
a
0
một nghiệm của phương trình trên. Khi đó theo định Viète thì phương trình
trên còn một nghiệm nữa, ta gọi a
1
, từ đó ta
a
0
+ a
1
= kb
0
a
0
+ a
1
= b
2
0
+ 2
Từ hệ thức trên ta thu được
a
1
số nguyên dương. Như vy cặp số
(a
1
; b
0
)
cũng thỏa mãn yêu cầu
bài toán. Theo cách chọn cặp số (a
0
; b
0
) ta được
a
0
+ b
0
6 a
1
+ b
0
a
0
6 a
1
Từ đó ta
a
0
= kb
0
a
1
6 kb
0
a
0
a
0
b
0
6
k
2
Mặt khác từ điều kiện a
2
0
+ b
2
0
+ 2 ka
0
b
0
= 0 ta lại được
a
0
b
0
+
b
0
a
0
+
2
a
0
b
0
= k
Do
a
0
b
0
6
k
2
nên k 6
k
2
+ 2 + 1 hay k 6 6. Theo bất đẳng thức AM GM ta
a
2
0
+ b
2
0
> 2a
0
b
0
k > 2
Đến đây ta xét các trường hợp sau
1.
Nếu
k
= 3, khi đó ta được
a
2
0
+
b
2
0
+ 2 = 3
a
0
b
0
. Từ đó suy ra 3
| a
2
0
+
b
2
0
+ 2 Do số chính phương
chia 3 1 nên trong hai số chính phương
a
2
0
và
b
2
0
phải một số chia hết cho 3 và một số
không chia hết cho 3. Từ đó suy ra vế phải chia hết cho 9 còn vế trái không chia hết cho 9. Điều
y dẫn đến mâu thuẫn.
2. Nếu k 6= 4, khi đó từ
a
0
b
0
+
b
0
a
0
+
2
a
0
b
0
= k ta được (a
0
; b
0
) 6= (1; 1).
Như vậy ta được
k 6
k
2
+ 1 + 1 hay
k 6
4, do đó
k
= 5 và
k
= 6 không thỏa mãn toán. Vậy ta suy
ra được chỉ k = 4 thỏa mãn yêu cầu bài toán.
Câu 9.
Cho các số nguyên dương
a, b, c, d
thỏa mãn
b
2
+ 1 =
ac
và
c
2
+ 1 =
bd
. Chứng minh
rằng a + c = 3b và b + d = 3c.
Lời giải. Từ giả thiết ta thấy (b, c) = 1. Thật vy, gọi (b, c) = k khi đó ta
b = b
0
k
c = c
0
k
kac
0
| k
2
b
0
2
+ 1
kdb
0
| k
2
c
0
2
+ 1
30
Chinh phục olympic toán Ứng dụng định Viète trong các bài toán số học
Từ đó ta được k
2
adb
0
c
0
|
k
2
b
0
2
+ 1
k
2
c
0
2
+ 1
, suy ra
k
2
|
k
2
b
0
2
+ 1
k
2
c
0
2
+ 1
k
2
| k
4
b
0
2
c
0
2
+ k
2
b
0
2
+ k
2
c
0
2
+ 1 k
2
| 1 k = 1
Hơn nữa từ giả thiết ta lại
b
2
+ 1 = ac
c
2
+ 1 = bd
do vậy ta được
b | c
2
+ 1
c | b
2
+ 1
.
Như vậy
ab | b
2
+
c
2
+ 1 hay
b
2
+
c
2
+ 1 =
mbc
với số nguyên dương
m
nào đó. Đến đây sử dụng kết
quả của
bài toán 4
trong phần
bước nhảy Viète
ta được
m
= 3, khi đó thì
b
2
+
c
2
+ 1 = 3
bc
. Từ
đây suy ra
b
2
+ 1 = ac
c
2
+ 1 = bd
b
2
+ c
2
+ 1 = c
2
+ ac
b
2
+ c
2
+ 1 = b
2
+ bc
ac + c
2
= 3bc
b
2
+ bd = 3bc
Hay ta được
a + c = 3b
b + d = 3c
, vậy bài toán được chứng minh.
Câu 10.
Giả sử phương trình
x
2
+
y
2
+
x
+
y
+ 1 =
xyz
nghiệm nguyên dương. Tìm tất cả
các giá trị của z.
Lời giải.
Trước tiên ta giả sử cặp số nguyên dương
(x
0
; y
0
)
với
x
0
+
y
0
nhỏ nhất thỏa mãn thỏa đẳng
thức trên. Không mất tính tổng quát ta giả sử x
0
> y
0
, khi đó ta được
x
2
0
+ y
2
0
+ x
0
+ y
0
+ 1 = zx
0
y
0
Xét phương trình bậc hai ẩn x
y
2
+ (1 zx
0
) y + x
2
0
+ x
0
+ 1 = 0
Khi đó
y
0
một nghiệm của phương trình trên, theo hệ thức Viète thì phương trình còn một
nghiệm nữa y
1
. Do đó ta được
y
0
+ y
1
= zx
0
1
y
0
.y
1
= x
2
0
+ x
0
+ 1
Từ các hệ thức trên ta suy ra được
y
1
giá trị nguyên dương. Khi đó xặp số nguyên dương
(y
1
; x
0
)
một nghiệm của phương trình đã cho. Theo cách chọn cặp số
(x
0
; y
0
)
ta suy ra được
y
0
6 y
1
. Ta xét
các trường hợp sau.
Nếu x
0
= y
0
thì từ x
2
0
+ y
2
0
+ x
0
+ y
0
+ 1 = zx
0
y
0
ta suy ra
2x
2
0
+ 2x
0
+ 1 = zx
2
0
Từ đó ta được x
0
= y
0
= 1 và z = 5.
Nếu x
0
> y
0
thì ta được y
0
< x
0
< y
1
, khi đó ta được
x
2
0
+ (1 zx
0
) x
0
+ x
2
0
+ x
0
+ 1 < 0 x < 2 +
2
x
0
+
1
x
2
0
Đến đây ta xét các giá trị x
0
> 2 đều không tìm được z thỏa mãn.
Như vậy z = 5 giá trị thỏa mãn yêu cầu đề bài.
Câu 11.
Tìm các số nguyên dương
x
và
y
sao cho
x
+ 1 chia hết cho
y
và
y
+ 1 chia hết cho
x
.
Lời giải.
Do
x
+ 1 chia hết cho
y
và
y
+ 1 chia hết cho
x
nên ta được
(x + 1) (y + 1)
chia hết cho
xy
.
Từ đó suy ra
xy | (x + y + 1) xy | (x + y + 1)
2
31
Hướng tới VMO 2020 Tạp chí và liệu toán học
Đến đây biến đổi tiếp ta được
xy |
x
2
+ y
2
+ 2x + 2y + 1
. Khi đó tồn tại số nguyên dương
k
sao cho
x
2
+
y
2
+ 2
x
+ 2
y
+ 1 =
kxy
. Giả sử cặp số nguyên dương
(x
0
; y
0
)
với
x
0
+
y
0
nhỏ nhất thỏa mãn yêu
cầu bài toán. Khi đó ta được
x
2
0
+ y
2
0
+ 2x
0
+ 2y
0
+ 1 kx
0
y
0
= 0
Không mất tính tổng quát ta giả sử x
0
> y
0
. Xét phương trình bậc hai ẩn x
x
2
+ (2 ky
0
) x + (y
0
+ 1)
2
= 0
Khi đó
x
0
một nghiệm của phương trình. Như vậy theo định Viète thì phương trình trên còn
một nghiệm khác x
1
. Do đó ta được
x
0
+ x
1
= ky
0
2
x
0
.x
1
= (y
0
+ 1)
2
Từ hệ thức
x
0
+
x
1
=
ky
0
2 ta suy ra được
x
1
số nguyên. Từ hệ thức
x
0
.x
1
=
(y
0
+ 1)
2
ta suy ra
được
x
1
số nguyên dương. Chú ý rằng lúc này cặp số nguyên dương
(x
1
; y
0
)
thỏa mãn bài toán. Ta
x
0
+ y
0
6 x
1
+ y
0
x
0
6 x
1
do vậy ta được y
0
6 x
0
6 x
1
. Đến đây ta xét các trường hợp sau
1. Nếu x
0
= y
0
, khi đó ta được
2x
2
0
+ 4x
0
+ 1 = kx
2
0
k = 2 +
1
x
2
0
+
4
x
0
Do
k
số nguyên dương nên ta suy ra được
x
0
= 1 thỏa mãn, do đó suy ra
x
0
=
y
0
= 1 thỏa
mãn bài toán.
2.
Nếu
x
0
=
x
1
> y
0
>
1, khi đó từ
x
0
.x
1
=
(y
0
+ 1)
2
ta được
x
0
=
y
0
+ 1. Thay vào hệ thức
x
0
+ x
1
= ky
0
2 ta được
2x
0
= k (x
0
1) 2 k = 2 +
4
x
0
1
Do k và
x
0
các số nguyên dương nên ta tìm được
x
0
= 2,
x
0
= 3 và
x
0
= 5, tương ứng thì
y
0
= 1,
y
0
= 2 và
y
0
= 4. Tuy nhiên khi thử vào bài toán thì ta thấy hai cặp số nguyên dương
thỏa mãn đó (2; 1) , (3; 2).
3. Nếu x
1
> x
0
> y
0
> 1, khi đó ta được x
1
> x
0
+ 1 > y
0
+ 2. Từ đó ta được
x
0
.x
1
> (y
0
+ 2) (y
0
+ 1) > (y
0
+ 1)
2
Do đó trường hợp này không thỏa mãn.
Vậy các cặp số nguyên dương thỏa mãn bài toán (x; y) = (1; 1) , (1; 2) , (2; 1) , (2; 3) , (3; 2).
Câu 12. Tìm các số nguyên dương x, y để x
2
+ 2 chia hết cho xy + 1.
Lời giải.
Ta biến đổi giả thiết thành
x
2
mxy
+ 2
m
= 0, khi đó giả sử cặp số nguyên dương
(x
0
; y
0
)
với
x
0
+
y
0
bé nhất thỏa mãn yêu cầu bài toán, tức ta
x
2
0
+ 2 =
m (x
0
y
0
+ 1)
. Xét phương trình
bậc hai ẩn x
x
2
mxy
0
m + 2 = 0
Khi đó
x
0
một nghiệm của phương trình trên. Theo định Viète thì phương trình trên còn một
nghiệm nữa, gọi nghiệm đó x
1
. Khi đó ta
x
0
+ x
1
= my
0
x
0
.x
1
= 2 m
32
Chinh phục olympic toán Ứng dụng định Viète trong các bài toán số học
Từ hệ thức
x
0
+
x
1
=
my
0
ta suy ra
x
1
số nguyên. Mặt khác nếu
x
1
<
0, khi đó ta được
x
1
6
1.
Do đó
x
2
1
mx
1
y
0
m + 2 > x
2
1
+ my
0
m + 2 > 0
Điều này mâu thuẫn với
x
1
nghiệm của phương trình trên. Như vậy
x
1
>
0 hay
x
1
số nguyên
không âm. Do đó từ hệ thức x
0
.x
1
= 2 m ta được
2 m > 0 m {1; 2}
Ta xét các trường hợp sau
1.
Nếu
m
= 1, khi đó ta được
x (x y)
=
1 nên ta được
x
= 1;
y
= 2 thỏa mãn yêu cầu bài toán.
2.
Nếu
m
= 2, khi đó
x (x 2y)
= 0. Lại do
x 6
= 0 nên suy ra
x
2
y
= 0
x
= 2
y
. Do đó
x = 2k; y = k với k số nguyên dương thỏa mãn yêu cầu bài toán.
Vậy cặp số nguyên dương thỏa mãn (x; y) = (1; 2) , (2k; k) với k số nguyên dương.
Câu 13.
Tìm tất cả các số ba chữ số chia hết cho 11 sao cho thương số của phép chia số đó
cho 11 bằng tổng bình phương của các chữ số của số đó.
Lời giải. Gọi số ba chữ số thỏa mãn yêu cầu bài toán A = abc. Trong đó các chữ số thỏa mãn
a {1; 2; ...9}; b, c {0; 1; 2; ...9}
Do A chia hết cho 11 nên ta được
ab
+
c
chia hết cho 11. Kết hợp với
a {1; 2; ...9}
;
b, c {0; 1; 2; ...9}
ta suy ra được a b + c = 0 hoặc a b + c = 11. Như vậy ta đi xét hai trường hợp sau
Với
ab
+
c
= 0, khi đó ta được
b
=
a
+
c
. Ta
A
= 100
a
+10
b
+
c
= 99
a
+10
b
+
a
+
c
= 99
a
+11
b
.
Khi
A
chia 11 thì thương số của phép chia bằng tổng bình phương các chữ số của
A
nên ta được
A
11
= a
2
+ b
2
+ c
2
9a + b = a
2
+ b
2
+ c
2
Kết hợp với b = a + c ta được
9a + (a + c) = a
2
+ (a + c)
2
+ c
2
10a + c = 2a
2
+ 2ac + 2c
2
Do a > 1 nên suy ra
10a + c > 2a
2
+ 2c + 2c
2
2c
2
+ c 6 10a 2a
2
6
25
2
Do vậy 2
c
2
+
c 6
12
c 6
2. Cũng từ 10
a
+
c
= 2
a
2
+ 2
ac
+ 2
c
2
ta suy ra được
c
số chẵn.
Từ đó ta được c = 0 hoặc c = 2. Ta xét 2 khả năng.
1. Với c = 0, khi đó ta được a = b nên số cần tìm dạng A = aa0. Do đó
A
11
= 50 = 2a
2
a = 5 a = b = 5
Từ đó ta tìm được A = 550.
2. Với c = 2, khi đó từ 10a + c = 2a
2
+ 2ac + 2c
2
ta được
10a + 2 = 2a
2
+ 4ac + 8 a
2
3a + 3 = 0
Nhận thấy phương trình trên không nghiệm nguyên dương nên không tồn tại số
A
thỏa
mãn bài toán.
33
Hướng tới VMO 2020 Tạp chí và liệu toán học
Với
a b
+
c
= 11, khi đó ta được
b
+ 11 =
a
+
c
. Do
a, b, c
các chữ số nên từ
b
+ 11 =
a
+
c
ta
suy ra được a > 2. Ta
A = 100a + 10b + c = 99a + 10b + a + c = 99a + 11b + 11
Ta xét các khả năng sau.
1. Xét a = 2, khi đó c = 9; b = 0. Ta được A = 209 không thỏa mãn bài toán.
2.
Xét
a
= 3, khi đó ta được
c
= 8;
b
= 0 hoặc
c
= 9;
b
= 1. Ta được
A
= 308 hoặc
A
= 319
không thỏa mãn.
3.
Xét
a >
4, khi đó
A
chia 11 thì thương số của phép chia bằng tổng bình phương các chữ số
của A nên ta được
A
11
= a
2
+ b
2
+ c
2
9a + b + 1 = a
2
+ b
2
+ c
2
Kết hợp với b = a + c 11 ta được
9a + (a + c 11) + 1 = a
2
+ (a + c 11)
2
+ c
2
10a + c 10 = 2a
2
+ 2ac + 2c
2
22 (a + c) + 121
32a + 23c 131 = 2a
2
+ 2ac + 2c
2
Do a > 4 nên suy ra
32a + 23c 131 > 2a
2
+ 8c + 2c
2
2c
2
15c 6 32a 2a
2
131 6 3
Do đó suy ra 2
c
2
15
c 6
3
c 6
7. Từ 32
a
+ 23
c
131 = 2
a
2
+ 2
ac
+ 2
c
2
ta suy ra
được
c
số lẻ. Do đó ta được
c
= 1; 3; 5; 7. Đến đây xét các trường hợp của
c
thì được
b = 0; a = 8 thỏa mãn.
Do đó số cần tìm A = 803.
Vậy các số thỏa mãn yêu cầu bài toán 550 và 803.
Câu 14 [Kiran Kedlaya].
Cho các số nguyên dương
a, b, c
thỏa mãn
(ab + 1) (bc + 1) (ca + 1)
số chính phương. Chứng minh rằng ba số ab + 1; bc + 1; ca + 1 đều số chính phương.
Lời giải.
Từ giả thiết ta nhận thấy để chứng minh
ab
+ 1;
bc
+ 1;
ca
+ 1 đều số chính ta cần chỉ ta
được ba số
ab
+ 1;
bc
+ 1;
ca
+ 1 nguyên tố với nhau theo từng đôi một. Tuy nhiên với lượng thông
tin hạn chế từ giả thiết ta không thể chứng minh được nhận định trên. Với ý tưởng sử dụng định
Viète, ta cần tạo ra một phương trình bậc hai nghiệm nguyên và biệt thức chứa biểu thức
(ab + 1) (bc + 1) (ca + 1)
. Ngoài ra do ta cần chứng minh
ab
+ 1;
bc
+ 1;
ca
+ 1 đều số chính phương
nên khi biến đổi phương trình bậc hai thì cần chứa các đại lượng
ab
+ 1;
bc
+ 1;
ca
+ 1. Xét phương
trình bậc hai ẩn t sau
t
2
+ a
2
+ b
2
+ c
2
2 (ab + bc + ca + ta + tb + tc) 4abct 4 = 0
t
2
2t (a + b + c + 2abc) + a
2
+ b
2
+ c
2
2 (ab + bc + ca) 4 = 0
Ta nhận thấy phương trình bậc hai trên tương đương với ba phương trình sau
(a + b c t)
2
= 4 (ab + 1) (ct + 1)
(a + c b t)
2
= 4 (ac + 1) (bt + 1)
(b + c a t)
2
= 4 (bc + 1) (at + 1)
Giải phương trình bậc hai trên ta được
t
1
= a + b + c + 2abc + 2
p
(ab + 1) (bc + 1) (ca + 1)
t
2
= a + b + c + 2abc 2
p
(ab + 1) (bc + 1) (ca + 1)
34
Chinh phục olympic toán Ứng dụng định Viète trong các bài toán số học
Do
(ab + 1) (bc + 1) (ca + 1)
số chính phương nên
t
nhận các giá trị nguyên. Từ ba phương trình
trên ta được
4
3
(ab + 1) (ac + 1) (bc + 1) (ct + 1) (bt + 1) (at + 1)
số chính phương, do đó (ct + 1) (bt + 1) (at + 1) số chính phương, ta lại
at + 1 > 0; bt + 1 > 0; ct + 1 > 0
nên
at
+ 1
>
0;
bt
+ 1
>
0;
ct
+ 1
>
0. Trong các b số nguyên dương
(a; b; c)
thỏa mãn bài toán ta xét
b số
(a; b; c)
sao cho
a
+
b
+
c
nhỏ nhất. Không mất tính tổng quát ta chọn
c
=
max {a; b; c}
. Dễ thấy
a
=
b
=
c
= 1 không thỏa mãn bài toán nên
c
=
max {a; b; c} >
1, do đó
t >
1
max {a; b; c}
>
1. Ta
xét các trường hợp sau
1. Nếu t = 0, khi đó từ phương trình bậc hai ta được
(a + b + c)
2
= 4 (ab + bc + ca) + 4 (a + b c)
2
= 4 (ab + 1)
Suy ra
ab
+ 1 số chính phương. Chứng minh hoàn toàn tương tự ta được
bc
+ 1;
ca
+ 1 các
số chính phương.
2.
Nếu
t >
0, khi đó do
a
+
b
+
c
giá trị nhỏ nhất nên ta được
t > c
và
t
chỉ nhận một trong hai
giá trị
t
1
= a + b + c + 2abc + 2
p
(ab + 1) (bc + 1) (ca + 1)
t
2
= a + b + c + 2abc 2
p
(ab + 1) (bc + 1) (ca + 1)
theo định Viète ta
t
1
t
2
= a
2
+ b
2
+ c
2
2 (ab + bc + ca) 4 6 c
2
a (2c a) b (2c b) < c
2
Điều y dẫn đến mâu thuẫn. Nên trường hợp y không giá trị t thỏa mãn.
Như vậy bài toán đã được giải quyết hoàn toàn.
Câu 15. Tồn tại hay không năm số nguyên dương a
1
; a
2
; a
3
; a
4
; a
5
thỏa mãn hệ điều kiện
a
2
2
+ 1 = (a
1
+ 1) (a
3
+ 1)
a
2
3
+ 1 = (a
2
+ 1) (a
4
+ 1)
a
2
4
+ 1 = (a
3
+ 1) (a
5
+ 1)
Lời giải.
Trước hết ta kiểm tra tính chẵn lẻ của các số đã cho trước khi các đánh giá hợp lí. Giả
sử
a
1
số lẻ, khi đó
a
2
số lẻ nên
a
2
2
+ 1 chia 4 2, từ đó suy ra
a
3
+ 1 số lẻ, dẫn đến
a
3
số
chẵn. Điều này
a
2
+ 1 số chẵn nên không thể ước của số lẻ
a
2
3
+ 1. Do vậy
a
1
phải số
chẵn. Lập luận hoàn toàn tương tự ta được
a
2
;
a
3
;
a
4
;
a
5
cùng số chẵn. Đặt
a
2
=
x
;
a
3
=
y
. Khi đó
từ hệ điều kiện trên ta được
y
2
+ 1
.
.
. (x + 1)
và
x
2
+ 1
.
.
. (y + 1)
. Ta sẽ chứng minh rằng không tồn
tại cặp số chẵn x và y thỏa mãn điều trên.
Giả sử tồn tại cặp số nguyên dương (
x, y
) thỏa mãn
y
2
+ 1
.
.
. (x + 1)
và
x
2
+ 1
.
.
. (y + 1)
, khi đó ta
được
y
2
+ 1 + x
2
1
.
.
. (x + 1)
x
2
+ y
2
.
.
. (x + 1)
Tương tự ta cũng
x
2
+ y
2
.
.
. (y + 1)
. Gọi
d
=
(x + 1, y + 1)
, khi đó
d
ước của
x
2
+1;
y
2
+1;
x
2
+
y
2
.
Do đó
x
2
+ 1 + y
2
+ 1
x
2
+ y
2

.
.
.d 2
.
.
.d
Như vy thì
d
= 1 hoặc
d
= 2.
x
+ 1;
y
+ 1 các số lẻ, do đó ta được
d
= 1 hay
x
+ 1;
y
+ 1 nguyên
tố cùng nhau. Từ đó dẫn đến
x
2
+ y
2
.
.
. (x + 1) (y + 1)
35
Hướng tới VMO 2020 Tạp chí và liệu toán học
Khi đó tồn tại số nguyên dương k sao cho
x
2
+ y
2
= k (x + 1) (y + 1)
Trong các cặp số nguyên dương
(x; y)
thỏa mãn ta chọn cặp số nguyên dương
(x
0
; y
0
)
với
x
0
+
y
0
bé
nhất. Không mất tính tổng quát ta giả sử x
0
> y
0
. Xét phương trình bậc hai ẩn x
x
2
k (y
0
+ 1) x + y
2
0
k (y
0
+ 1) = 0
Khi đó
x
0
một nghiệm của phương trình. Theo định Viète thì còn một nghiệm nữa
x
1
. Khi
đó ta
x
0
+ x
1
= k (y
0
+ 1)
x
0
x
1
= y
2
0
k (y
0
+ 1)
Nếu
x
1
= 0, khi đó từ
x
0
x
1
=
y
2
0
k (y
0
+ 1)
ta được
y
2
0
=
k (y
0
+ 1)
nên suy ra
y
2
0
.
.
. (y
0
+ 1)
, điều y
vô y
0
và y
0
+ 1 nguyên tố cùng nhau. Do đó x
1
6= 0. Mặt khác ta
(x
0
+ 1) (x
1
+ 1) = x
0
x
1
+ x
0
+ x
1
+ 1 = y
2
0
+ 1
nên x
1
+ 1 số lẻ. Do đó x
1
> 0 số chẵn. Đồng thời ta cũng
x
1
+ 1 =
y
2
0
+ 1
x
0
+ 1
6
y
2
0
+ 1
y
0
+ 1
6 y
0
6 x
0
Như vậy cặp số (y
0
; x
1
) một nghiệm của
x
2
+ y
2
= k (x + 1) (y + 1)
Cặp số
(y
0
; x
1
)
thỏa mãn
y
0
+
x
1
6 y
0
+
x
0
, điều này vô ta đã chọn
x
0
+
y
0
bé nhất. Vy điều
ta giả sử sai hay không tồn tại các số chẵn
x, y
thỏa mãn. Như vậy không tồn tại năm số nguyên
dương thỏa mãn yêu cầu bài toán.
!
Nhận xét. Từ bài toán y, ta thể giải được các bài toán tương tự sau.
1.
Cho các số nguyên dương
x, y
thỏa mãn
x
2
+ 1 chia hết cho
y
+ 1 và
y
2
+ 1 chia hết cho
x + 1. Chứng minh rằng x và y các số lẻ.
2.
Tìm số nguyên dương
k
để phương trình
x
2
+
y
2
=
k (x + 1) (y + 1)
nghiệm nguyên
dương.
3. Tìm số tự nhiên n bé nhất sao cho tồn tại số nguyên dương a
1
; a
2
; ...; a
n
thỏa mãn
a
2
k
= (a
k1
+ 1) (a
k+1
+ 1)
với các số 2 6 k 6 n 1.
6 Bài tập tự luyện.
Câu 1.
Tìm tất cả các số tự nhiên
a, b, c
sao cho tồn tại số nguyên dương
n, m, k
thỏa mãn các điều
kiện sau
a =
m
2
+ b
2m
; b =
n
2
+ c
2n
; c =
k
2
+ a
2k
Câu 2. Tìm nghiệm nguyên của phương trình sau x
3
+ x
2
y + xy
2
+ y
3
= 8
x
2
+ xy + y
2
+ 1
.
Câu 3. Tìm nghiệm nguyên của phương trình 3
x
2
xy + y
2
= 7 (x + y).
Câu 4 [Putnam 1998]. Chứng minh rằng với mỗi số thực N thì phương trình
x
2
1
+ x
2
2
+ x
2
3
+ x
2
4
= x
1
x
2
x
3
+ x
1
x
2
x
4
+ x
1
x
3
x
4
+ x
2
x
3
x
4
nghiệm (a
1
, a
2
, a
3
, a
4
) với a
1
, a
2
, a
3
, a
4
các số nguyên lớn hơn N.
Câu 5. Giả sử bốn số nguyên a, b, c, d đôi một khác nhau và thoả mãn hệ điều kiện sau
a
2
2ac 5d = b
2
2bc 5d = 0
c
2
2ca 5b = d
2
2bd 5b = 0
36
Chinh phục olympic toán Ứng dụng định Viète trong các bài toán số học
Chứng minh rằng a + b + c + d một hợp số.
Câu 6. Tìm các số nguyên dương x, y sao cho
x + y
x
y + 2 = 0.
Câu 7 [Turkey National Olympiad 2015]. Với m, n các số nguyên dương sao cho
k =
(m + n)
2
4m(m n)
2
+ 4
cũng số nguyên. Chứng minh rằng k số chính phương.
Câu 8.
Cho
p
một số nguyên dương. Giả sử phương trình
x
2
+
px
+ 1 = 0 hai nghiệm
a
1
;
a
2
và phương trình x
2
+ qx + 1 = 0 hai nghiệm b
1
; b
2
. Chứng minh rằng
(a
1
b
1
) (a
2
b
1
) (a
1
+ b
2
) (a
2
+ b
2
)
hiệu của hai số chính phương.
Câu 9. Tìm các cặp số nguyên (a; b) sao cho hai số a
2
+ 4b và b
2
+ 4a đều số chính phương.
Câu 10. Tìm các chữ số a, b, c, d, e thỏa mãn điều kiện ab + cde =
abcde.
Câu 11.
Cho
a, b
các số nguyên dương thỏa mãn
a
2
+
b
2
chia hết cho
ab
. Tính giá trị của biểu thức
A =
a
2
+ b
2
ab
Câu 12 [Đề thi trường Đông phía Bắc 2015].
Tìm tất cả các số nguyên dương
k
sao cho phương
trình x
2
(k
2
4)y
2
+ 24 = 0 nghiệm nguyên dương.
Câu 13.
Chứng minh rằng tất cả các nghiệm nguyên dương của phương trình
x
2
+
y
2
+ 1 = 3
xy
(x, y) = (F
2k1
, F
2k+1
) với F
n
số Fibonacci.
Câu 14. Tìm tất cả các số nguyên dương n sao cho phương trình sau nghiệm nguyên dương
x
2
+ y
2
= n(x + 1)(y + 1).
Câu 15.
Giả sử
a, b
các số nguyên dương thỏa mãn
b
+ 1
| a
2
+ 1
, a
+ 1
| b
2
+ 1
.
Chứng minh rằng
a
,
b đều các số lẻ.
Câu 16.
Chứng minh rằng nếu
a, b
các số nguyên dương sao cho
k
=
a
2
+ b
2
+ 6
ab
nguyên thì
k
= 8.
Câu 17. Chứng minh rằng số cặp số nguyên dương (a; b) thỏa mãn
a + 1
b
+
b + 1
a
= 4.
CHÚC C BẠN THI TỐT!
37
Hướng tới VMO 2020 Tạp chí và liệu toán học
Tài liệu
[1] Bước nhảy Viète - Tuấn Dũng, Đại học phạm Nội 2.
[2] Bước nhảy Viète - Phạm Huy Hoàng, Chuyên đề số học Mathscope.
[3] Đặng Hùng Thắng, Nguyễn Văn Ngọc, Vũ Kim Thủy - Bài giảng số học. NXB Giáo dục 1996.
[4] Vận dụng định Viète giải các bài toán số học - Nguyễn Công Lợi.
[5] Lời giải và bình luận VMO 2012 - Trần Nam Dũng. Diễn đàn Mathscope, 2012.
[6] The Method of Vieta Jumping - Yimin Ge, Mathematical Reflections 5 (2007).
[7] A Rational Function Whose Integral Values Are Sums of Two Squares - Sam Vandervelde.
[8] Diễn đàn AoPS Online, https://artofproblemsolving.com/community.
[9] Diễn đàn toán học Việt Nam - VMF, https://diendantoanhoc.net/.
38
| 1/38

Preview text:

ỨNG DỤNG ĐỊNH LÝ VIÈTE
TRONG CÁC BÀI TOÁN SỐ HỌC
Tạp chí và tư liệu toán học sưu tầm và tổng hợp Hướng tới VMO 2020 Ngày 14 tháng 12 năm 2019 Tóm tắt nội dung
Trong chương trình toán lớp 9, chúng ta đã được tìm hiểu đến một định lý cực kì nối tiếng đó là
định lý Viète, tuy nhiên ứng dụng của nó không chỉ có là biểu diễn mối quan hệ của các nghiệm
trong phương trình đa thức, mà còn ứng dụng trong nhiều mảng khác như số học, đa thức,... Ở bài
viết này, chúng tôi sẽ giới thiệu cho bạn đọc một số các bài toán số học có sử dụng định lý Viète và
nâng cao hơn nữa là phương pháp bước nhảy Viète - Vieta Jumping - để giải quyết các bài toán số
học hay và khó. Trong bài viết có tham khảo các tư liệu trong và ngoài nước, các bạn xem ở mục
tài liệu tham khảo ở cuối bài viết. Mọi ý kiến thắc mắc, đóng góp vui lòng gửi về địa chỉ.
1. Doãn Quang Tiến. fb.com/profile.php?id=100016406718327
2. Nguyễn Minh Tuấn. fb.com/tuankhmt.fpt 1
Nhà toán học Francois Viète.
Francois Viète (1540-1603) là nhà toán học Pháp vĩ đại. Ông là người đầu tiên đưa ra các kí hiệu bằng
chữ, do thế, người ta gọi ông là người cha của môn Đại số. Tên tuổi của ông gắn liền với một định lí về
nghiệm số của phương trình mà học sinh lớp 9 đều biết đó là định lí Viète, nhưng công lao của ông to lớn hơn nhiều.
Ông vốn là một trạng sư, từng làm “cố vấn cơ mật” cho các triều vua Henry III và Henri IV. Giữa
những bộn rộn của công việc ở cung đình, hễ có ít phút rảnh rỗi là ông lại giải trí bằng cách... nghiên
cứu Toán học! Trong cuộc chiến tranh Pháp... Tây Ban Nha thời ấy, quân Tây Ban Nha thường liên
lạc với những kẻ nội phản trong nước Pháp bằng các mật thư. Vì được viết bằng các mật mã gồm
toàn các chữ số, nên các mật thư ấy hầu như không thể khám phá được.
Biết vị “cố vấn” Viète thích toán, vua Henry III đã nhờ ông thử dò tìm “chìa khóa” các mật thư
này. Nhận lời, suốt hai tuần lễ, ông làm việc quên ăn quên ngủ. Cuối cùng, chính Viète đã xé tung
tấm màn bí mật: ông đã tìm ra quy luật thay thế các chữ và số trong cách viết mật thư. Đọc được
các mật thư, quân Pháp đã làm thất bại hoàn toàn những mưu đồ của Tây Ban Nha. Về phía địch,
chúng gắng dò tìm nguyên nhân: cuối cùng chúng biết được những kí hiệu đã bị phơi trần, dù nhiều
lần thay đổi mật mã, và kẻ tìm ra bí mật là Francois Viète! Quân Tây Ban Nha tuyên bố Viète là
kẻ tử thù và đã xử án hỏa thiêu vắng mặt ông, nhưng bản án dã man đó không bao giờ thực hiện
được. Không chỉ quan tâm sâu sắc đến Đại số; nghiên cứu các phương trình, Viète còn nghiên cứu
cả Hình học và Lượng giác. Ông cũng đã khảo cứu kĩ lưỡng nhiều công trình của các nhà toán học thời cổ.
Phần lớn cuộc đời của Viète bị các công việc pháp lí của nghề trạng sư chiếm mất nên khó có thể
tưởng tượng ông đã lấy đâu ra thời gian để làm nên những công trình toán học của mình. Bí quyết
của ông chính là khả năng tập trung cao độ khi làm việc. Người ta còn kể lại, lúc gặp đươc một vấn đề
thú vị, ông có thể ngồi ở bàn làm việc suốt ba ngày đêm liền. 1 Hướng tới VMO 2020
Tạp chí và tư liệu toán học 2 Định lý Viète.
Định lý Viète được trình bày trong sách giáo khoa toán 9 - tập 2, cho ta mối quan hệ giữa các nghiệm
của phương trình bậc hai và các hệ số của nó. Sau đây ta sẽ nhắc lại nó.
Định lý Viète. Nếu phương trình bậc hai ax2 + bx + c = 0 (a 6= 0) có hai nghiệm x1 và x2 thì
tổng và tích của chúng là  −b  S = x1 + x2 = a c  P = x1.x2 = a
Ngược lại nếu có hai số x1 và x2 thỏa mãn S = x1 + x2 và P = x1.x2 thì x1 và x2 là hai nghiệm
của phương trình t2 − St + P = 0.
Chú ý rằng trong khi giải toán, đôi khi ta không quan tâm tới giá trị của x1 và x2 mà chỉ cần quan
tâm đến 2 giá trị tổng và tích của chúng, từ đó ta có những đánh giá cần thiết. Ngoài ra cũng từ định
lí Viète ta nhận thấy nếu một phương trình bậc hai ax2 + bx + c = 0 có một nghiệm x1 thì nó sẽ có
thêm một nghiệm x2 nữa. Ngoài ra ta có thể mở rộng định lý cho phương trình đa thức bậc n bất kì.
Cho phương trình a0 + a1x + a2x2 + ... + anxn = 0, an 6= 0. Gọi x1, x2, . . . , xn là n nghiệm của phương trình trên, khi đó thì
a0 + a1x + a2x2 + ... + anxn = a(x − x1)(x − x2)...(x − xn)
Nhân toàn bộ vế phải ra, chúng ta sẽ có công thức Viète, được phát biểu như sau a = a  n    −a(x  1 + x2 + ... + xn) = an−1    . . . . . .     (−1)n−1a(x 
1x2...xn−1 + x1x2...xn−2xn + ... + x2x3...xn) = a1    (−1)na(x1x2...xn) = a0
và trong hàng k bất kỳ, vế phải của đẳng thức là an−kan−k còn vế trái được tính theo công thức
(−1)ka nhân với tổng của các tích từng cụm (n − k) các nghiệm của phương trình trên. ∇ 3 Các bài toán cơ bản.
Sau đây ta sẽ đi tìm hiểu một vài ví dụ trước khi đi tìm hiểu về phương pháp bước nhảy Viète.
Bài toán 1. Tìm tất cả các giá trị của m để phương trình x2 − mx + m + 2 = 0 có các nghiệm đều nguyên. Lời giải
Lời giải 1. Điều kiện để phương trình có nghiệm là ∆ = m2 − 4 (m + 2) > 0. Phương trình đầu có
nghiệm nguyên thì ∆ phải là số chính phương, tức là tồn tại số nguyên k sao cho m2 − 4 (m + 2) = k2. Ta có
m2 − 4 (m + 2) = k2 ⇔ (m − 2 + k) (m − 2 − k) = 12
Đến đây ta có một chú ý rằng, theo định lí Viète, ta có x1 + x2 = m, do vậy mà m sẽ là số nguyên. Từ
đây do m và k là các số nguyên nên ta tìm được m = −2 hoặc m = 6. Ta xét các trường hợp.
1. Với m = −2 thì ta được phương trình x2 + 2x = 0, khi đó tìm được hai nghiệm nguyên là x1 = 0 và x2 = −2. 2 Chinh phục olympic toán
Ứng dụng định lí Viète trong các bài toán số học
2. Với m = 6 thì ta được phương trình x2 − 6x + 8 = 0, khi đó ta được hai nghiệm nguyên là x1 = 2 và x2 = 8.
Như vậy đến đây bài toán đã được giải quyết.
Lời giải 2. Theo định lý Viète với hai nghiệm x1; x2 thì ta có x1 + x2 = m (1) x1.x2 = m + 2
Do đó nếu ta tìm được các nghiệm nguyên của phương trình thì ta sẽ tìm được giá trị của m. Điều này
làm ta có ý tưởng giải phương trình nghiệm nguyên 2 ẩn x1 và x2. Từ (1) ta được
x1.x2 − (x1 + x2) = 2 ⇔ (x1 − 1) (x2 − 1) = 3
Khi đó x1 − 1 và x2 − 1 là các ước của 3, lại có 3 = 1.3 = −1. (−3). Đến đây việc tìm hai nghiệm x1; x2
hoàn toàn đơn giản và qua đó ta tìm được các giá trị m là m = −2 và m = 6.
! Nhận xét. Ở lời giải 1 sẽ có một số học sinh mắc sai lầm là chưa chỉ ra được m là số nguyên vì
đề bài không có đề cập. Do đó ta phải sử dụng định lí Viète để chỉ ra điều này. xy + yz + zx = 8
Bài toán 2. Tìm nghiệm nguyên của hệ phương trình x + y + z = 5 Lời giải
Trước tiên ta sẽ đưa bài toán về việc giải phương trình nghiệm nguyên. Ta biến đổi xy + (y + x) z = 8 xy + (5 − z) z = 8 xy = 8 − (5 − z) z ⇔ ⇔ x + y = 5 − z x + y = 5 − z x + y = 5 − z
Theo định lí Viète thì x và y là hai nghiệm của phương trình bậc hai t2 − (5 − z) t + 8 − (5 − z) z = 0,
trong đó z là tham số. Phương trình có nghiệm thì ∆ > 0 hay ta được 7
3z2 − 10z + 7 6 0 ⇔ 1 6 z 6 3
Do z nguyên nên z = 1 hoặc z = 2. Ta xét các trường hợp sau.
• Với z = 1, khi đó phương trình trở thành t2 − 4t + 4 = 0, đến đây ta tìm được x = y = 2 thỏa mãn.
• Với z = 2 khi đó phương trình trở thành t2 − 3t + 2 = 0, đến đây ta tìm được x = 2; y = 1 hoặc x = 1; y = 2 thỏa mãn.
Vậy hệ phương trình có các nghiệm nguyên là (x; y; z) = (2; 2; 1) , (1; 2; 2) , (2; 1; 2).
Bài toán 3. Giải phương trình x2 − mx + n = 0, biết phương trình có hai nghiệm nguyên dương
phân biệt và m, n là hai số nguyên tố. Lời giải
Với bài toán này nếu dùng công thức nghiệm để xác định nghiệm của phương trình thì sẽ gây cho ta
nhiều khó khăn do phương trình có đến hai tham số. Do vậy ta sẽ sử dụng định lý Viète. Gọi x1; x2 là
các nghiệm nguyên dương của phương trình đã cho (x1 < x2). Ta biết rằng một số nguyên tố khi viết
thành tích hai số thì một thừa số là 1 và một thừa số là chính nó. Để ý ta lại thấy theo định lí Viète
thì x1x2 = n. Do đó rất tự nhiên ta nghĩ đến sử dụng định lí Viète giả quyết bài toán. Thật vậy theo định lí Viète ta được x1 + x2 = m x1x2 = n 3 Hướng tới VMO 2020
Tạp chí và tư liệu toán học
Do n là một số nguyên tố nên ta được x1 = 1; x2 = n, suy ra m = n + 1, do vậy m và n là hai số tự
nhiên liên tiếp nên ta được n = 2; m = 3, thử lại ta thấy tỏa mãn yêu cầu bài toán.
Bài toán 4. Cho phương trình 2x2 + mx + 2n + 8 = 0, trong đó m và n là các tham số nguyên.
Giả sử phương trình có các nghiệm đều là số nguyên. Chứng minh rằng m2 + n2 là hợp số. Lời giải
Để chứng minh được m2 + n2 là hợp số thì một suy nghĩ hết sức tự nhiên đó là xây dựng biểu thức
m2 + n2 theo các nghiệm của phương trình đề rồi từ đó phân tích biểu thức nghiệm thành nhân tử. Có
hai ý tưởng để xây dựng biểu thức m2 + n2 đó là áp dụng công thức nghiệm để tìm các nghiệm của
phương trình rồi từ đó tính m2 + n2 hoặc áp dụng định lí Viète. Rõ ràng trong hai ý tưởng đó việc áp
dụng định lí Viète giúp ta xây dựng biểu thức nghiệm mà không chứa các căn bậc hai.
Gọi x1, x2 là hai nghiệm của phương trình trên, theo định lí Viète ta được ( m x1 + x2 = − 2 x1.x2 = n + 4 Khi đó ta có
m2 + n2 = (2x1 + 2x2)2 + (x1x2 − 4)2 = 4x21 + 4x22x21 + x22x21 + 16 = x21 + 4 x22 + 4
Do x1; x2 là các số nguyên nên x2 + 4; x2 + 4 là các số nguyên dương lớn hơn 1. 1 2
Từ đó ta được m2 + n2 là hợp số. a2 + b2
Nhận xét. Xét 2m = a + b; 2n = a − b, khi đó từ m2 + n2 = , ta có bài toán sau. 2
Bài toán. Cho phương trình 4x2 + (a + b) x + 2 (a − b) + 16 = 0, trong đó a và b là các tham số ! a2 + b2
nguyên. Giả sử phương trình có các nghiệm đều là số nguyên, chứng minh rằng là hợp số. 2
Bài toán tương tự. Giả sử phương trình bậc hai x2 + ax + b + 1 = 0, trong đó a, b là các tham
số nguyên, đồng thời b 6= −1 có hai nghiệm đều là số nguyên khác 0, chứng minh rằng a2 + b2 là hợp số.
Bài toán 5. Tìm các số nguyên dương a và b, trong đó (a > b) sao cho phương trình bậc hai
x2 − abx + a + b = 0 có các nghiệm đều là số nguyên. Lời giải
Điều kiện để phương trình có nghiệm là ∆ = (ab)2 − 4 (a + b) > 0, giả sử phương trình có hai nghiệm
nguyên x1; x2 (x1 6 x2), khi đó theo định lí Viète ta được x1 + x2 = ab x1.x2 = a + b
Do a và b là các số nguyên dương nên suy ra các nghiệm x1; x2 cũng là các số nguyên dương. Ta chú ý
rằng với hai số lớn hơn 2 thì tích của chúng bao giờ cũng lớn hơn tổng của chúng. Do đó ta nghĩ đến
chứng minh một trong bốn số dương trên không vượt quá 2.
Thật vậy, nếu a > 2; b > 2 thì ta có ab > 2a; ab > 2b nên 2ab > 2 (a + b) hay ab > a + b.
Nếu cả bốn số dương x1; x2; a; b đều lớn hơn 2 thì x1.x2 > x1 + x2 và ab > a + b, khi đó định lí Viète
trên không thể xảy ra. Như vậy trong bốn số dương x1; x2; a; b tồn tại ít nhất một số không vượt quá
2. Theo giả thiết và theo cách chọn hai nghiệm x1; x2 thì trong hai số x1 và b có ít nhất một số không
lớn hơn 2. Do vai trò của hai số x1 và b là như nhau nên không mất tính tổng quát ta giả sử rằng
0 < x1 6 2, đến đây ta xét từng trường hợp của x1. 4 Chinh phục olympic toán
Ứng dụng định lí Viète trong các bài toán số học x 1 + x 1. Nếu x 1 + x2 = ab 2 = ab 1 = 1, khi đó từ ta suy ra
, từ đó ta được ab − a − b = 1, x1.x2 = a + b x2 = a + b
suy ra (a − 1) (b − 1) = 2. Chú ý a > b nên từ phương trình trên ta được a = 3; b = 2, khi đó ta tìm được x2 = 3.
2. Nếu x1 = 2, tương tự như trường hợp trên ta tìm được các cặp số nguyên dương (a; b) thỏa mãn là (a; b) = (5; 1) , (2; 2).
Bài toán được giải quyết hoàn toàn.
Bài toán 6. Tìm tất cả các số nguyên tố p, q sao cho tồn tại số tự nhiên m thỏa mãn pq m2 + 1 = p + q m + 1 Lời giải pq m2 + 1 Quan sát hệ thức =
thì hai đại lượng pq và p + q làm ta liên tưởng đến hệ thức Viète. p + q m + 1 m2 + 1 Nếu
là phân số tối giản thì từ hệ thức của bài toán ta được m + 1 p + q = m + 1 pq = m2 + 1 m2 + 1 Còn nếu
chưa tối giản thì chỉ cần rút gọn ta cũng được một hệ điều kiện tương tự. Vấn đề là m + 1 m2 + 1
ta cần kiểm tra xem phân số
có rút gọn được hay không. m + 1 pq m2 + 1 • Nếu p = q thì từ = ta được p + q m + 1 2 m2 + 1 4 p = = 2m − 2 + m + 1 m + 1
Do m ∈ N và p là số nguyên tố nên (m + 1) | 4 ⇒ m = 0; m = 1; m = 3. Từ đó ta tìm được
p = 2; p = 5 thỏa mãn yêu cầu bài toán.
• Nếu p 6= q thì pq và p + q là nguyên tố cùng nhau vì pq chỉ chia hết cho các ước nguyên tố là p
và q còn p + q thì không chia hết cho p và không chia hết cho q. Gọi r là một ước chung của
m2 + 1 và m + 1. Khi đó ta có
r | [(m + 1) (m − 1)] ⇒ r | m2 − 1
Do đó r | m2 + 1 − m2 − 1 ⇒ r | 2 suy ra r = 1 hoặc r = 2. p + q = m + 1 1. Với r = 1 suy ra
, khi đó p và q là hai nghiệm của phương trình pq = m2 + 1 x2 − (m + 1) x + m2 + 1 = 0
Ta có ∆ = −3m2 + 2m − 3 = −(m − 1)2 − 2m2 + 2 < 0 nên phương trình trên vô nghiệm. 2pq = m2 + 1 2. Với r = 2 suy ra
, khi đó p và q là hai nghiệm của phương trình 2 (p + q) = m + 1 2x2 − (m + 1) x + m2 + 1 = 0
Ta có ∆ = −7m2 + 2m − 7 = −(m − 1)2 − 6m2 + 6 < 0 nên phương trình vô nghiệm.
Vậy bộ các số nguyên tố cần tìm là (p; q) = (2; 2) , (5; 5). 5 Hướng tới VMO 2020
Tạp chí và tư liệu toán học
Bài toán 7. Tìm số nguyên tố p để p2 − p + 1 là lập phương của một số nguyên tố khác. Lời giải
Do p là số nguyên tố nên ta được p | q − 1 hoặc p | q2 + q + 1. Đến đây ta sẽ chứng minh p | q − 1
không xẩy ra. Thật vậy. nếu p | q − 1 thì ta được q − 1 = kp, k ∈ N, do đó q = kp + 1. Khi đó từ p2 − p + 1 = q3 ta được
p2 − p + 1 = (kp + 1)3 ⇔ p2 − p + 1 = k3p3 + 3k2p2 + 3kp + 1
Nhận thấy với k > 2 thì hiển nhiên
p2 − p + 1 < k3p3 + 3k2p2 + 3kp + 1
Từ đó suy ra được k 6 1.
1. Với k = 0, khi đó ta được p2 − p + 1 = 1 ⇔ p (p − 1) = 0, điều này vô lí do p là số nguyên tố.
2. Với k = 2, khi đó ta được
p3 − 2p2 + 4p = 0 ⇔ p2 − 2p + 4 = 0
không tồn tại p thỏa mãn.
Vậy với p | q − 1 thì không tồn tai số nguyên tố p thỏa mãn yêu cầu bài toán.
Như vậy ta phải có p | q2 + q + 1, khi đó (p, q − 1) = 1 và có (q − 1) | p (p − 1) nên q − 1 | p − 1. Đặt q2 + q + 1
p = (q − 1) k + 1 với k ∈ N, khi đó từ p | q2 + q + 1 ta suy ra được là số nguyên dương (q − 1) k + 1 hay ta được q2 + qk + k 3k − q − 2 = q + 2 + qk − k + 1 qk − k + 1
là số nguyên dương. Từ đó ta phải có |3k − q − 2| > qk − k + 1. Ta xét các trường hợp sau
• Nếu 3k − q − 2 > qk − k + 1, khi đó ta được k (4 − q) > q + 3. Từ đó nếu q > 4 thì ta được
k (4 − q) < q + 3, điều này mâu thuẫn. Do đó ta suy ra được q < 4, mà ta lại có q > 1 nên q = 2 hoặc q = 3.
1. Khi q = 2 thì từ p2 − p + 1 = q3 ta được p (p − 1) = 7, phương trình vô nghiệm.
2. Khi q = 3 thì từ p2 − p + 1 = q3 ta được p (p − 1) = 26, phương trình vô nghiệm.
• Nếu 3k − k − 2 6 − (qk − k + 1) thì ta suy ra được
2 + q − 3k > qk − k + 1 ⇒ k (q + 2) 6 q + 1 Điều này vô lí.
• Nếu 3k−q−2 = 0 ⇒ q = 3k−2, khi đó từ p = (q − 1) k+1 ta được p = 3k (k − 1)+1 = 3k2−3k+1 và đồng thời có
q2 + q + 1 = (3k − 2)2 + (3k − 2) + 1 = 9k2 − 9k + 3 Từ đó suy ra q2 + q + 1 9k2 − 9k + 3 = = 3 p 3k2 − 3k + 1
Do đó từ p (p − 1) = (q − 1) q2 + q + 1 ta được p − 1 = 3 (q − 1) nên suy ra ra được
(k − 1) (k − 3) = 0. Từ đó ta được k = 1 hoặc k = 3.
1. Với k = 1 thì p = q, khi đó ta được p2 − p + 1 = p3 ⇔ p p2 − p + 1 = 1, điều này vô lí.
2. Với k = 3 thì p = 3q − 2, khi đó ta được 9q2 − 15q + 7 = q3 ⇔ (q − 1) (q − 7) = 0 nên q = 1
hoặc q = 7. Thử trực tiếp ta được q = 7 thỏa mãn yêu cầu bài toán. Khi đó ta được p = 19.
Vậy p = 19 là số nguyên tố duy nhất thỏa mãn yêu cầu bài toán. 6 Chinh phục olympic toán
Ứng dụng định lí Viète trong các bài toán số học
Bài toán 8. Cho x = a + b − c; y = c + a − b; z = b + c − a với a, b, c là các số nguyên tố. Giả sử √ √ rằng x2 = y và z −
y là bình phương của một số nguyên tố. Tìm giá trị của biểu thức T = (a + 2) (b − 10) (c + 2) Lời giải
Biến đổi giả thiết tương đương  2a = x + y  2a = x + x2   2b = x + z ⇒ 2b = x + z  2c = y + z  2c = x2 + z
Xét phương trình bậc 2 là x2 + x = 2a ⇔ x2 + x − 2a = 0. Dễ thấy ∆ = 8a + 1 > 0 nên nên phương
trình có hai nghiệm phân biệt. Gọi hai nghiệm của phương trình là x1 và x2. Như vậy nếu một trong
hai nghiệm là số nguyên thì nghiệm còn lại cũng nguyên. Chú ý rằng a là số nguyên tố nên ta nghĩ
đến sử dụng định lí Viète để xác định các nghiệm. Theo định lí Viète ta có x1 + x2 = −1 x1.x2 = −2a
Do 2 và a là số nguyên tố nên từ x1.x2 = −2a ta được x1 ∈ {−2; −a; 2; a}. Ta xét các trường hợp sau.
• Nếu x1 = −2, khi đó ta tìm được a = 1 không phải là số nguyên tố.
• Nếu x1 = −a, khi đó ta được a2 − 3a = 0, do a là số nguyên tố nên a = 3. Từ đó ta tìm được hai
nghiệm của phương trình là x1 = −3 và x2 = 2.
• Nếu x1 = 2, khi đó ta tìm được a = 3, từ đó ta tìm được hai nghiệm là x1 = 2 và x2 = −3.
• Nếu x = a, khi đó ta được a2 − a = 0 nên a = 0 và a = 1, loại vì không phải là số nguyên tố.
Vậy phương trình trên có hai nghiệm nguyên là x = 2 và x = −3, đồng thời ta có a = 3. Bây giờ ta sẽ
xác định các số nguyên tố b, c ứng với mỗi trường hợp. √
1. Với x = 2 khi đó ta được y = 4. Do đó
z − 2 = p2 với p là số nguyên tố. Do x là số chẵn và
2b = x + z nên z là số chẵn. Khi đó p là số chẵn, dẫn đến p = 2. Khi đó ta được z = 36, suy ra
c = 20, loại do c không phải là số nguyên tố. √
2. Với x = −3, khi đó ta được y = 9. Do đó
z − 3 = p2 với p là số nguyên tố. Do x là số lẻ và
2b = x + z nên z là số chẵn. Khi đó p là số chẵn, dẫn đến p = 2. Khi đó ta được z = 49, suy ra
c = 29 và b = 23 là các số nguyên tố.
Như vậy ta tính được T = (a + 2) (b − 10) (c + 2) = (3 + 2) (23 − 10) (29 + 2) = 2015.
Bài toán 9. Tìm các cặp số nguyên (a; b) sao cho hai số a2 + 4b và b2 + 4a đều là số chính phương. Lời giải
Ta sẽ chứng minh các cặp số sau thỏa mãn yêu cầu bài toán
(a; b) = 0; k2 , k2; 0 , (−4; −4) , (−5; −6) , (−6 − 5) , (k; 1 − k) , (1 − k, k) , k ∈ Z
Thật vậy, do vai trò của a và b như nhau nên không mất tính tổng quát ta có thể giả sử |a| > |b|.
• Nếu b = 0, khi đó để a2 + 4b và b2 + 4a đều là số chính phương thì a = k2 với k là số nguyên.
• Nếu b 6= 0, khi đó biểu thức a2 + 4b là ta liên tưởng đến biệt thức ∆ của phương trình x2 + ax − b = 0. 7 Hướng tới VMO 2020
Tạp chí và tư liệu toán học
Do ∆ = a2 + 4b là số chính phương nên phương trình trên sẽ có hai nghiệm nguyên là x1 và x2. Theo định lí Viète ta được 1 1 1 1 x 1 + x2 a + > + = = > 1 |x 1| |x2| x1 x2 x1x2 b
Từ đó suy ra một trong hai nghiệm nguyên của phương trình trên, chẳng hạn x1 thỏa mãn |x1| 6 2.
Từ đó ta được x1 ∈ {−2; −1; 1; 2}. Đến đây ta xét các trường hợp sau.
1. Nếu x1 = 2, khi đó từ phương trình x2 + ax − b = 0 ta được b = 2a + 4. Suy ra
b2 + 4a = (2a + 4)2 + 4a = 4a2 + 20a + 16 = (2a + 5)2 − 9
là số chính phương. Đặt (2a + 5)2 − 9 = y2 với y ∈ N, từ đó ta được (2a + 5 − y) (2a + 5 + y) = 9.
Lúc này tìm được a = −4 và a = −1.
• Với a = −4, khi đó a = −4. Từ đó ta được (a; b) = (−4; −4) thỏa mãn yêu cầu bài toán.
• Với a = −1, khi đó b = 2. Trường hợp này loại do không thỏa mãn |a| > |b|.
2. Nếu x1 = −2, khi đó từ phương trình x2 + ax − b = 0 ta được b = 4 − 2a. Suy ra
b2 + 4a = (4 − 2a)2 + 4a = 4a2 − 12a + 16 = (2a − 3)2 + 7
là số chính phương. Đặt (2a − 3)2 + 7 = y2 với y ∈ N, từ đó ta được (y − 2a − 3) (y − 2a + 3) = 7.
Giải phương trình trên ta được (a; b) = (3; −2) , (0; 4), trong đó nghiệm (0; 4) bị loại do không
thỏa mãn |a| > |b|. Chú ý là (3; −2) có dạng (k; 1 − k).
3. Nếu x1 = 1, khi đó từ phương trình x2 + ax − b = 0 ta được b = a + 1. Suy ra
b2 + 4a = (a + 1)2 + 4a = a2 + 6a + 1 = (a + 3)2 − 8
là số chính phương. Đặt (a + 3)2 − 8 = y2 với y ∈ N, từ đó ta được (a + 3 − y) (a + 3 + y) = 8.
Giải phương trình trên ta được (a; b) = (−6; −5) , (0; 1), trong đó nghiệm (0; 1) bị loại do không thỏa mãn |a| > |b|.
4. Nếu x1 = −1, khi đó từ phương trình x2 + ax − b = 0 ta được b = 1 − a. Suy ra
b2 + 4a = (1 − a)2 + 4a = a2 + 2a + 1 = (a + 1)2 là số chính phương và
a2 + 4b = a2 + 4 (1 − a) = a2 − 4a + 4 = (a − 2)2
cũng là số chính phương. Do đó (a; b) = (k; 1 − k) với k là số nguyên thỏa mãn yêu cầu bài toán.
Chú ý với (a; b) thỏa mãn yêu cầu bài toán thì (b; a) cũng thỏa mãn yêu cầu bài toán. Do vậy kết hợp
các trường hợp lại ta được các cặp số nguyên (a; b) như trên thỏa mãn yêu cầu bài toán.
Bài toán 10. Cho a, b, c, d là các số thực thỏa mãn a2 − 1 b2 − 1 c2 − 1 d2 − 1 = = = = p 5a 5b 4c 4d
trong đó p là số nguyên dương. Chứng minh rằng (a − c) (b − c) (a + d) (b + d) là một số chính phương. Lời giải 8 Chinh phục olympic toán
Ứng dụng định lí Viète trong các bài toán số học
Biến đổi giả thiết tương đương
a2 + 5pa − 1 = b2 + 5pb − 1 = 0; c2 + 4pc − 1 = d2 + 4pd − 1 = 0
Xét hai phương trình bậc hai ẩn x là x2 + 5px − 1 = 0 và x2 + 4px − 1 = 0. Khi đó ta thấy a, b là hai
nghiệm của phương trình x2 + 5px − 1 = 0 và c, d là hai nghiệm của phương trình x2 + 4px − 1 = 0.
Theo định lý Viète ta được a + b = −5p c + d = −4p và ab = −1 cd = −1
Ta có (a − c) (b − c) (a + d) (b + d) = ab − (a + b) c + c2 ab + (a + b) d + d2. Áp dụng các hệ thức Viète trên ta được
ab − (a + b) c + c2 ab + (a + b) d + d2 = c2 + 5pc − 1 d2 − 9pd − 1
Chú ý rằng c2 + 5pc − 1 d2 − 9pd − 1 = c2 + 4pc − 1 + pc d2 + 4pd − 1 − 9pd, và đồng thời kết
với c2 + 4pc − 1 = d2 + 4pd − 1 = 0 ta được
(a − c) (b − c) (a + d) (b + d) = −9p2cd = 9p2 = (3p)2
Vậy (a − c) (b − c) (a + d) (b + d) là một số chính phương. 4
Phương pháp bước nhảy Viète - Vieta Jumping.
Đây là một phương pháp mạnh để xử lý lớp phương trình Diophantine bậc hai trở lên. Sau đây ta sẽ
tìm hiểu về phương pháp giải của nó.
Phương pháp. Ta tiến hành qua 2 bước sau.
1. Bước 1. Cố định một giá trị nguyên mà đề bài cho, rồi giả sử tồn tại một cặp nghiệm
thỏa mãn một vài điều kiện mà không làm mất tính tổng quát của bài toán.
2. Bước 2. Dựa vào định lý Viète để tìm các mối quan hệ và sự mâu thuẫn, từ đó tìm được kết luận của bài toán.
Một trong các bài toán nổi tiếng nhất để minh họa cho phương pháp này và luôn xuất hiện trong bất
kì các tài liệu nói về vấn đề này, mà mỗi khi nhắc tới học sinh chuyên toán không thể không biết đó
chính là bài toán trong kì thi IMO 1988. ∇
Bài toán 1 [IMO 1988]. Cho a, b là các số nguyên dương thỏa mãn ab + 1 | a2 + b2. Chứng a2 + b2 minh rằng là số chính phương. ab + 1 Lời giải
Bàn luận. Đây chính là bài toán khó nhất kì thi năm đó, và chỉ có mười một học sinh cho lời giải
hoàn chỉnh của bài toán. Trong số 11 học sinh giải được bài toán đó, Việt Nam chúng ta có một đại
diện chính là Giáo sư Ngô Bảo Châu. Sau đây là lời giải cho bài toán này. a2 + b2 Lời giải. Đặt k =
, khi đó theo phương pháp đã đề cập tới ở trên, ta cố định k, sau đó xét tất ab + 1
cả các cặp (a, b) nguyên dương thỏa mãn phương trình a2 + b2 k = ab + 1 a2 + b2
Hay có nghĩa là ta xét tập S = (a, b) ∈ ∗ ∗ N × N | k =
. Vì S là tập các cặp số nguyên dương ab + 1
nên luôn tồn tại một cặp (a0, b0) trong S mà a0 + b0 thỏa mãn a0 > b0 đạt giá trị nhỏ nhất. 9 Hướng tới VMO 2020
Tạp chí và tư liệu toán học Xét phương trình
x2 + b20 = k ⇔ x2 − kx.b0 + b2 − k = 0 xb 0 0 + 1
là một phương trình bậc hai ẩn x. Ta đã biết rằng phương trình trên có một nghiệm là a0. Như vậy
theo định lý Viète thì tồn tại nghiệm a1 thỏa mãn phương trình bậc hai với ẩn x trên và b2 − k a 0 1 = kb0 − a0 = a0
Từ đây ta có a1 cũng là số nguyên. Ta chứng minh a1 không âm. Thật vậy, nếu a1 < 0 thì a2 − − − 1 kb0a1 + b20 k > a21 + k + b20 k > 0
điều này mâu thuẫn. Do đó ta có a1 > 0. Đến đây ta xét a1 > 0 thì (a1, b0) là một cặp thuộc S.
Theo định nghĩa của (a0, b0) ta có a0 + b0 6 a1 + b0 ⇒ a0 6 a1
Mặt khác cũng theo định lí Viète thì a2 − ⇒ 0 6 a0a1 = b2 0 k < b20 a0 < b0
điều này trái với giả thiết ban đầu. Do đó a1 = 0, vì vậy suy ra k = b2 là một số chính phương, ta có 0 điều cần chứng minh.
Nhận xét. Trong bài toán này, ta đã sử dụng tới nguyên lí cực hạn: Trong tập hợp các số nguyên
! dương thì luôn tồn tại số nguyên dương nhỏ nhất. Mệnh đề trên không những hữu dụng trong
các lớp bài toán này mà còn trong nhiều bài toán tổ hợp, tổ hợp số học và số học. Bài toán các
bạn sẽ tìm hiểu sau đây cũng là một kết quả rất nối tiếng.
Bài toán 2 [Phương trình Markov]. Giải phương trình nghiệm nguyên x2 + y2 + z2 = 3xyz. Lời giải
Bàn luận. Đây là một phương trình cực kì nổi tiếng, xuất hiện trong luận án tiến sĩ tại trường Đại học
Saint Petersburg với chủ đề “Dạng toàn phương xác định dương” của nhà toán học Andrei Andreevich
Markov (1856 - 1922) - nhà toán học nổi tiếng người Nga. Luận án tiến sĩ của Markov đã giải quyết
được một số vấn đề khó trong “Lý thuyết số” và mở ra một hướng nghiên cứu trong toán học, đó là “Lý
thuyết xấp xỉ Diophant”. Phương trình Markov - một phương trình Diophant bậc hai đặc biệt đóng vai
trò chủ đạo trong các nghiên cứu của Markov về các dạng toàn phương.
Lời giải. Ta thấy rằng phương trình Markov có một nghiệm (1, 1, 1). Đặt S = {(x, y, z); x, y, z ∈ + Z | x2 + y2 + z2 = 3xyz}
là tập hợp tất cả các nghiệm nguyên dương của phương trình Markov thì S 6= ∅. Do vai trò của x, y, z
trong phương trình là như nhau, không mất tính tổng quát ta có thể giả sử rằng x 6 y 6 z. Với mỗi
cặp (x, y, z) ∈ S; (x0, y0, z0) ∈ S ta định nghĩa (x, y, z) > (x0, y0, z0) nếu x + y + z > x0 + y0 + z0. Markov
đã dùng ý tưởng “thông minh”sau để chứng minh có vô hạn bộ ba số nguyên dương (x, y, z) thỏa mãn
phương trình trên. Với mỗi nghiệm (xn, yn, zn) ∈ S ta xây dựng bộ nghiệm mới như sau: Ta coi xn là
ẩn và các biến còn lại là các tham số thì rõ ràng phương trình bậc hai x2 − 3ynznx + y2n + z2n = 0
có một nghiệm là xn, nên nó có nghiệm thứ hai là x0. Theo định lý Viète, ta có
xn + x0 = 3ynzn và xnx0 = y2n + z2n (1) 10 Chinh phục olympic toán
Ứng dụng định lí Viète trong các bài toán số học
Từ đây ta được x0 là một số nguyên dương, kết hợp với giả thiết xn 6 yn 6 zn và (1) ta được y2 2x2 x0 = n + z2n > n = 2xn > xn. xn xn
Đặt (xn+1, yn+1, zn+1) = (x0, yn, zn) thì (xn+1, yn+1, zn+1) là một nghiệm của phương trình Markov.
Cách xây dựng này cho ta một dãy vô hạn các nghiệm của phương trình Markov vì các nghiệm tiếp
theo lớn hơn các nghiệm trước theo định nghĩa thứ tự ở trên. Do đó phương trình Markov có vô số
nghiệm. Ta thấy ý tưởng của Markov trong chứng minh trên là coi một biến là nghiệm của tam thức
bậc hai khi cố định các nghiệm còn lại để từ đó xây dựng nghiệm mới từ một nghiệm đã biết bằng các
định lí Viète. Cụ thể ta xét phương trình Diophant là phương trình bậc hai đối với một biến nào đó, chẳng hạn x2 + G(x 1
2, x2, . . . , xn) 6= 0 là phương trình bậc 2 ẩn x1. Nếu phương trình này có nghiệm
(x1, x2, . . . , xn) = (a1, a2, . . . , an) thì rõ ràng a1 là nghiệm phương trình X2 + G(a2, a3, . . . , an) = 0.
Phương trình trên phải còn một nghiệm nữa là a0 . Kết hợp với định lý Viète và dữ kiện của đầu bài ta 1 sẽ xây dựng bộ (a0 , a 1
2, . . . , an) là nghiệm của phương trình trên.
Nhận xét. Thông qua 2 bài toán đầu tiên, ta đã phần nào hiểu được ý tưởng của phương pháp
này, bài toán thứ 3 sau đây là bài toán tổng quát của bài toán này, nó sẽ trả lời cho ta câu
! hỏi “Nếu tổng các bình phương S ba số nguyên dương chia hết cho tích P của chúng thì khi đó S thương số bằng bao nhiêu?”. P
Bài toán 3. Tìm tất cả các số nguyên dương k để phương trình x2 + y2 + z2 = kxyz có nghiệm nguyên dương. Lời giải
Lời giải 1. Trước tiên ta thấy rằng với k = 1 phương trình đã cho có nghiệm (3; 3; 3) và với k = 3
thì phương trình có nghiệm (1; 1; 1). Như vậy với k = 1 hoặc k = 3 thì phương trình luôn có nghiệm
nguyên dương. Bây giờ ta cần kiểm tra xem với k 6= 1 và k 6= 3 thì phương trình có nghiệm nguyên
dương không. Giả sử với k 6= 1 và k 6= 3 phương trình đã cho có nghiệm nguyên dương là (x0; y0; z0).
Không mất tính tổng quát ta giả sử x0 6 y0 6 z0 và x0 + y0 + z0 có giá trị bé nhất. Ta xét các trường hợp.
1. Nếu y0 < z0, ta xét phương trình bậc hai z2 − kx0y0z + x2 + y2 = 0. Khi đó phương trình có một 0 0
nghiệm là z0. Theo định lí Viète thì phương trình có một nghiệm nữa là z1. Như vậy thì z0 + z1 = kx0y0 , z0z1 = x2 + y2 0 0 từ đó suy ra x2 + y2 z 0 0 1 = kx0y0 − z0 = z0
Ta thấy z1 nhận giá trị nguyên dương nên (x0; y0; z1) là một nghiệm nguyên dương của phương
trình ban đầu. Từ điều giả sử ta có x0 + y0 + z0 6 x0 + y0 + z1
nên z0 6 z1. Do đó ta được x2 − − 0 + y2 0
kx0y0 = z0z1 − z1 − z0 = (z1 − 1) (z0 − 1) − 1 > y20 1
Suy ra 1 > x0 (ky0 − x0) > x0 (kx0 − x0) > x0. Do x0 là số nguyên dương nên ta được x0 = 1.
Từ đây ta đưa phương trình ban đầu về thành y2 + z2 + 1 = kyz. Đến đây ta cần chỉ ra rằng
phương trình y2 + z2 + 1 = kyz có nghiệm nguyên dương khi và chỉ khi k = 3, tuy nhiên điều này
đơn giản nếu ta sử dụng phương pháp bước nhảy Viète. Do đó điều này mâu thuẫn với k 6= 3. 11 Hướng tới VMO 2020
Tạp chí và tư liệu toán học 2. Nếu y0 = z0 thì ta có 2y2 − 0
kx20y20 + x20 = 0 ⇒ x20 = y20 (kx0 − 2) > x20 (kx0 − 2)
Từ đó dẫn đến 3 > kx0, mà ta lai có kx0 > 2 nên kx0 = 3, suy ra k = 1 hoặc k = 3, điều này trái với k 6= 1 và k 6= 3.
Vậy với k 6= 1 và k 6= 3 thì phương trình đã cho không có nghiệm nguyên dương. Như vậy với k = 1
hoặc k = 3 thì phương trình đã cho có nghiệm nguyên dương.
Lời giải 2. Với x, y, z ∈ +
Z , ta viết phương trình đã cho dưới dạng x2 − kxyz + y2 + z2 = 0. (1)
Giả sử k là số nguyên dương sao cho phương trình (1) có nghiệm nguyên dương. Cố định k và xét tập hợp S = {(x, y, z); x, y, z ∈ + Z | x2 − kxyz + y2 + z2 = 0}.
Theo điều giả sử ở trên thì S 6= ∅, khi đó theo nguyên lý sắp thự tự tốt tồn tại (x0, y0, z0) ∈ S sao
cho x0 + y0 + z0 là nhỏ nhất. Ta thấy rằng, nếu (x0, y0, z0) ∈ S thì các hoán vị của nó cũng thuộc S,
không mất tính tổng quát ta có thể giả sử x0 > y0 > z0. Phương trình
f (x) = x2 − xky0z0 + y20 + z20 = 0
hiển nhiên có một nghiệm x0. Gọi nghiệm còn lại là x1, theo định lý Viète, ta có
x0 + x1 = ky0z0; x0x1 = y20 + z20.
Từ đây, ta được x1 không âm, do đó (x1, y0, z0) ∈ S, theo cách xác định của bộ (x0, y0, z0) thì ta thu
được x1 + y0 + z0 > x0 + y0 + z0 hay x1 > x0. Do đó ta có x1 > x0 > y0 > z0. (2)
Theo định lý về dấu của tam thức bậc hai và từ (2) ta được 0 6 f (y0) 6 y2 − 0
ky20z0 + 2y20 = y20(3 − kz0). Suy ra kz +
0 6 3 ⇒ k 6 kz0 6 3 mà k ∈ Z nên k ∈ {1, 2, 3}.
1. Nếu k = 1, phương trình (1) có nghiệm nguyên dương x = y = z = 3.
2. Nếu k = 2, thì từ kz0 6 3 ta được z0 = 1 khi đó ta có (x0 − y0)2 + 1 = 0, mâu thuẫn.
3. Nếu k = 3, phương trình (1) có nghiệm nguyên dương x = y = z = 1.
Vậy với k ∈ {1, 3} thì phương trình có nghiệm nguyên dương.
Nhận xét. Trong lời giải 1 ta có đề cập tới một kết quả đó là Phương trình y2 + z2 + 1 = kyz có
! nghiệm nguyên dương khi và chỉ khi k = 3, đây là bài toán trong đề thi HSG toán 9 Tỉnh Thanh
Hóa 2015 – 2016. Sau đây ta sẽ cùng xét tới bài toán này.
Bài toán 4. Tìm các số nguyên dương m để phương trình x2 − mxy + y2 + 1 = 0 có nghiệm nguyên dương. Lời giải
Với các nghiệm nguyên dương (x; y) thỏa mãn phương trình, giả sử (x0; y0) là một nghiệm thỏa mãn
x0 + y0 nhỏ nhất. Do vai trò của x và y trong phương trình là như nhau nên không mất tính tổng quát 12 Chinh phục olympic toán
Ứng dụng định lí Viète trong các bài toán số học
ta có thể giả sử x0 6 y0.
Xét phương trình bậc hai có ẩn y là y2 − mx0y + x20 + 1 = 0
La có y0 là một nghiệm của phương trình trên. Ta gọi nghiệm còn lại là y1. Khi đó theo định lí Viète ta có y0 + y1 = mx0 y0.y1 = x2 + 1 0
Dễ dàng nhận thấy y1 có giá trị nguyên và từ cách chọn (x0; y0) ta suy ra được y0 6 y1. Đến đây ta xét các trường hợp sau. 1
1. Nếu x0 = y0 thì từ phương trình ban đầu ta được m = 2 +
. Nên để m và x0 có giá trị nguyên x20 thì x0 = 1 và m = 3.
Với m = 3 ta thấy (x; y) = (1; 1) là một nghiệm nguyên dương của phương trình đã cho.
2. Nếu y0 = y1 thì từ y0.y1 = x2 + 1 hay 0 (y0 − x0) (y0 + x0) = 1 Từ đó ta suy ra được y 0 − x0 = 1 y ⇒ 0 = 1 y0 + x0 = 1 x0 = 0
Trường hợp này loại vì (x0; y0) nguyên dương.
3. Nếu x0 < y0 < y1 khi đó ta được y0 > x0 + 1; y1 > x0 + 2
Kết hợp với y0.y1 = x2 + 1 ta được x2 + 1 + 3x 0 0 > x20
0 + 2 ⇒ 3x0 + 1 6 0, điều này vô lý vì x0 > 0.
Như vậy để phương trình đã cho có nghiệm nguyên dương thì m = 3 và khi đó phương trình có nghiệm
nguyên dương là (x; y) = (1; 1).
Bài toán 5. Tìm tất cả các số nguyên dương k sao cho phương trình x2 + y2 + x + y = kxy có nghiệm nguyên dương. Lời giải
Gọi (x0, y0) là bộ nghiệm nguyên dương của phương trình thỏa mãn x0 + y0 nhỏ nhất. Không mất
tính tổng quát, ta giả sử x0 > y0 > 1. Xét phương trình bậc hai ẩn x
x2 + y20 + x + y0 = kxy0 ⇔ x2 + x(1 − ky0) + y20 + y0 = 0
Phương trình bậc hai này hiển nhiên có một nghiệm x0, gọi nghiệm còn lại là x1. Theo định lí Viète ta có x0 + x1 = ky0 − 1 (1) x0x1 = y2 + y 0 0 (2)
Từ (1) ta có x1 nguyên, từ (2) ta có x1 dương. Như vậy (x1, y0) cũng là một nghiệm thỏa mãn phương
trình, mặt khác, do tính nhỏ nhất của tổng x0 + y0 mà ta có x1 > x0. Do đó từ (1), ta có 2x0 1 ky0 − 1 > 2x0 ⇒ + 6 k y0 y0 Ta có x 0 y0 1 1 x0 1 y0 1 1 k 1 k = + + + = + + + + 6 + 1 + 1 + y0 x0 x0 y0 y0 2y0 x0 x0 2y0 2 2
⇒ k 6 5 ⇒ k ∈ {1; 2; 3; 4; 5} 13 Hướng tới VMO 2020
Tạp chí và tư liệu toán học
1. Với k = 1, ta có x2 + y2 + x + y = xy, phương trình này vô nghiệm nguyên dương vì
x2 + y2 + x + y > 2xy + x + y > xy
2. Với k = 2, tương tự như trên, ta cũng lập luận được phương trình này vô nghiệm nguyên dương.
3. Với k = 3, phương trình có nghiệm nguyên dương (2; 2).
4. Với k = 4 thì phương trình có nghiệm (1; 1).
5. Với k = 5, dấu bằng phải đồng thời xảy ra ở các điểm x 1 k 5 1 1 y + = = , = 1, = 1, = 1 y 2y 2 2 x 2y x
Dễ thấy không tồn tại các số nguyên dương x, y thỏa mãn tất cả các điều trên. Trường hợp này bị loại.
Vậy các giá trị của k thỏa mãn là k ∈ {3; 4}.
Bài toán 6 [IMO 2007]. Cho trước a, b là hai số nguyên dương. Chứng minh rằng nếu 4ab − 1
là ước số của (4a2 − 1)2 thì a = b. Lời giải
Theo giả thiết thì 4ab − 1 | (4a2 − 1)2 nên ta có 4ab − 1 là ước của
b2(4a2 − 1)2 − (4ab − 1)(4a3b − 2ab + a2) = (a − b)2. (a − b)2 Đến đây ta đặt k = thì k ∈ +
Z . Cố định k và xét tập hợp 4ab − 1 S = (a, b); a, b ∈ +
Z ; a 6= b | a2 − 2ab(2k + 1) + b2 + k = 0
Giả sử S 6= ∅, khi đó theo nguyên lý sắp thứ tự tốt tồn tại cặp số (a0, b0) ∈ S sao cho a0 6= b0 và
a0 + b0 nhỏ nhất. Chú ý rằng, nếu (a0, b0) ∈ S thì (b0, a0) ∈ S, do vậy không mất tính tổng quát ta có
thể giả sử a0 > b0. Phương trình T 2 − 2T b0(2k + 1) + b2 + k = 0 có một nghiệm hiển nhiên là a 0 0. Gọi
nghiệm còn lại là a1, theo định lý Viète ta có a0 + a1 = 2b0(2k + 1) (1) a0a1 = b2 + k. 0
Suy ra a1 là số nguyên không âm, do đó (a1, b0) ∈ S, theo cách xác định (a0, b0) thì
a1 + b0 > a0 + b0 ⇔ a1 > a0.
Kết hợp với (1) ta được b2 + k b2 + k a 0 0 0 = 6 ⇔ k > a2 − b2 a 0 0. 1 a0 Như vậy ta suy ra
(a0 − b0)2 = k > a2 − b2 4a 0 0 = (a0 − b0)(a0 + b0). 0b0 − 1
Mặt khác lại do a0 > b0 nên a0 − b0 > 1, vì vậy
a0 − b0 > (a0 + b0)(4a0b0 − 1) > a0 + b0,
điều này là mâu thuẫn, do đó điều giả sử là sai hay S 6= ∅. 14 Chinh phục olympic toán
Ứng dụng định lí Viète trong các bài toán số học a2 + b2
Bài toán 7. Chứng minh rằng nếu a, b là các số nguyên dương sao cho k = là số nguyên ab − 1 thì k = 5. Lời giải
Lời giải 1. Đẳng thức đề bài tương đương với a2 − kab + b2 + k = 0. Không mất tính tổng quát giả sử
a > b. Do a, b là các số nguyên dương và ab 6= 1 nên ta xét a = 2; b = 1 thì được k = 5.
Như vậy ta cần chứng minh là với a + b > 3 thì k = 5, giả sử cặp số dương (a0; b0) có tổng nhỏ nhất
thỏa mãn bài toán. Khi đó ta được a2 − 0 ka0b0 + b20 + k = 0
Xét phương trình bậc 2 ẩn x là a2 − kab + b2 + k = 0
Ta thấy rằng a0 là một nghiệm của phương trình. Như vậy theo định lí Viète thì phương trình trên còn
có nghiệm là a1, khi đó ta có a = kb0 − a0 hay ta có cặp số (kb0 − a0; b0) thỏa mãn yêu cầu bài toán. a0 k a2 + b2
Theo điều giả sử ta có a0 6 kb0 − a0 hay 6 . Từ k = ta suy ra b0 2 ab − 1 a0 b0 k + + = k. b0 a0 a0b0 a0 k k k Do 6 và a0b0 > 3 nên k 6 + 1 +
hay k 6 6. Mặt khác ta lại có b0 2 2 3 a0 b0 + > 2 b0 a0
nên k > 3. Như vậy ta được 3 6 k 6 6.
1. Với a = 3; b = 1 ta tìm được k = 5.
2. Với a = b = 2 hay a = 4; b = 1 thì không tìm được giá trị của k.
3. Với ab > 5, lại dùng đánh giá tương tự như trên ta có k 6 3. Xét k = 3 thì a2 + 3ab + b2 = 3, ta
thấy không có cặp số dương (a; b) thỏa mãn.
Do đó suy ra ab > 6. Thử với a = 6; b = 1 hoặc a = 3; b = 2 đều không thỏa nên ta lại được ab > 7. 14
Lại dùng đánh giá như trên ta được suy ra k 6
, điều này mâu thuẫn với k nguyên và lớn hơn 2. 5
Vậy chỉ có k = 5 thỏa mãn bài toán. a2 + b2 Lời giải 2. Đặt k =
thì k là số nguyên dương. Theo bất đẳng thức AM − GM thì ab − 1 a2 + b2 2ab 2 k = > = 2 + > 2 ab − 1 ab − 1 ab − 1 2
hay k > 3. Nếu a = b thì ta được k = 2 + < 3, mâu thuẫn. a2 − 1
Ta sẽ chứng minh k = 5. Thật vậy, cố định k và xét tập hợp a2 + b2 S = (a, b); a, b ∈ + Z | k = ab − 1
Theo giả thiết S 6= ∅, khi đó theo nguyên lý sắp thứ tự tốt tồn tại cặp số (a0, b0) ∈ S sao cho a0 6= b0
và a0 + b0 nhỏ nhất. Ta thấy rằng nếu (a0, b0) ∈ S thì (b0, a0) ∈ S, không mất tính tổng quát ta có thể
giả sử a0 > b0. Nhận thấy rằng phương trình
T 2 + b20 = k ⇔ T2 − kTb0 + b2 T b 0 + k = 0 0 − 1 15 Hướng tới VMO 2020
Tạp chí và tư liệu toán học
có một nghiệm hiển nhiên là a0. Gọi nghiệm còn lại là a1, theo định lý Viète ta có a0 + a1 = kb0; a0a1 = b20 + k. Từ đây, ta được a +
1 ∈ Z , do đó (a1, b0) ∈ S, theo cách xác định (a0, b0) thì a1 + b0 > a0 + b0 hay
a1 > a0. Vì a0 > b0 nên a0 > b0 + 1, từ đó ta thu được: b2 −
0 + k − kb0 = a0a1 − a0 − a1 = (a0 − 1)(a1 − 1) − 1 > b2 0 1.
Do đó k(b0 − 1) 6 1. Nếu b0 6= 1 theo chứng minh trên thì k(b0 − 1) > 3 > 1, vì vậy ta phải có b0 = 1.
Khi đó a0 + a1 = k và a0a1 = k + 1, suy ra
a0a1 − a0 − a1 − 1 = 0 ⇔ (a0 − 1)(a1 − 1) = 2
mà a1 > a0 nên a0 = 2, a1 = 3, từ đây ta được k = a0 + a1 = 5.
Như vậy, ta có điều phải chứng minh.
Bài toán 8. Tìm tất cả các giá trị k sao cho phương trình (x + y + z)2 = kxyz có nghiệm nguyên dương. Lời giải
Ta gọi k là giá trị cần tìm và (x0; y0; z0) nghiện nguyên dương của phương trình (x + y + z)2 = kxyz
có x0 + y0 + z0 nhỏ nhất. Khi đó không mất tính tổng quát, ta có thể giả sử rằng x0 > y0 > z0. Phương
trình đã cho được viết dưới dạng
x2 − (kyz − 2y − 2z) x + (y + z)2 = 0 Theo định lí Viète ta có (y0 + z0)2
x1 = ky0z0 − 2y0 − 2z0 − x0 = x0
cũng là nghiệm của phương trình trên, suy ra (x1; y0; z0) là nghiệm của phương trình đầu. Ngoài ra ta
cũng suy ra được x1 nguyên dương, hay nói cách khác (x1; y0; z0) là nghiệm nguyên dương của phương
trình đầu. Từ tính nhỏ nhất của x0 + y0 + z0 ta có được x1 > x0, suy ra (y0 + z0)2
ky0z0 − 2y0 − 2z0 − x0 > x0 và > x0 x0
Từ bất đẳng thức thứ 2 ta có y0 + z0 > x0, áp dụng bất đẳng thức thứ nhất ta được ky0z0 > 4x0. Chia 2 vế của
x20 + y20 + z20 + 2x0y0 + 2y0z0 + 2z0x0 = kx0y0z0 cho x0y0z0 ta thu được x0 y0 z0 2 2 2 + + + + + = k y0z0 x0z0 x0y0 z0 x0 y0 k 32 Như vậy ta được
+ 1 + 1 + 2 + 2 + 2 > k hay k 6
, suy ra k 6 10. Chú ý rằng nếu x0 = 1 thì 4 3 k 1
y0 = z0 = 1 suy ra k = 9. Nếu k 6= 9 thì x0 > 2 và đánh giá ở trên trở thành + 1 + + 2 + 1 + 2 > k, 4 2 26
như thế thì ta suy ra được k 6
nên k 6 8. Giá trị k = 10 bị loại. Ta xét các trường hợp sau. 3
1. Với k = 1 phương trình có nghiệm, chẳng hạn (9; 9; 9).
2. Với k = 2 phương trình có nghiệm, chẳng hạn (4; 4; 8). 16 Chinh phục olympic toán
Ứng dụng định lí Viète trong các bài toán số học
3. Với k = 3 phương trình có nghiệm, chẳng hạn (3; 3; 3).
4. Với k = 4 phương trình có nghiệm, chẳng hạn (2; 2; 4).
5. Với k = 5 phương trình có nghiệm, chẳng hạn (1; 4; 5).
6. Với k = 6 phương trình có nghiệm, chẳng hạn (1; 2; 3).
7. Với k = 8 phương trình có nghiệm, chẳng hạn (1; 1; 2).
8. Với k = 9 phương trình có nghiệm, chẳng hạn (1; 1; 1).
Bây giờ ta cần chứng chứng minh được rằng trường hợp k = 7 phương trình không có nghiệm nguyên
dương. Thật vậy, giả sử với k = 7 thì phương trình đã cho có nghiệm nguyên dương (x0; y0; z0) có các
tính chất như trên. Khi đó ta có (x0 + y0 + z0)2 x0 y0 z0 2 2 2 7 = = + + + + + x0y0z0 y0z0 z0x0 x0y0 x0 y0 z0 1 1 y0 + z0 2 2 2 10 < + + + + + 6 y0 z0 y0z0 x0 y0 z0 z0 10 Do đó ta có z0 < nên z0 = 1, khi đó ta có 7 3 3 4 1 1 2 7 < + + ⇒ 1 < + 6 ⇒ y0 < 2 x0 y0 1 x0 y0 y0
Từ đó tại được y0 = 1. Như vậy ta có (x r 0 + y0 + z0)2 (x0 + 2) 4 4 7 = = = + x0 + 4 > 2. .x0 + 4 = 8 x0y0z0 x0 x0 x0
Như vậy khi k = 7 thì phương trình không có nghiệm nguyên dương.
Vậy các giá trị k cần tìm là k ∈ {1; 2; 3; 4; 5; 6; 8; 9}. Bài toán tương tự.
1. Cho các số nguyên dương x, y, z thỏa mãn điều kiện (x + y + z)2 chia hết cho xyz. Tính ! (x + y + z)2 các giá trị của A = . xyz
2. Chứng minh rằng phương trình (x + y + z)2 = 7xyz không có nghiệm nguyên dương.
Bài toán 9. Chứng minh rằng phương trình (x + y + z)2 = 7xyz không có nghiệm nguyên dương. Lời giải
Gọi (x0; y0; z0) là một nghiệm thỏa mãn phương trình với z0 là số nhỏ nhất. Không mất tính tổng quát,
ta giả sử x0 6 y0 6 z0, khi đó ta có
z0 | (x0 + y0 + z0)2 ⇒ z | (x0 + y0)2 + 2z0(x0 + y0) + z02 ⇒ z0 | (x0 + y0)2
Ta xét phương trình bậc hai ẩn z là z2 − (7x0y0 − 2x0 − 2y0)z + (x0 + y0)2 = 0, hiển nhiên phương trình (x0 + y0)2
này có một nghiệm z0, nên theo định lí Viète thì nghiệm còn lại của nó là ∈ Z. z0 (x0 + y0)2 Như vậy x0; y0;
cũng là một bộ số thỏa mãn phương trình. Nếu giả sử z0 (x0 + y0)2 x0 + y0 < z0 ⇒ < z0 z0 17 Hướng tới VMO 2020
Tạp chí và tư liệu toán học
thì vô lí vì (x0; y0; z0) cũng là một bộ số thỏa mãn phương trình và vì tính nhỏ nhất của z0. Do đó phải
có z0 6 x0 + y0. Khai triển phương trình ban đầu và chia hai vế của nó cho x0y0z0 ta được x0 y0 z0 2 2 2 1 1 x0 + y0 2 2 2 7 6 + + + + + 6 + + + + + y0z0 z0x0 x0y0 x0 y0 z0 z0 x0 x0y0 x0 y0 z0 4 3 3 10 10 = + + 6 ⇒ x0 6 ⇒ x0 = 1 x0 y0 z0 x0 7
Khi đó ta được y0 6 z0 6 y0 + 1 ⇒ z0 = y ∨ z0 = y0 + 1. Xét các trường hợp
1. Nếu z0 = y0 thì ta có phương trình √ 2 ± 7
(1 + 2z0)2 = 7z02 ⇔ 3z02 − 4z0 − 1 = 0 ⇔ z0 = 3 trường hợp này loại.
2. Nếu z0 = y0 + 1 thì ta có phương trình 4
(2 + 2z0)2 = 7z0(z0 + 1) ⇔ 3z02 − z0 − 4 = 0 ⇔ z ∈ −1; 3
Như vậy không tồn tại nghiệm nguyên dương của phương trình đã cho.
Bài toán 10 [VMO 2012]. Xét các số tự nhiên lẻ a, b thỏa mãn a | b2 + 2 và b | a2 + 2. Chứng
minh rằng a, b là các số hạng của dãy số tự nhiên (vn) được xác định bởi công thức v1 = v2 = 1
vn = 4vn−1 − vn−2, ∀n > 2 Lời giải
Đầu tiên ta sẽ đi chứng minh rằng b | a2 + 2 ∧ a | b2 + 2 ⇔ ab | a2 + b2 + 2. Thật vậy, ta có
b | a2 + 2 ∧ a | b2 + 2 ⇒ ab | a2 + 2 b2 + 2 ⇒ ab | 2 a2 + b2 + 2
Do a, b lẻ nên ab | a2 + b2 + 2. Ngược lại nếu có ab | a2 + b2 + 2 thì dễ dàng suy ra ngay được b | a2 + 2
và a | b2 + 2. Từ đó giả thiết đề bài tương đương với việc tồn tại số nguyên dương k sao cho a2 + b2 + 2 = kab.
Sử dụng phương pháp bước nhảy Viète ta sẽ đi chứng minh k = 4. Cố định k và xét tập a2 + b2 + 2 S = a, b ∈ +2 ∗ Z | k = ∈ N ab
Trong S ta chọn ra cặp (A, B) sao cho tổng A + B là nhỏ nhất. Không mất tính tổng quát, ta giả
sử A > B. Xét phương trình bậc hai ẩn t, t2 − ktB + B2 + 2 = 0. Dễ thấy phương trình này có một
nghiệm là A, gọi nghiệm còn lại là t0. Theo định lí Viète ta có t0 + A = kB t0A = B2 + 2
Từ đây suy ra được t0 nguyên dương. Chú ý vì A + B là nhỏ nhất nên ta được t0 > A. A k Suy ra t0 + A = kB > 2A hay 6 . B 2
Nếu có một trong hai số a, b bằng 1, giả sử b = 1 thì ka = a2 + 3, dễ suy ra k = 4.
Nếu cả hai số a, b > 2. Ta có A > B > 2. Thì A B 2 k 2 k = + + 6 + 1 + ⇔ k 6 3 B A AB 2 2.2 18 Chinh phục olympic toán
Ứng dụng định lí Viète trong các bài toán số học
Theo bất đẳng thức AM − GM ta có
kab = a2 + b2 + 2 > 2(ab + 1) ⇒ k > 3
lúc này ta được k = 3. Khi đó A2 + B2 + 2 = 3AB. Từ đẳng thức này dễ dàng suy ra phải có một
trong hai số chia hết cho 3, giả sử 3 | A thì A > 3. Nếu B = 1 ta gặp mâu thuẫn, do đó B > 2. Tức AB > 6. Tuy nhiên A B 2 3 2 3 = + + 6 + 1 + B A AB 2 6
Điều này vô lí. Vậy k = 4 là giá trị duy nhất cần tìm. Ta chứng minh xong việc các số a, b thỏa giả
thiết thì cũng phải thỏa mãn phương trình a2 + b2 + 2 = 4ab (1)
Bài toán sẽ hoàn tất nếu ta chỉ rằng nếu cặp (a, b) bất kỳ thỏa mãn (1) thì sẽ luôn tồn tại số tự nhiên
n sao cho a = xn, b = xn+1. Giả sử (u0, u1) là một cặp số nguyên dương bất kỳ thỏa (1). Ta hoàn
toàn có quyền giả sử u0 > u1. Nếu u0 = 1 thì u1 = 1, tức tồn tại n = 1 để u0 = v1, u1 = v2. Tương tự
khi xét u1 = 1. Do đó ta chỉ cần xét u0, u1 > 1. Khi đó ta chọn cặp (u1, u2) = (u1, 4u1 − u0), dễ thấy
u1, u2 nguyên dương và (u1, u2) cũng thỏa mãn (1). Lúc này ta chú ý 4u1 − u0 < u0 vì u2 + 2 2 − (u0 − u1)(u0 + u1) 4u 1 1 − u0 = − u0 = 6 0. u0 u0
Suy ra u1 + u2 = u1 + (4u1 − u0) < u1 + u0. Tương tự ta cũng chọn được cặp (u2, u3) = (u2, 4u2 − u1)
cũng thỏa u2, u3 nguyên dương, cũng thỏa (1) và u2 + u3 < u1 + u2 < u1 + u0. Cứ tiếp tục quá trình này, ta được
... < ui + ui+1 < ... < u1 + u2 < u1 + u0
Thế nhưng u1 + u0 > 2 nên phải tồn tại k sao cho uk + uk+1 = 2, suy ra uk = uk+1 = 1. Tức là ta có
uk = v2, uk+1 = v1. Ta có thể thấy được cách xác định un là như sau
un+2 = 4un+1 − un hay un = 4un+1 − un+2. Từ đó ta có
uk−1 = 4uk − uk+1 = 4v2 − v1 = v3
uk−2 = 4uk−1 − uk = 4v3 − v2 = v4 ...
u1 = 4u2 − u3 = 4vk − vk−1 = vk+1
Như vậy tồn tại n = k + 1 để với cặp (u0, u1) bất kỳ thỏa (1) thì ta có (u1, u0) = (vk+1, vk+2).
Bài toán tương tự [Canada MO 1998]. Cho m là một số nguyên dương. Dãy số (un) với
n > 0 được xác định như sau u0 = 0, u1 = m ! un+1 = m2un − un−1
với mọi n > 1. Chứng minh rằng, các cặp số (a, b) với a, b ∈ +
Z , a > b, là nghiệm của phương a2 + b2 trình
= m2 khi và chỉ khi (a, b) = (un, un+1) với mọi số tự nhiên n. ab + 1
Bài toán 11 [Vietnam TST 1992]. Tìm tất cả các nghiệm nguyên dương (x, y) của phương trình x2 − 5xy + y2 + 5 = 0. (1) Lời giải 19 Hướng tới VMO 2020
Tạp chí và tư liệu toán học
Lời giải và bình luận bài toán này chúng tôi xin được trích từ chuyên đề bước nhảy Viète của thầy Hà
Tuấn Dũng - Khoa Toán - ĐH Sư Phạm Hà Nội 2.
Đầu tiên, chúng ta chứng minh bổ đề.
Bổ đề. Xét hai dãy số (un) và (vn) được xác định như sau
u0 = 1, u1 = 2, un+2 = 5un+1 − un, ∀n = 0, 1, . . . ;
v0 = 1, v1 = 3, vn+2 = 5vn+1 − vn, ∀n = 0, 1, . . .
Khi đó, với mọi n ∈ N các cặp số (un, un+1) và (vn, vn+1) là nghiệm nguyên dương của (1).
Chứng minh. Ta sẽ chứng minh mệnh đề sau bằng phương pháp quy nạp toán học.
Với n = 0, ta có u2 + u2 − 5u 1 0
0u1 = −5. Do đó (u0, u1) là nghiệm của phương trình (1). Như vậy, mệnh đề đúng với n = 0.
Giả sử mệnh đề đúng với n = k > 0, tức là u2 − k + u2 k+1 5ukuk+1 + 5 = 0. Khi đó u2 − − k+1 + u2 k+2
5uk+1uk+2 = uk+2(uk+2 − 5uk+1) + u2k+1 = u2k+1 + u2k 5ukuk+1.
Từ giả thiết quy nạp, ta được u2 − k+1 + u2 k+2 5uk+1uk+2 + 5 = 0.
Do đó (uk+1, uk+2) cũng là nghiệm của phương trình (1). Theo nguyên lý quy nạp toán học thì mệnh
đề đúng với mọi n ∈ N. Chứng minh tương tự, ta cũng thu được với mọi n ∈ N thì (vn, vn+1) là nghiệm
của phương trình (1). Từ công thức xác định số hạng tổng quát của hai dãy số (un) và (vn) ta được
các số hạng của hai dãy đều là các số nguyên dương. Do đó, các cặp số (un, un+1) và (vn, vn+1) là
nghiệm nguyên dương của phương trình (1). Như vậy bổ đề được chứng minh. ∇
Quay lại bài toán. Xét tập hợp S = {(a, b); a, b ∈ + Z | a2 − 5ab + b2 + 5 = 0}.
Với (a, b) ∈ S nếu a = b thì ta có 5 3a2 − 5 = 0 ⇒ a2 = , 3
điều này mâu thuẫn. Do đó a 6= b. Ta thấy rằng (a, b) ∈ S thì (b, a) ∈ S, không giảm tính tổng quát ta
có thể giả sử rằng với mọi (a, b) ∈ S thì a < b. Với (a, b) là một phần tử bất kì thuộc S. Xét dãy số
(an) được xác định như sau
a0 = b, a1 = a, an+2 = 5an+1 − an, ∀n ∈ N.
Ta có b(5a − b) = a2 + 5 > 0 ⇒ 5a > b. Từ công thức xác định số hạng tổng quát của dãy số (an) ta được a + n ∈ Z
với mọi n ∈ N. Ta có (a0, a1) = (a, b) ∈ S, giả sử (ak, ak+1) ∈ S với mọi k > 1, khi đó a2 − − k+1 + a2 k+2
5ak+1ak+2 = ak+2(ak+2 − 5ak+1) + a2k+1 = a2k+1 + a2k 5akak+1.
Từ đây ta được (ak+1, ak+2) ∈ S, theo nguyên lý quy nạp toán học thì (an, an+1) ∈ S, ∀n ∈ N. Nếu
a = 1 thì từ (1) ta được b2 − 5b + 6 = 0 ⇔ b ∈ {2, 3}..
1. Nếu b = 2 ta có (a, b) = (u0, u1).
2. Nếu b = 3 thì (a, b) = (v0, v1).
Ta xét trường hợp a > 1, khi đó (4a − b)(a − b) = 3a2 − 5 > 0, mà a < b nên 4a < b, mà a0 > a1 nên từ
đây ta được an > an+1 với mọi n ∈ N. Như vậy với a > 1 thì dãy (an) là một dãy giảm ngặt, nên phải
tồn tại một chỉ số k sao cho a0 > a1 > · · · > ak+1 = 1. Do (ak, ak+1) là một nghiệm của phương trình
(1) nên ta có ak ∈ {2, 3}. Với ak = 2 thì ta có ak+1 = u0, ak = u1, khi đó ak−1 = 5ak − ak+1 = u2, từ
đó ta được ai = uk+1−i. Tương tự với ak = 3 thì (a, b) là các số hạng liên tiếp trên dãy (vn). Như vậy, 20 Chinh phục olympic toán
Ứng dụng định lí Viète trong các bài toán số học
các bộ (un, un+1) và (vn, vn+1) (với mọi n ∈ N) là tập tất cả các nghiệm nguyên dương của phương trình (1).
Nhận xét. Ta thiết lập quan hệ thứ tự trên S như sau nếu (x, y) ∈ S, (x0, y0) ∈ S thì
(x, y) > (x0, y0) ⇔ x > x0 và y > y0
Từ một nghiệm bất kì của phương trình (1) bằng phương pháp bước nhảy Viète ta thiết lập được
mới nhỏ hơn nghiệm (a, b) theo quan hệ thứ tự nói trên. Từ nghiệm mới vừa thu được này ta lại
xây dựng nghiệm mới nhỏ hơn, cứ tiếp tục quá trình như vậy đến khi không thể xây dựng được
nữa. Khi đó, ta thu được nghiệm nhỏ nhất. Dãy (an) đã mô tả các nghiệm của phương trình (1)
được xây dựng từ quá trình trên và được xây dựng dựa vào các tính chất: a, b là hai số hạng đầu
tiên của dãy: (ai, ai+1) là một nghiệm của phương trình (1). Để xác định được công thức truy
! hồi của dãy (an) ta đã sử dụng phương pháp bước nhảy Viète. Xét phương trình
T 2 − 5T an+1 + a2n+1 + 5 = 0
có một nghiệm là an, gọi nghiệm còn lại là an+2 thì theo hệ thức Viète ta có an + an+2 = 5an+1 (2) anan+2 = a2 + 5 n+1
Từ đây, ta có an+2 là số nguyên dương, do đó (an, an+2) cũng là một nghiệm của phương trình,
và từ (2) ta được an+2 = 5an+1 − an. Sau khi thu được nghiệm nhỏ nhất, ta xây dựng các nghiệm
của phương trình từ nghiệm nhỏ nhất đó thông qua hai dãy (un) và (vn).
Bài toán 12 [VMO 2002]. Tìm tất cả các giá trị nguyên dương k sao cho phương trình (x + y + z + t)2 = k2xyzt có nghiệm nguyên dương. Lời giải
Biến đổi phương trình ban đầu về dạng
x2 + (2y + 2z + 2t − k2yzt)x + (y + z + t)2 = 0
Trong các nghiệm nguyên dương của phương trình, ta chọn ra bộ nghiệm (x0, y0, z0, t0) có tổng
x0 + y0 + z0 + t0 nhỏ nhất. Khi đó dễ thấy x0 là một nghiệm của phương trình bậc hai
x2 + (2y0 + 2z0 + 2t0 − k2y0z0t0)x + (y0 + z0 + t0)2 = 0 (*)
Gọi nghiệm còn lại của (∗) là x1, theo định lí Viète ta có
x0 + x1 = k2y0z0t0 − 2y0 − 2z0 − 2t0 (1) x0.x1 = (y0 + z0 + t0)2 (2)
Từ (1) ta có x1 nguyên và từ (2) ta có x1 dương. Như vậy (x1, y0, z0, t0) cũng là một bộ số thỏa (∗),
nhưng vì tính nhỏ nhất của tổng x0 + y0 + z0 + t0 mà ta có x1 > x0. Do đó từ (2) ta suy ra (y0 + z0 + t0)2 x1 =
> x0 ⇒ y0 + z0 + t0 > x0 x0
Kết hợp với (1) ta được
k2y0z0t0 − 2y0 − 2z0 − 2t0 − x0 > x0 ⇒ k2y0z0t0 > 2x0 + 2(y0 + z0 + t0) > 4x0. 21 Hướng tới VMO 2020
Tạp chí và tư liệu toán học
Chia hai vế của đẳng thức x2 + y2 + z2 + t2 + 2x 0 0 0 0
0y0 + 2x0z0 + 2x0t0 + 2y0z0 + 2y0t0 + 2z0t0 = k2x0y0z0t0 cho x0y0z0t0, ta được x0 y0 z0 t0 2 2 2 2 2 2 + + + + + + + + + = k2 y0z0t0 x0z0t0 x0y0t0 x0y0z0 z0t0 y0t0 y0z0 x0t0 x0z0 x0y0
Không mất tính tổng quát ta giả sử rằng x0 > y0 > z0 > t0 > 1. Khi đó suy ra x0 k2 y0 1 z0 1 t0 1 6 , 6 6 1 , 6 6 1, 6 6 1 y0z0t0 4 z0t0x0 z0t0 x0y0t0 x0t0 x0y0z0 x0y0 Như vậy ta được k2 k2 k2 6
+ 1 + 1 + 1 + 2 + 2 + 2 + 2 + 2 + 2 =
+ 15 ⇒ k2 6 20 ⇒ k ∈ {1, 2, 3, 4} 4 4
1. Nếu k = 1, phương trình có nghiệm (4, 4, 4, 4).
2. Nếu k = 2, phương trình có nghiệm (2, 2, 2, 2).
3. Nếu k = 3, phương trình có nghiệm (1, 1, 2, 2).
4. Nếu k = 4, phương trình có nghiệm (1, 1, 1, 1).
Như vậy để phương trình có nghiệm nguyên dương thì tập hợp tất cả các giá trị nguyên dương của k là k ∈ {1, 2, 3, 4}. a2
Bài toán 13 [IMO 2003]. Hãy tìm tất cả các cặp số nguyên dương (a; b) sao cho 2ab2 − b3 + 1
là một số nguyên dương. Lời giải a2
Giả sử tồn tại cặp số nguyên dương (a, b) thỏa mãn điều kiện bài toán. Đặt k = thì k là 2ab2 − b3 + 1
một số nguyên dương. Cố định k và xét tập hợp S = {(a, b); a, b ∈ + Z
| a2 − 2akb2 + k(b3 − 1) = 0}. Như vậy ta ta có S 6= + ∅. Do k ∈ Z
nên với (a, b) ∈ S ta có 2ab2 − b3 + 1 > 0 suy ra
b2(2a − b) > −1 ⇒ b2(2a − b) > 0.
Do đó 2a = b hoặc 2a > b. Nếu 2a > b thì do k > 1 nên ta được
a2 > 2ab2 − b3 + 1 > b2(2a − b) > b2.
Từ đó suy ra nếu (a, b) ∈ S thì 2a = b hoặc a > b. Gọi (a0, b0) là một phần tử bất kì thuộc S. Xét
phương trình T 2 − 2T kb2 + k(b3 − 1) = 0 là phương trình bậc hai ẩn T có một nghiệm là a 0 0 0. Gọi
nghiệm còn lại a1, theo định lí Viète ta có a0 + a1 = 2kb20 (1) a0a1 = k b3 − 1 0
Như vậy ta được a1 ∈ Z và a1 > 0. Nếu a1 = 0, thì từ (1) ta có b0 = 1 và a0 = 2k, như thế thì (2k, 1)
là một cặp số thỏa mãn điều kiện bài toán. Nếu a + 1 ∈ Z
thì (a1, b0) ∈ S. Không giảm tính tổng quát
ta có thể giả sử a1 > a0. Chú ý rằng, theo nhận xét ở trên thì 2a0 = b0 hoặc a0 > b0. Nếu a0 > b0 thì
ta có ngay a1 > a0 > b0, kết hợp với (1) ta thu được k(b3 − 1) k(b3 − 1) kb2 0 0 0 6 a1 = 6 < kb2 a 0. 0 b0
Điều này mâu thuẫn. Với 2a0 = b0 thì ta được (k, 2k) là một cặp số thỏa mãn điều kiện bài toán. Từ
hệ thức a0a1 = k(b3 − 1) ta thu được (8k3 − 1, 2k) là một cặp số cần tìm. Vậy các cặp số (a, b) thỏa 0
mãn điều kiện bài toán (2k, 1), (k, 2k) và (8k3 − 1, 2k) với k là số nguyên không âm. 22 Chinh phục olympic toán
Ứng dụng định lí Viète trong các bài toán số học
Bài toán 14. Cho phương trình x2 + y2 + z2 + t2 − N xyzt − N = 0 trong đó N là một số nguyên dương cho trước.
a) Chứng tỏ rằng, có vô số giá trị nguyên dương N để phương trình trên có nghiệm nguyên
dương (nghĩa là mỗi nghiệm gồm 4 số nguyên dương x, y, z, t).
b) Cho N = 4k(8m + 7) với k, m là các số nguyên không âm. Chứng minh rằng, khi đó phương
trình trên không có nghiệm nguyên dương. Lời giải
a) Biến đổi phương trình tương đương
x2 + y2 + z2 + t2 − N xyzt − N = 0 (1)
⇔ t(t − N xyzt) = N − x2 + y2 + z2 (2)
Với ba số nguyên dương bất kỳ a, b, c và N = a2 + b2 + c2 thì dễ thấy phương trình (2) có nghiệm
x0 = a, y0 = b, z0 = c, t0 = N abc = a2 + b2 + c2 abc (*)
Chú ý rằng khi hoán vị bốn số a, b, c, N abc ta lại được nghiệm (x1, y1, z1, t1) của phương trình (1).
b) Giả sử phương trình (1) có nghiệm nguyên dương, chọn (x0, y0, z0, t0) là nghiệm nguyên dương của
(1) sao cho tổng x0 + y0 + z0 + t0 là số nguyên dương nhỏ nhất. Không làm mất tính chất tổng quát,
giả sử rằng x0 6 y0 6 z0 6 t0. Ta sẽ chứng minh rằng với N > 7 thì nghiệm nguyên dương của phương
trình (1) với x0 6 y0 6 z0 6 t0 nếu có phải có dạng (∗) như trên.
Theo giả thiết t0 là nghiệm của phương trình bậc hai t2 − N x0y0z0t + x2 − 0 + y2 0 + z2 0 N = 0 (3)
Phương trình (3) có nghiệm thứ hai t1 thoả mãn t1 + t0 = N (x0y0z0) (4) t1 · t0 = x2 + y2 + z2 − N (5) 0 0 0
Từ (4) suy ra t1 ∈ Z. Lại theo giả thiết ta có
N (1 + x0y0z0t1) = t21 + x20 + y20 + z20 > 0 1 nên ta được t1 > −
, vì t1 ∈ Z nên t1 > 0. Giả sử t1 > 0 khi đó (x0, y0, z0, t1) là nghiệm nguyên x0y0z0
dương của (1). Do cách chọn (x0, y0, z0, t0) thì
x0 + y0 + z0 + t1 > x0 + y0 + z0 + t0 ⇒ t1 > t0 Từ đó theo (5) ta có t2 − 0 6 t1t0 = x2 0 + y2 0 + z2 0 N < x20 + y20 + z20 6 3z20 Ta có N 1 + x 0y0z2 0
6 N (1 + x0y0z0t0) = x20 + y20 + z20 + t20 6 z20 + z20 + z20 + 3z20 = 6z20
Từ đó, do N > 7, nên ta suy ra được N 1 + x 0y0z2 ⇒ 0 6 6z20 < N z20 1 + x0y0z20 < z20
Điều vô lý này chứng tỏ t1 > 0 là sai, suy ra t1 = 0. Từ (4), (5) suy ra N = x2 + y2 + z2 0 0 0 t0 = N x0y0z0
là nghiệm (∗) của phương trình (1). Với N = 4k(8m + 7) > 7, áp dụng kết quả trên thì N = x2 + y2 + z2.
Do đó nếu chứng minh được phương trình x2 + y2 + z2 = 4k(8m + 7) không có nghiệm nguyên dương
thì phương trình (1) cũng không có nghiệm nguyên dương. Ta xét các trường hợp sau. 23 Hướng tới VMO 2020
Tạp chí và tư liệu toán học
1. Khi k = 0 ta có x2 + y2 + z2 = 8m + 7 hay x2 + y2 + z2 ≡ 7(mod8). Trong ba số x, y, z phải có
một số lẻ hoặc cả ba số lẻ. Nếu số a lẻ thì a2 ≡ 1 (mod 8), do đó x2 + y2 + z2 6= 7(mod8). 2. Khi k > 0 ta có x2 + y2 + z2 = 4k(8m + 7) (**)
hay x2 + y2 + z2 ≡ 0(mod4). Trong ba số x, y, z phải có một số chẵn hoặc ba số chẵn. Nếu có
một số chẵn, còn hai số a, b lẻ thì a2 + b2 ≡ 2( mod 4), suy ra x2 + y2 + z2 6= 7( mod 8). Nếu x, y, z
đều chẵn, đặt x = 2x1, y = 2y1, z = 2z1 thì (∗∗) tương đương với x2 + y2 + z2 = 4k−1(8m + 7)
Sau k lần biến đổi như thế ta có x2 + y2 + z2 = 8m + 7, nhưng phương trình này vô nghiệm
nguyên dương như khi xét k = 0.
Bài toán được giải quyết. 5 Các bài toán tổng hợp. 5.1 Đề bài a2 + ab + b2
Câu 1. Cho a, b, k là các số nguyên dương thỏa mãn k =
. Chứng minh rằng k là một số ab + 1 chính phương. ab(5a2 + 5b2 − 2)
Câu 2. Cho các số nguyên dương a, b thỏa mãn
∈ Z. Chứng minh rằng a = b. 5ab − 1 x2 + y2 + 30
Câu 3. Cho các số nguyên dương x, y, A thỏa mãn hệ thức A = . Chứng minh rằng A là xy
lũy thừa bậc năm của một số nguyên. x2 + y2 + z2
Câu 4. Cho các số nguyên dương x, y, z thỏa mãn
nhận giá trị nguyên dương. Chứng xyz + 1 x2 + y2 + z2 minh rằng
có thể biểu diễn được thành tổng của hai số chính phương. xyz + 1
Câu 5. Cho a, b, c là các số nguyên dương thỏa mãn 0 < a2 +b2 −abc 6 c. Chứng minh rằng a2 +b2 −abc là số chính phương.
Câu 6. Cho m > n là các số nguyên dương lẻ và n2 − 1 chia hết cho m2 − n2 + 1. Chứng minh rằng
m2 − n2 + 1 là một số chính phương.
Câu 7. Cho x và y là các số nguyên dương thỏa mãn điều kiện x2 + y2 + 1 chia hết cho 2xy + 1. Chứng minh rằng x = y.
Câu 8. Cho a, b là các số nguyên dương lẻ thỏa mãn a2 + 2 chia hết cho b và b2 + 2 chia hết cho a. a2 + b2 + 2 Chứng minh rằng là số chính phương. ab
Câu 9. Cho các số nguyên dương a, b, c, d thỏa mãn b2 + 1 = ac và c2 + 1 = bd. Chứng minh rằng a + c = 3b và b + d = 3c.
Câu 10. Giả sử phương trình x2 + y2 + x + y + 1 = xyz có nghiệm nguyên dương. Tìm tất cả các giá trị của z.
Câu 11. Tìm các số nguyên dương x và y sao cho x + 1 chia hết cho y và y + 1 chia hết cho x.
Câu 12. Tìm các số nguyên dương x, y để x2 + 2 chia hết cho xy + 1.
Câu 13. Tìm tất cả các số có ba chữ số chia hết cho 11 sao cho thương số của phép chia số đó cho 11
bằng tổng bình phương của các chữ số của số đó.
Câu 14 [Kiran Kedlaya]. Cho các số nguyên dương a, b, c là thỏa mãn (ab + 1) (bc + 1) (ca + 1) là
số chính phương. Chứng minh rằng ba số ab + 1; bc + 1; ca + 1 đều là số chính phương.
Câu 15. Tồn tại hay không năm số nguyên dương a1; a2; a3; a4; a5 thỏa mãn hệ điều kiện  a2 + 1 = (a1 + 1) (a3 + 1)  2 a2 + 1 = (a 3 2 + 1) (a4 + 1)  a2 + 1 = (a 4 3 + 1) (a5 + 1) 24 Chinh phục olympic toán
Ứng dụng định lí Viète trong các bài toán số học 5.2
Hướng dẫn giải - Lời giải a2 + ab + b2
Câu 1. Cho a, b, k là các số nguyên dương thỏa mãn k = . Chứng minh rằng k là ab + 1 một số chính phương.
Lời giải. Trước tiên ta sẽ cố định k và xét tập a2 + ab + b2 S = (a, b) ∈ N × N | k = ab + 1
Giả sử phản chứng k không là số chính phương, khi đó trong các phần tử của S ta chọn ra cặp (A, B)
thỏa mãn điều kiện A + B là nhỏ nhất. Không mất tính tổng quát, ta giả sử rằng A > B > 0. Xét
phương trình bậc hai ẩn x x2 + xB + B2 k =
⇔ x2 + (B − kB)x + B2 − k = 0 xB + 1
Phương trình này hiển nhiên có hai nghiệm là A và x0, khi đó theo định lí Viète ta có x0 + A = kB − B (1) x0.A = B2 − k (2)
Từ (1) ta suy ra được x0 là số nguyên.
1. Nếu x0 < 0 thì x0 6 −1 ⇒ x2 − (Bk − B)x + B2 − k > x2 + (Bk − B) + B2 − k > 0, điều này là mâu thuẫn.
2. Nếu x0 = 0 thì k = B2 là một số chính phương, trường hợp này loại.
3. Nếu x0 > 0 thì (x0, B) ∈ S. Như vậy ta được B2 − k B2 A2 x0 + B = + B < + B < + B = A + B A A A
điều này mâu thuẫn với tính nhỏ nhất của tổng A + B. Do đó giả thiết phản chứng là sai, từ đó ta có
k phải là một số chính phương. ab(5a2 + 5b2 − 2)
Câu 2. Cho các số nguyên dương a, b thỏa mãn
∈ Z. Chứng minh rằng a = b. 5ab − 1 5a2 + 5b2 − 2 5a2 + 5b2 − 2
Lời giải. Vì gcd(ab, 5ab − 1) = 1 nên ta có ∈ Z. Đặt = k ∈ Z, ta dễ dàng 5ab − 1 5ab − 1 có được 5a2 + 5b2 − 2
5(a2 + b2) − 2 > 10ab − 2 > 5ab − 1 ⇒ k = > 1 ⇒ k > 2. 5ab − 1 5a2 + 5b2 − 2 Xét tập S = (a, b) ∈ + + Z × Z | k =
∈ Z . Cố định k và trong các phần tử của S, ta 5ab − 1
chọn ra cặp số (A, B) nguyên dương thỏa mãn tổng A + B nhỏ nhất. Gỉa sử A 6= B, không mất tính
tổng quát, xét A > B. Xét phương trình bậc hai ẩn x
5x2 + 5B2 − 2 = k ⇔ 5x2 − 5xBk + 5B2 + k − 2 = 0 5xB − 1
Dễ thấy phương trình này có một nghiệm là A, gọi nghiệm còn lại là x0. Theo định lí Viète, ta có  A + x  0 = Bk (1) 5B2 + k − 2 Ax (2)  0 = 5 Từ (1) ta có x0 nguyên. 25 Hướng tới VMO 2020
Tạp chí và tư liệu toán học
1. Nếu x0 < 0 thì x0 6 −1 ⇒ 5x2 − 5x 0
0Bk + 5B2 + k − 2 > 5 + 5Bk + 5B2 + k − 2 > 0, điều này là mâu thuẫn.
2. Nếu x0 > 0 thì (x0, B) ∈ S. Khi đó do tính nhỏ nhất của tổng A + B mà ta có 5B2 + k − 2 5A2 + 5B2 − 2 x0 > A ⇒ > A ⇔ − 2 > 5(A − B)(A + B) 5A 5AB − 1 5(A − B)2 ⇔ > 5(A − B)(A + B) 5AB − 1
⇔ A − B > (A + B)(5AB − 1).
Rõ ràng điều này vô lí.
Như vậy phải có x0 = 0, suy ra 5B2 = 2 − k > 0, lại có k > 2, do đó k = 2. Suy ra
5a2 + 5b2 − 2 = 2 ⇔ (a − b)2 = 0 ⇔ a = b 5ab − 1
Bài toán được giải quyết. x2 + y2 + 30
Câu 3. Cho các số nguyên dương x, y, A thỏa mãn hệ thức A = . Chứng minh rằng xy
A là lũy thừa bậc năm của một số nguyên.
Lời giải. Gọi (x0; y0) là cặp số thỏa mãn đề bài và có tổng x0 + y0 nhỏ nhất. Ta giả sử x0 6 y0. Xét
phương trình bậc hai ẩn y y2 − A.x0.y + x20 + 30 = 0 (*)
Vì (x0; y0) thỏa mãn đề bài nên y0 là một nghiệm của phương trình (∗). Gọi nghiệm còn lại là y1. Theo định lí Viète ta có y0 + y1 = Ax0 (1) y0y1 = x2 + 30 (2) 0
Ta có x0, y0, A ∈ Z nên từ (1) suy ra y1 ∈ Z. Các cặp (x0; y0) ; (x0; y1) đều thỏa mãn (∗) mà x0 + y0 nhỏ nhất nên ta được x0 + y0 6 x0 + y1 ⇔ y0 6 y1.
Như vậy x0 6 y0 6 y1. Ta xét các trường hợp sau.
• Trường hợp 1. Nếu x0 = y0 khi đó ta thay vào A thì ta được 30 A = 2 + ∈ Z ⇒ x0 = 1 ⇒ A = 32 x20
• Trường hợp 2. Nếu y0 = y1 thì từ (2) ta được x2 ⇔ 0 + 30 = y2 0 (y0 + x0) (y0 − x0) = 30
Dễ thấy y0 + x0; y0 − x0 cùng tính chẵn lẻ mà 30 = 1.30 = 2.15 = 5.6 = 3.10. Trường hợp này không xảy ra.
• Trường hợp 3. Nếu x0 < y0 < y1, thì ta suy ra y0 > x0 + 1 y1 > x0 + 2 Do đó từ (2) suy ra
x20 + 30 > (x0 + 1) (x0 + 2) ⇔ x0 6 9
Khi x0 = 9 thì từ (2) suy ra y0y1 = 92 + 30 = 111, vì y0 < y1 ⇒ (y0; y1) = (1; 111); (3; 37). Điều
này là vô lí vì phải có x0 < y0. Tương tự khi xét x = 1; 2; 3; 4; 5; 6; 7; 8. Tất cả đều dẫn đến vô lí. Trường hợp này loại.
Do đó ta luôn có A = 32 = 25 là lũy thừa bậc năm của một số nguyên. 26 Chinh phục olympic toán
Ứng dụng định lí Viète trong các bài toán số học x2 + y2 + z2
Câu 4. Cho các số nguyên dương x, y, z thỏa mãn
nhận giá trị nguyên dương. xyz + 1 x2 + y2 + z2 Chứng minh rằng
có thể biểu diễn được thành tổng của hai số chính phương. xyz + 1 x2 + y2 + z2 Lời giải. Ta đặt n =
, khi đó ta sẽ chứng minh n là tổng của hai số chính phương. Viết xyz + 1
lại đẳng thức trên thành x2 + y2 + z2 = n (xyz + 1). Giả sử (x0; y0; z0) là một bộ số nguyên dương
thỏa mãn yêu cầu bài toán, điều đó có nghĩa là
x20 + y20 + z20 = n (x0y0z0 + 1) Hay ta viết lại được x2 − − 0 nx0y0z0 + y20 + z20 n = 0
Xét phương trình bậc hai x2 − nxy0z0 + y2 + z2 − n = 0, khi đó ta thấy x 0 0
0 là một nghiệm của phương
trình. Theo định lí Viète thì ngoài nghiệm x0 phương trình còn có một nghiệm nữa, ta gọi nghiệm đó
là x1. Như vậy theo định lí Viète ta có x1 + x0 = ny0z0 x1x0 = y2 + z2 − n 0 0
Từ hệ thức trên ta suy ra được x1 nhận giá trị nguyên. Không mất tính tổng quát ta chọn x0 + y0 + z0
bé nhất và x0 > y0 > z0. Ta xét các trường hợp sau
• Trường hợp 1. Nếu y2 + z2 < n, khi đó x 0 0
1 là số nguyên âm. Từ đó suy ra 0 = x2 − − − 1 ny0z0x1 + y20 + z20 n > x21 + n + y20 + z20 n = x21 + y20 + z20 > 0 điều này vô lí.
• Trường hợp 2. Nếu y2 + z2 > n, khi đó x 0 0
1 là số nguyên dương. Khi đó (x1; y0; z0) là một bộ số
nguyên dương thỏa mãn yêu cầu bài toán. Theo cách chọn (x0; y0; z0) ta suy ra được x0 6 x1.
Khi đó từ định lí Viète ta có y2 − 0 + z2 0
n − ny0z0 = (x0 − 1) (x1 − 1) − 1 > (x0 − 1)2 − 1
Ta lại xét 2 khả năng sau.
1. Nếu x0 > y0 thì ta được (x0 − 1)2 − 1 > y2 − 1. Do đó 0 y2 − − 0 + z2 0 n − ny0z0 > y20 1 Từ đó suy ra z2 + 1 + 1, như vậy ta có n = 1. 0 > ny0z0 + n > n z20
2. Nếu x0 = y0, khi đó ta được 2y2 + z2 = n y2z 0 0 0 0 + 1. Do đó
z20 = y20 (nz0 − 2) + n > z20 (nz0 − 2) + n > z20 (nz0 − 2)
Từ đây ta suy ra được nz0 < 3 nên n = 1 hoặc n = 2, chú ý rằng n = 1 = 02 + 12 và n = 2 = 12 + 12.
• Trường hợp 3. Nếu y2 + z2 = n thì có nghĩa là n viết được thành tổng của hai số chính phương. 0 0
Vậy bài toán được chứng minh xong.
Câu 5. Cho a, b, c là các số nguyên dương thỏa mãn 0 < a2 + b2 − abc 6 c. Chứng minh rằng
a2 + b2 − abc là số chính phương. 27 Hướng tới VMO 2020
Tạp chí và tư liệu toán học
Lời giải. Giả sử tồn tại các số nguyên dương a, b, c thỏa mãn 0 < a2 + b2 − abc 6 c mà k = a2 + b2 − abc
không phải là số chính phương. Khi đó ta có 0 < k 6 c. Không mất tính tổng quát ta giả sử a > b. Xét
phương trình bậc hai ẩn x là x2 − bcx + b2 − k = 0
Khi đó a là một nghiệm của phương, khi đó theo định lí Viète thì phương trình còn có một nghiệm
nữa là x = a1. Từ đó ta được a + a1 = bc a.a1 = b2 − k
Như vậy ta suy ra a1 là số nguyên.
1. Nếu a1 = 0, khi đó từ hệ thức a.a1 = b2 − k ta được k = b2 là số chính phương, điều này mâu
thuẫn với giả sử ở trên.
2. Nếu a1 < 0, khi đó k = a2 + b2 − a
+ b2 + bc > c, mâu thuẫn do 0 < k 1 1bc > a2 1 6 c.
Như vậy ta được a1 là số nguyên dương. Cũng theo định lí Viète ta có  b2 − k   a1 = a ⇒ a b2 − k 1 < a   < a a
Ta thấy cặp số (a1; b) cũng là một nghiệm. Khi đó ta có a1 + b < a + b, điều này sẽ vô lí khi ta chọn
cặp số (a; b) với a + b bé nhất. Như vậy không thể tồn tại k để k = a2 + b2 − abc không phải là số
chính phương. Như vậy a2 + b2 − abc phải là số chính phương.
Câu 6 [Taiwan MO 1998]. Cho m > n là các số nguyên dương lẻ và n2 − 1 chia hết cho
m2 − n2 + 1. Chứng minh rằng m2 − n2 + 1 là một số chính phương.
Lời giải. Theo giả thiết ta có n2 − 1 chia hết cho m2 − n2 + 1 nên ta được m2 − m2 − n2 + 1 chia
hết cho m2 − n2 + 1. Từ đó suy ra m2 − n2 + 1 | m2
Từ điều trên ta suy ra tồn tại k để m2 = k m2 − n2 + 1 m + n m − n 2 m + n m − n
Ở đây ta chú ý rằng m2 = + và m2−n2+1 = 4. . −1. Do m và n là các số 2 2 2 2 m + n m − n m + n m − n
nguyên dương lẻ, lại có m > n nên ;
là các số nguyên dương, đặt x = ; y = , 2 2 2 2 khi đó ta được (x + y)2 = k (4xy + 1)
Bây giờ ta sẽ đi chứng minh rằng 4xy + 1 là số chính phương, tuy nhiên trước tiên ta cần phải chứng
minh k là số chính phương. Thật vậy, giả sử cặp số nguyên dương (x0; y0) với x0 + y0 nhỏ nhất thỏa
mãn thỏa đẳng thức trên, khi đó ta có (x0 + y0)2 = k (4x0y0 + 1)
Xét phương trình bậc hai ẩn x là x2 − (4k − 2) y0x + y2 − k = 0. Khi đó x 0
0 là một nghiệm của phương
trình trên. Như vậy theo định lí Viète thì phương trình còn có một nghiệm nữa là x1, lúc này ta có x0 + x1 = (4k − 2) y0 x0.x1 = y2 − k 0
Từ hệ thức thứ nhất x0 + x1 = (4k − 2) y0 ta suy ra được x1 là số nguyên. Ta xét các trường hợp sau 28 Chinh phục olympic toán
Ứng dụng định lí Viète trong các bài toán số học
• Nếu x1 < 0 thì từ hệ thức thứ hai x0.x1 = y2 − k ta được y2 − k < 0 ⇒ y2 < k, ta suy ra 0 0 0 x2 − − 1 (4k − 2) y0x1 + y20
k = (x1 + y0)2 + k (−4x1 − 1) > 0
điều này mâu thuẫn vì x1 là nghiệm của phương trình.
• Nếu x1 = 0, khi đó từ x0.x1 = y2 − k ta được y2 − k = 0 ⇒ k = y2 là số chính phương. 0 0 0
• Nếu x1 > 0 thì ta được y2 − k > 0 ⇒ k > y2. Khi đó (x 0 0
1; y0) là một nghiệm của phương trình
(x + y)2 = k (4xy + 1). Theo cách chọn cặp số (x0; y0) ta có
x0 + y0 6 x1 + y0 ⇒ y0 6 x0 6 x1
Kéo theo y2 − (4k − 2) y2 + y2 − k = (4 − 4k) y2 − k 0 0 0 0
> 0, điều này vô lí vì k là số nguyên dương.
Vậy ta được k là số chính phương nên dẫn đến m2 − n2 + 1 là số chính phương.
Câu 7. Cho x và y là các số nguyên dương thỏa mãn điều kiện x2 + y2 + 1 chia hết cho 2xy + 1. Chứng minh rằng x = y. x2 + y2 + 1
Lời giải. Do 2xy + 1 | x2 + y2 + 1 nên ta đặt k =
với k là số nguyên dương. Nhận thấy 2xy + 1
khi x = y thì k = 1 và ngược lại vẫn đúng. Do đó ta đi chứng minh k = 1. Giả sử cặp số nguyên dương
(a0; b0) với a0 + b0 bé nhất thỏa mãn yêu cầu bài toán. Không mất tính tổng quát ta giả sử a0 > b0. Xét phương trình x2 + b2 + 1 k = 0 ⇔ x2 − 2kb0x + b2 2xb 0 + 1 − k = 0 0 + 1 Khi đó ta có ∆0 = b2 − 0k2 − b2 0
1 + k = (k − 1) b20 (k + 1) + 1 > 0
Do đó phương trình luôn có nghiệm. Dễ thấy a0 là một nghiệm của phương trình nên theo định lí
Viète thì phương trình còn có thêm một nghiệm nữa là a1. Khi đó ta có a0 + a1 = 2kb0 a0.a1 = b2 − k + 1 0
Từ hệ thức thứ nhất ta được a1 là số nguyên. Giả sử a1 < 0, khi đó a2 − 1
2kb0a1 + b20 + 1 − k > a21 + 2kb0 + b20 + 1 − k > 0
điều này vô lí vì a1 là một nghiệm của phương trình x2 − 2kb0x + b2 + 1 − k = 0, suy ra a 0 1 > 0. Ta xét các trường hợp sau.
• Trường hợp 1. Nếu a1 = 0, khi đó ta được a1b0 = 0, điều này dẫn đến xy = 0.
• Trường hợp 2. Nếu a1 > 0, khi đó nếu a0 > b0 thì ta được b2 − k + 1 a2 − 1 + 1 a 0 0 1 + b0 = + b0 < + b0 = a0 + b0 a0 a0
Đều này mâu thuẫn với các chọn (a0; b0) với a0 + b0 bé nhất. 2a2 + 1 Như vậy a 0 0 = b0, suy ra k =
= 1. Do đó ta được x = y. 2a2 + 1 0
Câu 8. Cho a, b là các số nguyên dương lẻ thỏa mãn a2 + 2 chia hết cho b và b2 + 2 chia hết cho a2 + b2 + 2 a. Chứng minh rằng là số chính phương. ab 29 Hướng tới VMO 2020
Tạp chí và tư liệu toán học
Lời giải. Ta thấy rằng a, b là 2 số nguyên tố cùng nhau. Thật vậy nếu d = (a, b) thì do b | a2 + 2 nên
d | a2 + 2, lại có d | a nên d | 2. Mặt khác a, b là số lẻ nên suy ra d = 1. Do b | a2 + 2 và a | b2 + 2 nên
ab | a2 + 2 b2 + 2 ⇔ ab | 2a2 + 2b2 + 4
Do a và b là số lẻ nên suy ra ab | a2 + b2 + 2. Khi đó tồn tại số nguyên dương k sao cho a2 + b2 + 2 = kab.
Bây giờ ta sẽ đi chứng minh k là số chính phương. Trước tiên thử một vài giá trị đặc biệt ta nhận thấy
được k = 4. Đến đây thì tư tưởng giống như các ví dụ trên đó là sử dụng định lý Viète đển chứng
minh k = 4 thỏa mãn bài toán. Giả sử cặp số dương (a0; b0) với a0 + b0 nhỏ nhất thỏa mãn yêu cầu
bài toán, tức là ta sẽ có a20 + b20 + 2 − ka0b0 = 0
Không mất tính tổng quát ta giả sử a0 > b0. Xét phương trình bậc hai ẩn a a2 − kb0a + b20 + 2 = 0
Khi đó ta thấy a0 là một nghiệm của phương trình trên. Khi đó theo định lí Viète thì phương trình
trên còn có một nghiệm nữa, ta gọi là a1, từ đó ta có a0 + a1 = kb0 a0 + a1 = b2 + 2 0
Từ hệ thức trên ta thu được a1 là số nguyên dương. Như vậy cặp số (a1; b0) cũng thỏa mãn yêu cầu
bài toán. Theo cách chọn cặp số (a0; b0) ta được a0 + b0 6 a1 + b0 ⇒ a0 6 a1 Từ đó ta có a0 k
a0 = kb0 − a1 6 kb0 − a0 ⇒ 6 b0 2
Mặt khác từ điều kiện a2 + b2 + 2 − ka 0 0 0b0 = 0 ta lại được a0 b0 2 + + = k b0 a0 a0b0 a0 k k Do 6 nên k 6
+ 2 + 1 hay k 6 6. Theo bất đẳng thức AM − GM ta có b0 2 2
a20 + b20 > 2a0b0 ⇒ k > 2
Đến đây ta xét các trường hợp sau
1. Nếu k = 3, khi đó ta được a2 + b2 + 2 = 3a
+ b2 + 2 Do số chính phương 0 0 0b0. Từ đó suy ra 3 | a2 0 0
chia 3 dư 1 nên trong hai số chính phương a2 và b2 phải có một số chia hết cho 3 và một số 0 0
không chia hết cho 3. Từ đó suy ra vế phải chia hết cho 9 còn vế trái không chia hết cho 9. Điều
này dẫn đến mâu thuẫn. a0 b0 2 2. Nếu k 6= 4, khi đó từ + +
= k ta được (a0; b0) 6= (1; 1). b0 a0 a0b0 k Như vậy ta được k 6
+ 1 + 1 hay k 6 4, do đó k = 5 và k = 6 không thỏa mãn bà toán. Vậy ta suy 2
ra được chỉ có k = 4 thỏa mãn yêu cầu bài toán.
Câu 9. Cho các số nguyên dương a, b, c, d thỏa mãn b2 + 1 = ac và c2 + 1 = bd. Chứng minh
rằng a + c = 3b và b + d = 3c.
Lời giải. Từ giả thiết ta thấy (b, c) = 1. Thật vậy, gọi (b, c) = k khi đó ta có b = b0k kac0 | k2b02 + 1 ⇒ c = c0k kdb0 | k2c02 + 1 30 Chinh phục olympic toán
Ứng dụng định lí Viète trong các bài toán số học 0 0
Từ đó ta được k2adb0c0 | k2b 2 + 1 k2c 2 + 1 , suy ra 0 0 0 0 0 0 k2 | k2b 2 + 1
k2c 2 + 1 ⇒ k2 | k4b 2c 2 + k2b 2 + k2c 2 + 1 ⇒ k2 | 1 ⇒ k = 1 b2 + 1 = ac b | c2 + 1
Hơn nữa từ giả thiết ta lại có do vậy ta được . c2 + 1 = bd c | b2 + 1
Như vậy ab | b2 + c2 + 1 hay b2 + c2 + 1 = mbc với số nguyên dương m nào đó. Đến đây sử dụng kết
quả của bài toán 4 trong phần bước nhảy Viète ta có được m = 3, khi đó thì b2 + c2 + 1 = 3bc. Từ đây suy ra b2 + 1 = ac b2 + c2 + 1 = c2 + ac ac + c2 = 3bc ⇒ ⇒ c2 + 1 = bd b2 + c2 + 1 = b2 + bc b2 + bd = 3bc a + c = 3b Hay ta được
, vậy bài toán được chứng minh. b + d = 3c
Câu 10. Giả sử phương trình x2 + y2 + x + y + 1 = xyz có nghiệm nguyên dương. Tìm tất cả các giá trị của z.
Lời giải. Trước tiên ta giả sử cặp số nguyên dương (x0; y0) với x0 + y0 nhỏ nhất thỏa mãn thỏa đẳng
thức trên. Không mất tính tổng quát ta giả sử x0 > y0, khi đó ta được
x20 + y20 + x0 + y0 + 1 = zx0y0
Xét phương trình bậc hai ẩn x là
y2 + (1 − zx0) y + x20 + x0 + 1 = 0
Khi đó y0 là một nghiệm của phương trình trên, mà theo hệ thức Viète thì phương trình còn có một
nghiệm nữa là y1. Do đó ta được y0 + y1 = zx0 − 1 y0.y1 = x2 + x 0 0 + 1
Từ các hệ thức trên ta suy ra được y1 có giá trị nguyên dương. Khi đó xặp số nguyên dương (y1; x0) là
một nghiệm của phương trình đã cho. Theo cách chọn cặp số (x0; y0) ta suy ra được y0 6 y1. Ta xét các trường hợp sau.
• Nếu x0 = y0 thì từ x2 + y2 + x 0 0 0 + y0 + 1 = zx0y0 ta suy ra 2x20 + 2x0 + 1 = zx20
Từ đó ta được x0 = y0 = 1 và z = 5.
• Nếu x0 > y0 thì ta được y0 < x0 < y1, khi đó ta được 2 1
x20 + (1 − zx0) x0 + x20 + x0 + 1 < 0 ⇒ x < 2 + + x0 x20
Đến đây ta xét các giá trị x0 > 2 đều không tìm được z thỏa mãn.
Như vậy z = 5 là giá trị thỏa mãn yêu cầu đề bài.
Câu 11. Tìm các số nguyên dương x và y sao cho x + 1 chia hết cho y và y + 1 chia hết cho x.
Lời giải. Do x + 1 chia hết cho y và y + 1 chia hết cho x nên ta được (x + 1) (y + 1) chia hết cho xy. Từ đó suy ra
xy | (x + y + 1) ⇒ xy | (x + y + 1)2 31 Hướng tới VMO 2020
Tạp chí và tư liệu toán học
Đến đây biến đổi tiếp ta được xy | x2 + y2 + 2x + 2y + 1. Khi đó tồn tại số nguyên dương k sao cho
x2 + y2 + 2x + 2y + 1 = kxy. Giả sử cặp số nguyên dương (x0; y0) với x0 + y0 nhỏ nhất thỏa mãn yêu
cầu bài toán. Khi đó ta được
x20 + y20 + 2x0 + 2y0 + 1 − kx0y0 = 0
Không mất tính tổng quát ta giả sử x0 > y0. Xét phương trình bậc hai ẩn x là
x2 + (2 − ky0) x + (y0 + 1)2 = 0
Khi đó x0 là một nghiệm của phương trình. Như vậy theo định lí Viète thì phương trình trên còn có
một nghiệm khác là x1. Do đó ta được x0 + x1 = ky0 − 2 x0.x1 = (y0 + 1)2
Từ hệ thức x0 + x1 = ky0 − 2 ta suy ra được x1 là số nguyên. Từ hệ thức x0.x1 = (y0 + 1)2 ta suy ra
được x1 là số nguyên dương. Chú ý rằng lúc này cặp số nguyên dương (x1; y0) thỏa mãn bài toán. Ta có x0 + y0 6 x1 + y0 ⇒ x0 6 x1
do vậy ta được y0 6 x0 6 x1. Đến đây ta xét các trường hợp sau
1. Nếu x0 = y0, khi đó ta được 1 4 2x2 ⇒ 0 + 4x0 + 1 = kx2 0 k = 2 + + x2 x 0 0
Do k là số nguyên dương nên ta suy ra được x0 = 1 thỏa mãn, do đó suy ra x0 = y0 = 1 thỏa mãn bài toán.
2. Nếu x0 = x1 > y0 > 1, khi đó từ x0.x1 = (y0 + 1)2 ta được x0 = y0 + 1. Thay vào hệ thức
x0 + x1 = ky0 − 2 ta được 4
2x0 = k (x0 − 1) − 2 ⇔ k = 2 + x0 − 1
Do k và x0 là các số nguyên dương nên ta tìm được x0 = 2, x0 = 3 và x0 = 5, tương ứng thì
y0 = 1, y0 = 2 và y0 = 4. Tuy nhiên khi thử vào bài toán thì ta thấy có hai cặp số nguyên dương
thỏa mãn đó là (2; 1) , (3; 2).
3. Nếu x1 > x0 > y0 > 1, khi đó ta được x1 > x0 + 1 > y0 + 2. Từ đó ta được
x0.x1 > (y0 + 2) (y0 + 1) > (y0 + 1)2
Do đó trường hợp này không thỏa mãn.
Vậy các cặp số nguyên dương thỏa mãn bài toán là (x; y) = (1; 1) , (1; 2) , (2; 1) , (2; 3) , (3; 2).
Câu 12. Tìm các số nguyên dương x, y để x2 + 2 chia hết cho xy + 1.
Lời giải. Ta biến đổi giả thiết thành x2 − mxy + 2 − m = 0, khi đó giả sử cặp số nguyên dương (x0; y0)
với x0 + y0 bé nhất thỏa mãn yêu cầu bài toán, tức là ta có x2 + 2 = m (x 0 0y0 + 1). Xét phương trình bậc hai ẩn x là x2 − mxy0 − m + 2 = 0
Khi đó x0 là một nghiệm của phương trình trên. Theo định lí Viète thì phương trình trên còn có một
nghiệm nữa, gọi nghiệm đó là x1. Khi đó ta có x0 + x1 = my0 x0.x1 = 2 − m 32 Chinh phục olympic toán
Ứng dụng định lí Viète trong các bài toán số học
Từ hệ thức x0 + x1 = my0 ta suy ra x1 là số nguyên. Mặt khác nếu x1 < 0, khi đó ta được x1 6 −1. Do đó x2 − 1
mx1y0 − m + 2 > x21 + my0 − m + 2 > 0
Điều này mâu thuẫn với x1 là nghiệm của phương trình trên. Như vậy x1 > 0 hay x1 là số nguyên
không âm. Do đó từ hệ thức x0.x1 = 2 − m ta được
2 − m > 0 ⇒ m ∈ {1; 2}
Ta xét các trường hợp sau
1. Nếu m = 1, khi đó ta được x (x − y) = −1 nên ta được x = 1; y = 2 thỏa mãn yêu cầu bài toán.
2. Nếu m = 2, khi đó x (x − 2y) = 0. Lại do x 6= 0 nên suy ra x − 2y = 0 ⇔ x = 2y. Do đó
x = 2k; y = k với k là số nguyên dương thỏa mãn yêu cầu bài toán.
Vậy cặp số nguyên dương thỏa mãn là (x; y) = (1; 2) , (2k; k) với k là số nguyên dương.
Câu 13. Tìm tất cả các số có ba chữ số chia hết cho 11 sao cho thương số của phép chia số đó
cho 11 bằng tổng bình phương của các chữ số của số đó.
Lời giải. Gọi số có ba chữ số thỏa mãn yêu cầu bài toán là A = abc. Trong đó các chữ số thỏa mãn
a ∈ {1; 2; ...9} ; b, c ∈ {0; 1; 2; ...9}
Do A chia hết cho 11 nên ta được a−b+c chia hết cho 11. Kết hợp với a ∈ {1; 2; ...9} ; b, c ∈ {0; 1; 2; ...9}
ta suy ra được a − b + c = 0 hoặc a − b + c = 11. Như vậy ta đi xét hai trường hợp sau
• Với a − b + c = 0, khi đó ta được b = a + c. Ta có A = 100a + 10b + c = 99a + 10b + a + c = 99a + 11b.
Khi A chia 11 thì thương số của phép chia bằng tổng bình phương các chữ số của A nên ta được
A = a2 + b2 + c2 ⇔ 9a + b = a2 + b2 + c2 11
Kết hợp với b = a + c ta được
9a + (a + c) = a2 + (a + c)2 + c2 ⇔ 10a + c = 2a2 + 2ac + 2c2 Do a > 1 nên suy ra 25
10a + c > 2a2 + 2c + 2c2 ⇒ 2c2 + c 6 10a − 2a2 6 2
Do vậy mà 2c2 + c 6 12 ⇒ c 6 2. Cũng từ 10a + c = 2a2 + 2ac + 2c2 ta suy ra được c là số chẵn.
Từ đó ta được c = 0 hoặc c = 2. Ta xét 2 khả năng.
1. Với c = 0, khi đó ta được a = b nên số cần tìm có dạng A = aa0. Do đó
A = 50 = 2a2 ⇒ a = 5 ⇒ a = b = 5 11
Từ đó ta tìm được A = 550.
2. Với c = 2, khi đó từ 10a + c = 2a2 + 2ac + 2c2 ta được
10a + 2 = 2a2 + 4ac + 8 ⇔ a2 − 3a + 3 = 0
Nhận thấy phương trình trên không có nghiệm nguyên dương nên không tồn tại số A thỏa mãn bài toán. 33 Hướng tới VMO 2020
Tạp chí và tư liệu toán học
• Với a − b + c = 11, khi đó ta được b + 11 = a + c. Do a, b, c là các chữ số nên từ b + 11 = a + c ta
suy ra được a > 2. Ta có
A = 100a + 10b + c = 99a + 10b + a + c = 99a + 11b + 11 Ta xét các khả năng sau.
1. Xét a = 2, khi đó c = 9; b = 0. Ta được A = 209 không thỏa mãn bài toán.
2. Xét a = 3, khi đó ta được c = 8; b = 0 hoặc c = 9; b = 1. Ta được A = 308 hoặc A = 319 không thỏa mãn.
3. Xét a > 4, khi đó A chia 11 thì thương số của phép chia bằng tổng bình phương các chữ số của A nên ta được
A = a2 + b2 + c2 ⇔ 9a + b + 1 = a2 + b2 + c2 11
Kết hợp với b = a + c − 11 ta được
9a + (a + c − 11) + 1 = a2 + (a + c − 11)2 + c2
⇔ 10a + c − 10 = 2a2 + 2ac + 2c2 − 22 (a + c) + 121
⇔ 32a + 23c − 131 = 2a2 + 2ac + 2c2 Do a > 4 nên suy ra
32a + 23c − 131 > 2a2 + 8c + 2c2 ⇒ 2c2 − 15c 6 32a − 2a2 − 131 6 −3
Do đó suy ra 2c2 − 15c 6 −3 ⇒ c 6 7. Từ 32a + 23c − 131 = 2a2 + 2ac + 2c2 ta suy ra
được c là số lẻ. Do đó ta được c = 1; 3; 5; 7. Đến đây xét các trường hợp của c thì được b = 0; a = 8 thỏa mãn.
Do đó số cần tìm là A = 803.
Vậy các số thỏa mãn yêu cầu bài toán là 550 và 803.
Câu 14 [Kiran Kedlaya]. Cho các số nguyên dương a, b, c là thỏa mãn (ab + 1) (bc + 1) (ca + 1)
là số chính phương. Chứng minh rằng ba số ab + 1; bc + 1; ca + 1 đều là số chính phương.
Lời giải. Từ giả thiết ta nhận thấy để chứng minh ab + 1; bc + 1; ca + 1 đều là số chính ta cần chỉ ta
được ba số ab + 1; bc + 1; ca + 1 nguyên tố với nhau theo từng đôi một. Tuy nhiên với lượng thông
tin hạn chế từ giả thiết ta không thể chứng minh được nhận định trên. Với ý tưởng sử dụng định lí
Viète, ta cần tạo ra một phương trình bậc hai có nghiệm nguyên và có biệt thức ∆ có chứa biểu thức
(ab + 1) (bc + 1) (ca + 1). Ngoài ra do ta cần chứng minh ab + 1; bc + 1; ca + 1 đều là số chính phương
nên khi biến đổi phương trình bậc hai thì cần chứa các đại lượng ab + 1; bc + 1; ca + 1. Xét phương trình bậc hai ẩn t sau
t2 + a2 + b2 + c2 − 2 (ab + bc + ca + ta + tb + tc) − 4abct − 4 = 0
⇔ t2 − 2t (a + b + c + 2abc) + a2 + b2 + c2 − 2 (ab + bc + ca) − 4 = 0
Ta nhận thấy phương trình bậc hai trên tương đương với ba phương trình sau
(a + b − c − t)2 = 4 (ab + 1) (ct + 1)
(a + c − b − t)2 = 4 (ac + 1) (bt + 1)
(b + c − a − t)2 = 4 (bc + 1) (at + 1)
Giải phương trình bậc hai trên ta được
t1 = a + b + c + 2abc + 2p(ab + 1) (bc + 1) (ca + 1)
t2 = a + b + c + 2abc − 2p(ab + 1) (bc + 1) (ca + 1) 34 Chinh phục olympic toán
Ứng dụng định lí Viète trong các bài toán số học
Do (ab + 1) (bc + 1) (ca + 1) là số chính phương nên t nhận các giá trị nguyên. Từ ba phương trình trên ta được
43 (ab + 1) (ac + 1) (bc + 1) (ct + 1) (bt + 1) (at + 1)
là số chính phương, do đó (ct + 1) (bt + 1) (at + 1) là số chính phương, ta lại có
at + 1 > 0; bt + 1 > 0; ct + 1 > 0
nên at + 1 > 0; bt + 1 > 0; ct + 1 > 0. Trong các bộ số nguyên dương (a; b; c) thỏa mãn bài toán ta xét
bộ số (a; b; c) sao cho a + b + c nhỏ nhất. Không mất tính tổng quát ta chọn c = max {a; b; c}. Dễ thấy −1
a = b = c = 1 không thỏa mãn bài toán nên c = max {a; b; c} > 1, do đó t > > −1. Ta max {a; b; c} xét các trường hợp sau
1. Nếu t = 0, khi đó từ phương trình bậc hai ta được
(a + b + c)2 = 4 (ab + bc + ca) + 4 ⇔ (a + b − c)2 = 4 (ab + 1)
Suy ra ab + 1 là số chính phương. Chứng minh hoàn toàn tương tự ta được bc + 1; ca + 1 là các số chính phương.
2. Nếu t > 0, khi đó do a + b + c có giá trị nhỏ nhất nên ta được t > c và t chỉ nhận một trong hai giá trị
t1 = a + b + c + 2abc + 2p(ab + 1) (bc + 1) (ca + 1)
t2 = a + b + c + 2abc − 2p(ab + 1) (bc + 1) (ca + 1)
Mà theo định lí Viète ta có
t1t2 = a2 + b2 + c2 − 2 (ab + bc + ca) − 4 6 c2 − a (2c − a) − b (2c − b) < c2
Điều này dẫn đến mâu thuẫn. Nên trường hợp này không có giá trị t thỏa mãn.
Như vậy bài toán đã được giải quyết hoàn toàn.
Câu 15. Tồn tại hay không năm số nguyên dương a1; a2; a3; a4; a5 thỏa mãn hệ điều kiện  a2 + 1 = (a1 + 1) (a3 + 1)  2 a2 + 1 = (a 3 2 + 1) (a4 + 1)  a2 + 1 = (a 4 3 + 1) (a5 + 1)
Lời giải. Trước hết ta kiểm tra tính chẵn lẻ của các số đã cho trước khi có các đánh giá hợp lí. Giả
sử a1 là số lẻ, khi đó a2 là số lẻ nên a2 + 1 chia 4 dư 2, từ đó suy ra a 2
3 + 1 là số lẻ, dẫn đến a3 là số
chẵn. Điều này vô lý vì a2 + 1 là số chẵn nên không thể là ước của số lẻ a2 + 1. Do vậy a 3 1 phải là số
chẵn. Lập luận hoàn toàn tương tự ta được a2; a3; a4; a5 cùng là số chẵn. Đặt a2 = x; a3 = y. Khi đó . .
từ hệ điều kiện trên ta được y2 + 1 .. (x + 1) và x2 + 1 .. (y + 1). Ta sẽ chứng minh rằng không tồn
tại cặp số chẵn x và y thỏa mãn điều trên. . .
Giả sử tồn tại cặp số nguyên dương (x, y) thỏa mãn y2 + 1 .. (x + 1) và x2 + 1 .. (y + 1), khi đó ta được . .
y2 + 1 + x2 − 1 .. (x + 1) ⇒ x2 + y2 .. (x + 1) .
Tương tự ta cũng có x2 + y2 .. (y + 1). Gọi d = (x + 1, y + 1), khi đó d là ước của x2 + 1; y2 + 1; x2 + y2. Do đó . .
x2 + 1 + y2 + 1 − x2 + y2 ..d ⇒ 2..d
Như vậy thì d = 1 hoặc d = 2. Mà x + 1; y + 1 là các số lẻ, do đó ta được d = 1 hay x + 1; y + 1 nguyên
tố cùng nhau. Từ đó dẫn đến . x2 + y2 .. (x + 1) (y + 1) 35 Hướng tới VMO 2020
Tạp chí và tư liệu toán học
Khi đó tồn tại số nguyên dương k sao cho x2 + y2 = k (x + 1) (y + 1)
Trong các cặp số nguyên dương (x; y) thỏa mãn ta chọn cặp số nguyên dương (x0; y0) với x0 + y0 bé
nhất. Không mất tính tổng quát ta giả sử x0 > y0. Xét phương trình bậc hai ẩn x là x2 − k (y0 + 1) x + y2 − 0 k (y0 + 1) = 0
Khi đó x0 là một nghiệm của phương trình. Theo định lí Viète thì còn có một nghiệm nữa là x1. Khi đó ta có x0 + x1 = k (y0 + 1) x0x1 = y2 − k (y 0 0 + 1) . Nếu x .
1 = 0, khi đó từ x0x1 = y2 − k (y = k (y . (y 0 0 + 1) ta được y2 0 0 + 1) nên suy ra y2 0 0 + 1), điều này
vô lí vì y0 và y0 + 1 nguyên tố cùng nhau. Do đó x1 6= 0. Mặt khác ta có
(x0 + 1) (x1 + 1) = x0x1 + x0 + x1 + 1 = y20 + 1
nên x1 + 1 là số lẻ. Do đó x1 > 0 là số chẵn. Đồng thời ta cũng có y2 + 1 y2 + 1 x 0 0 1 + 1 = 6 6 y0 6 x0 x0 + 1 y0 + 1
Như vậy cặp số (y0; x1) là một nghiệm của x2 + y2 = k (x + 1) (y + 1)
Cặp số (y0; x1) thỏa mãn y0 + x1 6 y0 + x0, điều này vô lí vì ta đã chọn x0 + y0 bé nhất. Vậy điều
ta giả sử là sai hay không tồn tại các số chẵn x, y thỏa mãn. Như vậy không tồn tại năm số nguyên
dương thỏa mãn yêu cầu bài toán.
Nhận xét. Từ bài toán này, ta có thể giải được các bài toán tương tự sau.
1. Cho các số nguyên dương x, y thỏa mãn x2 + 1 chia hết cho y + 1 và y2 + 1 chia hết cho
x + 1. Chứng minh rằng x và y là các số lẻ.
2. Tìm số nguyên dương k để phương trình x2 + y2 = k (x + 1) (y + 1) có nghiệm nguyên ! dương.
3. Tìm số tự nhiên n bé nhất sao cho tồn tại số nguyên dương a1; a2; ...; an thỏa mãn a2k = (ak−1 + 1) (ak+1 + 1)
với các số 2 6 k 6 n − 1. 6 Bài tập tự luyện.
Câu 1. Tìm tất cả các số tự nhiên a, b, c sao cho tồn tại số nguyên dương n, m, k thỏa mãn các điều kiện sau m2 + b n2 + c k2 + a a = ; b = ; c = 2m 2n 2k
Câu 2. Tìm nghiệm nguyên của phương trình sau x3 + x2y + xy2 + y3 = 8 x2 + xy + y2 + 1.
Câu 3. Tìm nghiệm nguyên của phương trình 3 x2 − xy + y2 = 7 (x + y).
Câu 4 [Putnam 1998]. Chứng minh rằng với mỗi số thực N thì phương trình
x21 + x22 + x23 + x24 = x1x2x3 + x1x2x4 + x1x3x4 + x2x3x4
có nghiệm (a1, a2, a3, a4) với a1, a2, a3, a4 là các số nguyên lớn hơn N .
Câu 5. Giả sử bốn số nguyên a, b, c, d đôi một khác nhau và thoả mãn hệ điều kiện sau
a2 − 2ac − 5d = b2 − 2bc − 5d = 0
c2 − 2ca − 5b = d2 − 2bd − 5b = 0 36 Chinh phục olympic toán
Ứng dụng định lí Viète trong các bài toán số học
Chứng minh rằng a + b + c + d là một hợp số. √ √ √
Câu 6. Tìm các số nguyên dương x, y sao cho x + y − x − y + 2 = 0.
Câu 7 [Turkey National Olympiad 2015]. Với m, n là các số nguyên dương sao cho (m + n)2 k = 4m(m − n)2 + 4
cũng là số nguyên. Chứng minh rằng k là số chính phương.
Câu 8. Cho p là một số nguyên dương. Giả sử phương trình x2 + px + 1 = 0 có hai nghiệm là a1; a2
và phương trình x2 + qx + 1 = 0 có hai nghiệm b1; b2. Chứng minh rằng
(a1 − b1) (a2 − b1) (a1 + b2) (a2 + b2)
là hiệu của hai số chính phương.
Câu 9. Tìm các cặp số nguyên (a; b) sao cho hai số a2 + 4b và b2 + 4a đều là số chính phương. √
Câu 10. Tìm các chữ số a, b, c, d, e thỏa mãn điều kiện ab + cde = abcde.
Câu 11. Cho a, b là các số nguyên dương thỏa mãn a2 + b2 chia hết cho ab. Tính giá trị của biểu thức a2 + b2 A = ab
Câu 12 [Đề thi trường Đông phía Bắc 2015]. Tìm tất cả các số nguyên dương k sao cho phương
trình x2 − (k2 − 4)y2 + 24 = 0 có nghiệm nguyên dương.
Câu 13. Chứng minh rằng tất cả các nghiệm nguyên dương của phương trình x2 + y2 + 1 = 3xy là
(x, y) = (F2k−1, F2k+1) với Fn là số Fibonacci.
Câu 14. Tìm tất cả các số nguyên dương n sao cho phương trình sau có nghiệm nguyên dương x2 + y2 = n(x + 1)(y + 1).
Câu 15. Giả sử a, b là các số nguyên dương thỏa mãn b + 1 | a2 + 1, a + 1 | b2 + 1. Chứng minh rằng a, b đều là các số lẻ. a2 + b2 + 6
Câu 16. Chứng minh rằng nếu a, b là các số nguyên dương sao cho k = nguyên thì k = 8. ab a + 1 b + 1
Câu 17. Chứng minh rằng có vô số cặp số nguyên dương (a; b) thỏa mãn + = 4. b a CHÚC CÁC BẠN THI TỐT! 37 Hướng tới VMO 2020
Tạp chí và tư liệu toán học Tài liệu
[1] Bước nhảy Viète - Hà Tuấn Dũng, Đại học Sư phạm Hà Nội 2.
[2] Bước nhảy Viète - Phạm Huy Hoàng, Chuyên đề số học Mathscope.
[3] Đặng Hùng Thắng, Nguyễn Văn Ngọc, Vũ Kim Thủy - Bài giảng số học. NXB Giáo dục 1996.
[4] Vận dụng định lí Viète giải các bài toán số học - Nguyễn Công Lợi.
[5] Lời giải và bình luận VMO 2012 - Trần Nam Dũng. Diễn đàn Mathscope, 2012.
[6] The Method of Vieta Jumping - Yimin Ge, Mathematical Reflections 5 (2007).
[7] A Rational Function Whose Integral Values Are Sums of Two Squares - Sam Vandervelde.
[8] Diễn đàn AoPS Online, https://artofproblemsolving.com/community.
[9] Diễn đàn toán học Việt Nam - VMF, https://diendantoanhoc.net/. 38
Document Outline

  • Nhà toán học Francois Viète.
  • Định lý Viète.
  • Các bài toán cơ bản.
  • Phương pháp bước nhảy Viète - Vieta Jumping.
  • Các bài toán tổng hợp.
    • Đề bài
    • Hướng dẫn giải - Lời giải
  • Bài tập tự luyện.